Ky Yeu Mon Toan Duyen Hai 2010

You might also like

Download as pdf or txt
Download as pdf or txt
You are on page 1of 132

HI CC TRNG THPT CHUYN

KHU VC DUYN HI V NG BNG BC B




K YU
HI THO KHOA HC, LN TH III

MN TON HC
(TI LIU LU HNH NI B)












H NAM, THNG 11 NM 2010


=========================================================== 4
HI CC TRNG THTP CHUYN KHU VC DUYN HI V NG BNG BC B

Hi tho khoa hc mn Ton hc ln th III - 2010
MC LC

STT NI DUNG TRANG
1
LI NI U
5
2
MT S DNG PHNG TRNH V T CHO HC SINH GII
Nguyn Anh Tun (THPT chuyn Bc Giang)
6
3
LM NGC BT NG THC
Nguyn c Vang (THPT chuyn Bc Ninh)
27
4
CHNG MINH BT NG THC BNG CCH S DNG BT
NG THC SP XP LI V BT NG THC CHEBYSHEV
o Quc Huy, T Ton Tin, Trng THPT Chuyn Bin Ha H Nam
31
5
TNH TUN HON TRONG DY S NGUYN
Ng Th Hi, trng THPT chuyn Nguyn Tri, Hi Dng
43
6
NH L PASCAL V NG DNG
L c Thnh, THPT Chuyn Trn Ph Hi Phng
47
7
HM S HC V MT S BI TON V HM S HC
Trng THPT Chuyn Hng Yn
56
8
MT S BI TON S HC TRONG CC K THI OLYMPIC TON
Trn Xun ng (THPT Chuyn L Hng Phong Nam nh)
67
9
NH L LAGRANGE V NG DNG
ng nh Sn, Chuyn Lng Vn Ty Ninh Bnh
73
10
T S KP V PHP CHIU XUYN TM
Trng THPT chuyn Thi Bnh Thi Bnh
93
11
MT S DNG TON V DY S V GII HN
Trn Ngc Thng - THPT Chuyn Vnh Phc
105
12
S DNG CNG C S PHC GII CC BI TON HNH HC
PHNG
Trng THPT chuyn H Long
123
13
BT BIN TRONG CC BI TON L THUYT TR CHI
Phm Minh Phng, trng THPT chuyn i hc S phm H Ni
130





=========================================================== 5
HI CC TRNG THTP CHUYN KHU VC DUYN HI V NG BNG BC B

Hi tho khoa hc mn Ton hc ln th III - 2010
















DI TRUYN HC
















LI NI U



Hi cc trng chuyn vng Duyn Hi Bc B n nay c 12 trng
tham gia. Trong c nhiu trng c truyn thng lu nm, c thnh tch
cao trong cc k thi hc sinh gii Quc gia v Quc t mn Ton.
Nm nay, ln th 3 hi tho khoa hc. Vi cng v l n v ng cai,
chng ti nhn c 12 bi vit v cc chuyn chuyn su cho hc
sinh gii Ton. l cc chuyn tm huyt ca cc thy c dy chuyn
Ton ca cc trng chuyn trong hi.
Xin trn trng gii thiu cc bi vit ca cc thy c trong k yu mn
Ton ca hi trong dp hi tho khoa hc ln th 3. Hy vng rng cun k
yu ny s mt ti liu tham kho cho cc thy c!

T TON - TIN
TRNG THPT CHUYN BIN HO - H NAM

=========================================================== 6
HI CC TRNG THTP CHUYN KHU VC DUYN HI V NG BNG BC B

Hi tho khoa hc mn Ton hc ln th III - 2010
MT S DNG PHNG TRNH V T CHO HC SINH GII
Nguyn Anh Tun (THPT chuyn Bc Giang)


Li m u

Ton hc c mt v p li cun v quyn r, ai am m th mi mi am m
Trong v p y huyn b th cc bi ton lin quan n Phng trnh v t (cha cn
thc) - c nt p tht s xao xuyn v quyn r.

C l v l do m trong cc k thi HSG cc nc, thi HSG Quc gia (VMO) ca
chng ta, bi ton lin quan n Phng trnh v t thng c mt thch thc cc nh
Ton hc tng lai vi dung nhan mun hnh, mun v. Ri th cn trong cc k thi HSG
cp tnh, thi HSG cp thnh ph, thi i hc, thi
Tht l iu th v !

Chuyn : Mt s dng phng trnh v t cho hc sinh gii ti vit vi mong
mun phn no gip cc Thy c gio dy Ton, cc em hc sinh ph thng trong cc i
tuyn thi hc sinh gii Ton c th tm thy nhiu iu b ch v nhiu iu th v i vi
dng ton ny. Trong Chuyn c c nhng bi vi cp gii tr cho hc sinh gii (rn
luyn phn x nhanh).

i vi vic gii phng trnh v t th hu ht cc phng php gii, cc phng
php bin i hay u c trong cun Chuyn ny. Cch phn tch nhn dng mt
phng trnh v chn la phng php gii thch hp l kh v a dng. c kh nng ny
chng ta phi gii quyt nhiu phng trnh v t rt ra nhng nhn xt, kinh nghim v hay
hn na l mt vi thut gii ton, cng nh lu rng mt bi ton c th c nhiu cch
gii khc nhau.

Ti vit Chuyn ny vi mt tinh thn trch nhim cao. Ti hy vng rng Chuyn
s li trong lng Thy c v cc em hc sinh mt n tng tt p.
Vi mi v d trong tng phng php gii, ngi c c th t sng tc cho mnh
nhng bi ton vi nhng con s m mnh yu thch. Tuy nhin Chuyn chc chn s
khng th trnh khi nhng iu khng mong mun. Ti rt mong nhn c s ng vin
v nhng kin ng gp chn thnh ca Qu Thy c v cc em hc sinh Chuyn
tip tc c hon thin hn.

Ti xin chn thnh cm n!




=========================================================== 7
HI CC TRNG THTP CHUYN KHU VC DUYN HI V NG BNG BC B

Hi tho khoa hc mn Ton hc ln th III - 2010
1. MT S PHNG PHP GII PHNG TRNH V T

1. MT S QUY C KHI C CHUYN
1.1 Vt: V tri ca phng trnh. Vt
2
: Bnh phng ca v tri phng trnh.
1.2 Vp: V phi ca phng trnh. Vp
2
: Bnh phng ca v phi phng trnh.
1.3 Vt (1) : V tri ca phng trnh (1) .
1.4 Vp (1) : V phi ca phng trnh (1) .
1.5 k, k: iu kin.
1.6 BT: Bt ng thc. HSG, HSG: Hc sinh gii.
1.7 VMO, VMO: Thi hc sinh gii Vit Nam, CMO: Thi hc sinh gii Canada.

2. PHNG PHP T N PH

2.1 Mt s lu
Khi gii phng trnh v t bng phng php t n ph ta c th gp cc dng
nh:
2.1.1 t n ph a phng trnh cho v phng trnh i s khng cn cha
cn thc vi n mi l n ph.
2.1.2 t n ph m vn cn n chnh, ta c th tnh n ny theo n kia.
2.1.3 t n ph a phng trnh v h hai phng trnh vi hai n l hai n ph,
cng c th hai n gm mt n chnh v mt n ph, thng khi ta c mt h i xng.
2.1.4 t n ph c phng trnh c hai n ph, ta bin i v phng trnh
tch vi v phi bng 0.
Thng gii phng trnh ta hay bin i tng ng, nu bin i h qu th nh
phi th li nghim.

2.2 Mt s v d

V d 1. Gii cc phng trnh sau:
1)
2
18 18 17 8 2 0 x x x x x = .
2)
2 4 2
3
3 1 1
3
x x x x + = + + .
3)
2
2
1 1
2 2 4 x x
x x
| |
+ = +
|
\
.
4)
2 2
2 1 2 1 1 x x x x + + = .

Hng dn (HD): 1) t x y = vi 0 y . Khi phng trnh cho tr thnh
2 2
(3 4 2)(6 2 1) 0 y y y y + + = , suy ra
2
(3 4 2) 0 y y = , ta c
2 10
3
y
+
= . T
phng trnh c nghim l
14 4 10
9
x
+
= .
2) Ta c
4 2 2 2 2 2 2
1 ( 1) ( 1)( 1) 0 x x x x x x x x + + = + = + + + > , vi mi x.
Mt khc
2 2 2
3 1 2( 1) ( 1) x x x x x x + = + + + .
t
2
2
1
1
x x
y
x x
+
=
+ +
(c th vit k 0 y hoc chnh xc hn l
3
3
3
y ), ta c

=========================================================== 8
HI CC TRNG THTP CHUYN KHU VC DUYN HI V NG BNG BC B

Hi tho khoa hc mn Ton hc ln th III - 2010
2 2
3
2 1 0 6 3 3 0
3
y y y y = = + = , ta c
3
3
y = (loi
3
2
y = ).
T phng trnh c nghim l 1 x = .
3) Ta thy 0 x < khng tha mn.
Khi phng trnh tng ng vi h
2
2
2
2
0
1
4 0
1 1
2 2 4
1
x
x
x
x x
x

>

| |
+ >

|
\

| |
| | | |

+ = +
|
| |
|

\ \
\
.
t
1
x y
x
+ = , ta c
2 2 2
2 4(1)
4 ( 2) 2 5 2( 2) (4 ) (2)
y
y y y
<

+ =

.
Xt
2 2
(2) 9 2 4 5 y y y = +
4 3 2
8 28 40 16 0 y y y y + + = (do hai v khng
m).

3 2
2
( 2)( 6 16 8) 0
( 2)(( 2)( 4 8) 8) 0
y y y y
y y y y
+ =
+ + =

Dn n 2 y = (do
2
(( 2)( 4 8) 8) 0 y y y + + > vi mi y tha mn (1)).
T phng trnh c nghim l 1 x = .
Nhn xt: Bi ton ny ta c th gii bng Phng php nh gi trong phn sau.
4) Ta c phng trnh tng ng vi
2 2
1 1 2 2 1 x x x x =
4 2 2 2 2 3 2
1 1 4 4 (1 ) 4 4 1 8 1 x x x x x x x x x = + + +

2 2 2
2 2 2
(1 4 1 8 1 ) 0
0
1 4 1 8 1 0(1)
x x x x
x
x x x
+ =
=


+ =


Xt (1), t
2
1 y x = , suy ra 0 y v
2 2
1 x y = .
Ta c
2 3
1 4 8 (1 ) 0 8 4 1 0 y y y y y + = =

2
(2 1)(4 2 1) 0 y y y + =

1 5
4
y
+
= . T suy ra
5 5
8
x

= .
Th li ta c nghim ca phng trnh l 0 x = v
5 5
8
x

= .

Nhn xt: Bi ton ny ta c th gii bng Phng php lng gic trong phn sau.
V d 2. Gii phng trnh
2 2
3 1 ( 3) 1 x x x x + + = + + .
HD: t
2
1 x y + = , vi 1 y . Khi ta c
2
3 ( 3) y x x y + = +
( 3)( ) 0 y y x = .
Dn n 3 y = v y x = . T phng trnh c nghim l 2 x = .

=========================================================== 9
HI CC TRNG THTP CHUYN KHU VC DUYN HI V NG BNG BC B

Hi tho khoa hc mn Ton hc ln th III - 2010

V d 3. Gii phng trnh
8 3 8 4
17 2 1 1 x x = .

HD: t
8 4
17 x y = vi 0 y v
3 8
2 1 x z = . Khi ta c h
4 3 4 3
1 1
2 33 2 ( 1) 33
y z z y
y z y y
= =


+ = + =

.
Xt
4 3 3 2
2 ( 1) 33 ( 2)(2 5 7 17) 0 y y y y y y + = + + + = .
Suy ra c y - 2 = 0. T nghim ca phng trnh l x = 1 v x = -1.

V d 4. Gii cc phng trnh sau:
1)
2 2
4 2 3 4 x x x x + = + .
2)
3 2 3
4
81 8 2 2
3
x x x x = + .

HD: 1) t
2
4 x y = , vi 0 2 y .
Khi ta c h
2 2
2 3
4
x y xy
x y
+ = +

+ =

.
Th hoc li t ; x y S xy P + = = ri gii tip ta c nghim ca phng trnh l
0 x = ; 2 x = v
2 14
3
x

= .
2) t
3 2 3
4
81 8 2 3 3 2
3
x y x y y y + = = + .
Khi ta c h
3 2
3 2
4
3 2
3
4
3 2
3
x y y y
y x x x

= +

= +

.
Xt hiu hai phng trnh dn n x y = (do
2 2 2
1 1 1 1
( ) ( 2) ( 2) 0
2 2 2 3
x y x y + + + + > ).
Thay vo h v gii phng trnh ta c
3 2 6
0;
3
x x

= = .

V d 5. Gii phng trnh
2 2
5 14 9 20 5 1 x x x x x + + = + .

HD: k 5 x . Vi iu kin ta bin i phng trnh cho nh sau:
2 2
2 2
5 14 9 20 5 1
5 14 9 20 25( 1) 10 ( 1)( 4)( 5)
+ + = + +
+ + = + + + + +
x x x x x
x x x x x x x x

2
2 5 2 5 ( 1)( 5) 4 + = + + x x x x x
2( 1)( 5) 3( 4) 5 ( 1)( 5) 4 + + + = + + x x x x x x
t ( 1)( 5) ; 4 x x y x z + = + = , vi 0; 3 y z .

=========================================================== 10
HI CC TRNG THTP CHUYN KHU VC DUYN HI V NG BNG BC B

Hi tho khoa hc mn Ton hc ln th III - 2010
Ta c
2 2
2 3 5 ( )(2 3 ) 0 y z yz y z y z + = = , t ta c
3
2
y z
y z
=

.
Nu y z = th ta c
5 61
2
x
+
= (do 5 x ).
Nu
3
2
y z = th ta c
7
8;
4
x x = = . Vy phng trnh c ba nghim trn.

V d 6. Gii phng trnh
2
4 9
7 7
28
x
x x
+
+ = , vi 0 x > .
Nhn xt: Dng phng trnh ny ta thng t
4 9
28
x
ay b
+
= + , sau bnh
phng ln ri ta c bin i v h i xng vi hai n , x y . T ta s bit c gi
tr ca a, b. Vi bi ton ny ta tm c
1
1;
2
a b = = . (Nu a = 1 v b = 0 m gii c th
l phng trnh qu n gin, ta khng xt y).
HD: t
4 9 1
28 2
x
y
+
= + , do 0 x > nn
4 9 9 1
28 28 2
x +
> > , t 0 y > .
Ta c h
2
2
1
7 7
2
1
7 7
2
, 0
x x y
y y x
x y

+ = +

+ = +

>

. Gii h bnh thng theo dng ta c


6 50
14
x
+
= .

V d 7. Gii phng trnh
3 2 3
2 2 x x = .

Nhn xt: Khi gii mt phng trnh khng phi lc no cng c nghim thc, c
nhng phng trnh v nghim nhng khi cho hc sinh lm bi ta cng kim tra c nng
lc ca hc sinh khi trnh by li gii bi ton . Chng hn nh bi ton trong v d ny.
HD: t
3 2 3
2 2 x x = = y vi 0 y . Khi ta c h
2 3
3 2
2
2
x y
x y
= +

v t
phng trnh ban u ta c 2 x . Xt hiu hai phng trnh ca h ta c phng trnh
2 2
( )( ) 0 x y x xy y x y + + + = .
Vi x y = th
3 2
2 x x = , dn n v nghim.
Cn
2 2 2
( )(1 ) 0 x xy y x y y x x y + + = + > vi mi 0 y v 2 x . Do h
v nghim hay phng trnh cho v nghim.
2.3 Mt s bi tp tng t

Bi 1. Gii cc phng trnh sau:
1)
2 2
2 2 2 x x x x + = .

(HD: t 2 ; 0 y x y = , ta c
2 2
( 1)( 1)(2 4) 0 y y y y y + = .

=========================================================== 11
HI CC TRNG THTP CHUYN KHU VC DUYN HI V NG BNG BC B

Hi tho khoa hc mn Ton hc ln th III - 2010
T
5 1 33 1
1; ;
2 8
y y y
+
= = = v c nghim ca phng trnh l
5 1 33 1
1; ;
2 8
x x x
+ +
= = = ).
2)
2 3
2 5 1 7 1 x x x + = .
(HD: T phng trnh suy ra 1 x . t
2
1
1
x x
y
x
+ +
=

, bnh phng dn n
3 2 3 y + . Phng trnh tr thnh
2
2 7 3 0 y y + = , ta c 3 y = . T 4 6 x = ).

Bi 2. Gii phng trnh
2 2
(4 1) 1 2 2 1 x x x x + = + + .

(HD: t
2
1 x y + = , vi 1 y . T ta c
1
2 1
2
y y x = = . Phng trnh c
nghim
4
3
x = ).

Bi 3. Gii cc phng trnh sau:
1) 3(2 2) 2 6 x x x + = + + .

(HD: t 3 2 , 6 x y x z = + = , vi 0; 0 y z .
Ta c 3 4 x y z = + = . T phng trnh c 2 nghim
11 3 5
3;
2
x x

= = ).
2)
4
2 2(1 ) 2 1 x x + + = .

(HD: k 0 2 1 x . t
4
2 2(1 ) 2 2 1 x y y x + = =
v
4 4 4
2 2 x z z x = = vi 0; 0 y z .
Suy ra
4
2 4
2( ) 1(1)
2 1(2)
y z
y z

+ =

+ =

. T (1) thay
4
1
2
y z = vo (2) ta c
2 2 2
4
1
( 1) ( ) 0
2
z z + + = . Xt hiu hai bnh phng suy ra
4
4 3 2
1
4 2
2
z

= .
T ta c nghim ca phng trnh l
4
4
4
4 3 2
1
2
2
x
| |

|
|
=
|
|
|
\
).
Bi 4. Gii phng trnh
2
1000 1 8000 1000 x x x + = .

(HD: t 1 1 8000x + + =2y , ta c
2
2
2000
(*)
2000
x x y
y y x
=

.

=========================================================== 12
HI CC TRNG THTP CHUYN KHU VC DUYN HI V NG BNG BC B

Hi tho khoa hc mn Ton hc ln th III - 2010
T (*) suy ra ( )( 1999) 0 x y x y + + = v , do 1999 0 x y + + > .
Suy ra x y = , ta c nghim 2001 x = , loi 0 x = ).

Bi 5. Gii cc phng trnh sau:
1)
3
2
1 2
2 5
x
x
+
=
+
.

(HD: t
2
1 0; 1 y x z x x = + = + , ta c
2
2 2
5
5 2( ) 2 2
y y
yz y z
z z
| |
= + = +
|
\
2
5 1
2 2 0 2
2
y y y y
z z z z
| |
+ = = =
|
\
.
Nu 2
y
z
= ta c
2
1 2 1 x x x + = +
2
1
4 5 3 0
x
x x


+ =

(v nghim).
Nu
1
2
y
z
= ta c
2
2 1 1 x x x + = +
1
5 37
5 37
2
2
x
x
x


=

(tha mn)).
2)
2 3
2 5 2 4 2( 21 20 x x x x + = .

(HD: k
4 1
5
x
x

. t
2
2 8 10 x x y = v 4 x z + = , vi 0; 0 y z .
Khi ta c ( )( 3 ) 0 y z y z = . T phng trnh c bn nghim l
9 193
4
x

=
v
17 3 73
4
x

= ).

Bi 6. Gii cc phng trnh sau:
1)
2
4 3 5 x x x = + .

(HD: t 5 2 x y + = , ta c
5 29
1;
2
x x
+
= = ).
2)
2
3
2 4
2
x
x x
+
+ = , vi 1 x .
(HD: t
3
1
2
x
y
+
= + ,c
3 17
1
4
x
+
= < (loi), nu 1 x th
3 17
4
x
+
= ).
3)
2
4
27 18
3
x x x + = + , vi 0 x > .
(HD: Tng t, ta c
5 37
18
x
+
= ).
3. PHNG PHP NH GI

3.1 Mt s lu
Khi gii phng trnh v t (chng hn ( ) ( ) f x g x = ) bng phng php nh gi,
thng l ta ch ra phng trnh ch c mt nghim (nghim duy nht).Ta thng s dng

=========================================================== 13
HI CC TRNG THTP CHUYN KHU VC DUYN HI V NG BNG BC B

Hi tho khoa hc mn Ton hc ln th III - 2010
cc bt ng thc c in C si, Bunhiacopxki, a v tri v tng bnh phng cc biu
thc, ng thi v phi bng 0. Ta cng c th s dng tnh n iu ca hm s (c th
thy ngay hoc s dng o hm xt s bin thin ca hm s) nh gi mt cch hp l.
Thng ta nh gi nh sau:
( ) ( )
( ) ( ) ( ) ( )
( ) ( )
f x g x
f x C C f x g x C
g x C C
=

= =

, hoc nh gi
( ) ( ) f x g x cng nh l ( ) ( ) f x g x
Ngoi ra i vi bi c th no ta s c cch nh gi khc.
Cng c mt s phng trnh v t c nhiu hn mt n m ta gii bng phng php
nh gi.

3.2 Mt s v d

V d 1. Gii phng trnh
2
4 1 4 1 1 x x + = .

HD: Bi ton ny c trong thi vo i hc Bch Khoa v HQG nm 2001. Bi
ny c nhiu cch gii, p n s dng o hm.
Ta c th lm n gin nh sau: Ta thy
1
2
x = l nghim ca phng trnh.
Nu
1
2
x > th Vt > 1 = Vp.
Nu
1
2
x < th Vt < 1 = Vp.
Do phng trnh khng c nghim trong hai trng hp ny.
Vy phng trnh c mt nghim l
1
2
x = .

V d 2. Gii phng trnh
2 2 2
3 6 7 5 10 14 4 2 x x x x x x + + + + + = .
HD: Bi ny qu n gin, nh gi Vt 5 cn Vp 5 , do hai v cng bng 5.
Ta c phng trnh c nghim duy nht l 1 x = .
V d 3. Gii phng trnh
2 2 2
19 7 8 13 13 17 7 3 3( 2) x x x x x x x + + + + + + + = + .

HD: Bi ny cch gii c v hi mt t nhin bi cch c cho nh vy. Gio vin
v hc sinh c th sng tc nhng bi kiu .
k 2 x . Vi k Vt =
2 2 2 2 2
1 75 1 3
( ) (2 1) 3( 2) (2 1) (4 3)
2 4 4 4
x x x x x + + + + + + +

75 3
3 2 4 3
4 2
x x + + + +


5 3
3 3( 2) (4 3)
2 2
x x + + + +
3 3.( 2) x + = Vp.
Du ng thc xy ra khi
1
2
x = . Vy phng trnh c nghim duy nht l
1
2
x = .

=========================================================== 14
HI CC TRNG THTP CHUYN KHU VC DUYN HI V NG BNG BC B

Hi tho khoa hc mn Ton hc ln th III - 2010

V d 4. Gii phng trnh
2
4
28 27
2 27 24 1 6
3 2
x x x + + = + + .

HD: Phng trnh cho tng ng vi phng trnh
2
4
(9 4) 3(9 4)
2 4 1
3 2
x x + +
+ = + , k
4
9
x . t (9 4) x y + = , suy ra 0 y .
Khi ta c
2 2
4
3 3
2 4 1 4 4 1 6
3 2 3 2
y y y y
y + = + + = + + (bnh phng hai v).
Theo BT C-si ta c
6
6
2
y
y
+
, do
2 2
2
4 4 2 4 4 4 ( 2)
3 3
y y
y y
| |
+ + + +
|
\


2 2
2
2
4 48 3 12 12
12 36 0
( 6) 0.
y y y
y y
y
+ + +
+


T ta c 6 y = , suy ra
2
9
x = tha mn k.
Vy phng trnh c nghim duy nht l
2
9
x = .

V d 5. Gii phng trnh
2
4 3 2
3
2 7 3 3 2
2
x x
x x x x

+ + + = .

HD: Phng trnh cho tng ng vi
2 2 2
2 2
3 4 (2 1) ( 3)
(2 1)( 3) (1)
2 2
x x x x x
x x x
+ + + +
+ + = = . Phng trnh xc nh
vi mi x l s thc. Theo BT C-si cho hai s dng ta c Vt(1) Vp(1).
Do (1)
2 2 2
2 1 3 2 0 x x x x x + = + = . T phng trnh c nghim l
1 x = v 2 x = .

V d 6. Gii phng trnh
2
2
1 1
2 2 4 x x
x x
| |
+ = +
|
\
.
HD: k
2
2
2
2
2
2
x
x

. Vi k , phng trnh cho tng ng vi


phng trnh
2
2
1 1
2 2 4(1) x x
x x
+ + + = .

=========================================================== 15
HI CC TRNG THTP CHUYN KHU VC DUYN HI V NG BNG BC B

Hi tho khoa hc mn Ton hc ln th III - 2010
Theo BT Bunhiacopxki, ta c
2 2 2 2
2 2
2 2
( 2 ) ( 2 .1 .1) 4
1 1 1 1
2 2 .1 .1 4
x x x x
x x x x

+ = +

| | | |
+ = +
| |
| |
\ \
.
Suy ra Vt (1) 4 = Vp(1) . Do
2
2
2 2
(1)
1 1
2 2
x x
x x

+ =


+ =

, ngha l du bng trong h


xy ra. T phng trnh c nghim duy nht l 1 x = .

V d 7. Gii phng trnh
2 2
9
1
x x
x
+ = +
+
.

HD: k 0 x .
Theo BT Bunhiacopxki, ta c
2
Vt =
2
1 1
2 2 1 ( 9)
1 1 1 1
x x
x x
x x x x
| |
| |
+ + + + =
|
|
|
+ + + + \
\
2
Vp .
Phng trnh c nghim khi du ng thc xy ra hay
1
2 2
1
1
1
x
x x
x
+
=
+
+
1
7
x = .
Vy phng trnh c nghim duy nht l
1
7
x = .

V d 8. Gii phng trnh
2 4 2 4
13 9 16 x x x x + + = .

HD: k 1 1 x .
Vi k phng trnh tng ng vi
2 2 2 2 2 2
(13 1 9 1 ) 16 (13 1 9 1 ) 256(1) x x x x x x + + = + + =
Theo BT Bunhiacopxki, ta c
2 2 2 2 2 2
(13 1 9 1 ) ( 13. 13 1 3. 3. 3 1 ) x x x x + + = + +

2 2
2
(13 27)(13(1 ) 3(1 ))
40(16 10 ).
x x
x
+ + +
=

Theo BT C-si cho hai s dng ta c
2
2 2
2 2
10 (16 10 )
10 (16 10 ) 64
2
x x
x x
| | +
=
|
\
.
Do Vt(1) 4 64 256 . = , ta c
(1)
2
2 2
2
2
2 2
1
9 9 1
1
3
20 16
10 16 10
x
x x
x
x
x x

+
= +
=


=

. T dn n
2 5
5
x = .
Vy phng trnh c hai nghim l
2 5
5
x = .

=========================================================== 16
HI CC TRNG THTP CHUYN KHU VC DUYN HI V NG BNG BC B

Hi tho khoa hc mn Ton hc ln th III - 2010

V d 9. Gii phng trnh
3 2 3
2 2 x x = .

Nhn xt: Trong phn gii phng trnh v t bng Phng php t n ph ta
gii bi ton ny, ta cng c th gii n bng phng php nh gi nh sau.
HD: k
3 3
2 0 2 x x .
Gi s x l nghim ca phng trnh. Khi
2
2 0 x
2
2
x
x

, ta c 2 x .
M 6 hai v suy ra
9 6 4 3 2
6 12 4 4 0 x x x x x + + = (*).
Cch th nht ta bin i Vt thnh
9 6 2 4 2 3 2
5 ( 1) 12 3 4 x x x x x x x + + l mt biu
thc m khi 2 x .
Cch th hai ta bin i Vt thnh
9 4 2 3 2
(6 1) 12 4 4 x x x x x + cng l mt biu thc
m khi 2 x
Ta c th bin i tip phng trnh (*) sau khi chia hai v cho 1 0 x , ta c

8 7 6 5 4 3 2
5 5 4 8 4 4 0 x x x x x x x x + + + + + =
6 2 4 2 2
( 1) 5 ( 1) 4 ( 1) 4(2 1) 0 x x x x x x x x + + + + + = v nghim v Vt lun dng
khi 2 x . Vy phng trnh v nghim.

V d 10. Gii phng trnh ( 2)(2 1) 3 6 4 ( 6)(2 1) 3 2 x x x x x x + + = + + + .

HD: Bin i phng trnh thnh ( 6 2)( 2 1 3) 4 x x x + + + = , suy ra 5 x .
Vt l hm s ng bin trn on [ ) 5; + . T dn n 7 x = l nghim duy nht ca
phng trnh cho.

V d 11. Gii phng trnh
2 3
2 11 21 3 4 4 0 x x x + = .

HD: Phng trnh tng ng vi

2 3 3
12( 3)
( 3)(2 5)
(4 4) 2 4 4 4
x
x x
x x

=
+ +
.
Ta thy 3 x = l nghim ca phng trnh.
Nu 3 x th phng trnh tng ng vi
2 3 3
12
(2 5) (1)
(4 4) 2 4 4 4
x
x x
=
+ +

Nu 3 x > th Vt(1) > 1 > Vp(1).
Nu 3 x < th Vt(1) < 1 < Vp(1).
Vy phng trnh c nghim duy nht l 3 x = .

V d 12. Gii phng trnh
2 2 2 2
2 1 3 2 2 2 3 6 x x x x x x x + + = + + + + .

Nhn xt: Vi bi ton ny ta s dng mt nh gi t gp sau y:
( ) 0; ( ) 0
( ) ( ) ( ) ( ) ( ) ( )
( ) 0
f x g x
f x g x f x ah x g x bh x
h x

+ = + + +

=

, vi a, b l hai
s thc dng.
HD: Bin i phng trnh

=========================================================== 17
HI CC TRNG THTP CHUYN KHU VC DUYN HI V NG BNG BC B

Hi tho khoa hc mn Ton hc ln th III - 2010
2 2
2 2 2 2
2 1 0; 3 2 0
2 1 3 2 2 1 2( 2) 3 2 2( 2)
2 0
x x x
x x x x x x x x
x
+
+ + = + + + + + +

+ =


T ta c phng trnh c nghim l 2 x = .

V d 13. Gii phng trnh
16 1
10 ( 1996 2008)
1996 2008
x y
x y
+ = +

.

Nhn xt: Vi bi ton ny, ta thy y l mt phng trnh gm hai n. Do ta
ngh n bin i phng trnh thnh phng trnh mi c Vt l tng cc bnh phng, cn
Vp bng 0.
HD: Bin i phng trnh thnh

2
2
4
4
4
4
4 1
1996 2008 0
1996 2008
x y
x y
| |
| |
+ =
|
|
|

\
\
.
T ta c phng trnh c nghim l ( ; ) (2012; 2009) x y = .

V d 14. Gii phng trnh
3
1 2 1
2
x y y x xy + = .

HD: k 1; 1 x y .
Ta c
1 3
1 2 1 ( 2 1) ( 2 1)
2 2
x y y x y x x x y y xy + = +

2 2
1 3
( 1 1) ( 1 1)
2 2
y x x y xy = + .
Khi phng trnh cho tng ng vi
2 2
1; 1
1
( 1 1) ( 1 1) 0
2
x y
y x x y

+ =

.
T ta c phng trnh c nghim l ( ; ) (2; 2) x y = .

3.3 Mt s bi tp tng t: (Chuyn cn tip tc hon thin)
4. PHNG PHP LNG GIC

4.1 Mt s lu
Khi gii phng trnh v t bng phng php lng gic ta c th t
( ) sin f x = nu [ ] ( ) 1;1 f x vi iu kin ;
2 2

(

(

hoc ( ) cos f x = vi iu
kin
[ ] 0; . Cng c khi t ( ) tan ; ( ) cot f x f x = = a phng trnh cho
v phng trnh lng gic. Gii phng trnh lng gic ri t tm nghim ca phng
trnh cho.

4.2 Mt s v d

V d 1. Gii phng trnh
2
4 1 4 1 1 x x + = .


=========================================================== 18
HI CC TRNG THTP CHUYN KHU VC DUYN HI V NG BNG BC B

Hi tho khoa hc mn Ton hc ln th III - 2010
Nhn xt: Bi ton ny ( xt trn) cng c th gii bng phng php lng gic,
tuy nhin vi bi ny cch gii bng lng gic ch mang tnh cht tham kho.
HD: t
4
2 4
4 1 cos
; 0;
2
4 1 sin
x y
y
x y

=
(

(

=

. Khi ta c phng trnh


8 4 2
2 6 4 2
cos 2cos 8cos 7 0
( 1)(...) 0
(cos 1)(cos cos cos 7) 0
cos 1
y y y
cosy
y y y y
y
+ =
=
+ + =
=

Do vy phng trnh c mt nghim l
1
2
x = .

V d 2. Gii phng trnh
2
1 1
2 2
1
x
x
+ =

.
HD: t cos , (0; ),
2
x y y y

= . Phng trnh cho tr thnh
1 1
2 2 sin cos 2.sin 2
cos sin
y y y
y y
+ = + = . t sin cos , 2 2 y y z z + = .
suy ra
2
sin 2 2sin cos 1 y y y z = = , ta c 2 z = v
2
2
z = .
Vi 2 z = th
4
y

= , do
2
2
x = .
Vi
2
2
z = th
11
12
y

= , do
1 3
2 2
x
+
= .
Vy phng trnh c nghim l
2
2
x = v
1 3
2 2
x
+
= .

V d 3. Gii phng trnh
3 2 3 2
(1 ) 2(1 ) x x x x + = .

HD: k 1 1 x .
t sin , ;
2 2
x y y
(
=
(

suy ra cos 0 y .
Khi phng trnh tr thnh
3 3
sin cos 2 sin cos y y y y + = .
t sin cos , 2; 2 y y z z
(
+ =

(chnh xc l 1; 2 z
(


), bin i phng trnh
ta c
3 2
2. 3 2 0 z z z + = ( 2)( 2 1)( 2 1) 0 z z z + + + =
2 1 2 z z = = .
Nu 2 z = th th
4
y

= , do
2
2
x = .
Nu 1 2 z = th sin cos 1 2 y y + =
2
1 1 2 x x + =

=========================================================== 19
HI CC TRNG THTP CHUYN KHU VC DUYN HI V NG BNG BC B

Hi tho khoa hc mn Ton hc ln th III - 2010

2
1 1 2 0
1 2 2 2 1
2
x x
x
=

=

Vy phng trnh c 2 nghim trn.

4.3 Mt s bi tp tng t

Bi 1. Gii phng trnh
3 2
4 3 1 x x x = .
(HD: t cos x y = , phng trnh c tp nghim l
5 3 2
cos ; cos ; cos
8 8 4 2
S



= =
`

)
).
Bi 2. Gii phng trnh
( )
2 6 2 3
5 3 1 8 (1 ) x x x + = + .

Bi 3. Gii phng trnh
2
2 2
1
x
x
x
+ =

.

Bi 4. Gii phng trnh
2 2
( 3 2 ) 1 3 2 x x x x = .

Bi 5. Gii phng trnh
2
2
2
(1 )
3 1
1
x x
x
x
+
=

.

Bi 6. Gii phng trnh
2 3
2
5 3
(1 )
1
6 20 6
x
x
x x x
+
= +
+
.

Bi 7. Gii phng trnh
2 2
2 1 2 1 1 x x x x + + = .
5. MT S PHNG PHP KHC

5.1 Mt s lu
Ngoi nhng phng php thng gp trn, i khi ta cng c nhng li gii khc
l i vi mt s phng trnh v t. Cng c th ta s dng kt hp cc phng php trn
gii mt phng trnh.

5.2 Mt s v d
V d 1. Gii phng trnh
2 2
3 2. 9 4 2. 16 5 x x x x + + + = .

HD: Nu 0 x th Vt 3 4 7 5 + = > = Vp (phng trnh khng c nghim).
Nu 0 x > th ta xt tam gic vung ABC vi
0
90 A = , AB = 4; AC = 3.
Gi AD l phn gic ca gc A, ly M thuc tia AD.
t AM = x, xt
2 2
9 3 2. ACM CM x x = + v xt
2 2
16 4 2. ABM BM x x = + .
T suy ra Vt = 5 CM BM BC + = . Du ng thc xy ra khi M D ,hay

=========================================================== 20
HI CC TRNG THTP CHUYN KHU VC DUYN HI V NG BNG BC B

Hi tho khoa hc mn Ton hc ln th III - 2010

2 2
2 2
3
4
1 6 9
1 6 1 6 . 9 4 8 2 . 9 1 6 . 9 3 6 2 .
7 1 2 2 . 0
1 2 2
7
C M
B M
C M B M
x x x x
x x
x
=
=
+ = +
=
=

Vy phng trnh c nghim l
12 2
7
x = .

V d 2. Gii phng trnh
2 2 2 4 4
4 4 1 2 3 5 16 x x x y y y x + + + + = + .

Nhn xt: Bi ton ny khng kh, ch kim tra tnh cn thn ca hc sinh m thi v
sau khi t iu kin tm c gi tr ca x. Tuy nhin nu hc sinh hc hi ht s ngi
nhn m khng lm c bi.
HD: t k cho phng trnh xc nh ta s c 2 x = . Khi phng trnh tr
thnh 1 2 y y = , suy ra
3
2
y = . Vy phng trnh c mt nghim l
3
( ; ) 2;
2
x y
| |
=
|
\
.
V d 3. Gii phng trnh
3 2 3 2 3
7 1 8 8 1 2 x x x x x + + = .

HD: t
3 2 3 2 3
7 1; 8; 8 1 y x z x x t x x = + = = ,
suy ra 2 y z t + + = v
3 3 3
8 y z t + + = (1).
Mt khc ( )
3
8 y z t + + = (2).
T (1) v (2) ta c
3 3 3 3
( ) ( ) 3( )( )( ) 0 y z t y z t y z z t t y + + + + = + + + =

0 (3)
0 (4)
0 (5)
y z y z
z t z t
t y t y
+ = =

+ = =

+ = =

.
Xt (3) ta c 1 9 x x = = , xt (4) c 1 x = v (5) c 0 1 x x = = .
Vy tp nghim ca phng trnh l { } 1; 0;1; 9 S = .

V d 4. Gii phng trnh
2 2
4 20 4 29 97 x x x x + + + + = .

HD: Trong mt phng ta xt hai vc t ( 2; 4) a x =
r
v ( 2; 5) b x =
r
.
Khi ta c ( 4; 5) a b + =
r r
, suy ra 97 a b + =
r r
v ta cng c
2
4 20 a x x = +
r
,
2
4 29 b x x = + +
r
. Phng trnh tr thnh a b a b + = +
r r r r
, ng thc xy ra khi a
r

v b
r
cng chiu
2 2
4 5
x x
= . T ta c phng trnh c mt nghim l
2
9
x = .

V d 5. Gii phng trnh
2 2 4 2
1 2 1 2 2( 1) (2 4 1) x x x x x x x + + = + .
HD: t
2 2
2 1 ( 1) y x x x = = , suy ra
2 2
0 1
( 1) 1
y
x y

.

=========================================================== 21
HI CC TRNG THTP CHUYN KHU VC DUYN HI V NG BNG BC B

Hi tho khoa hc mn Ton hc ln th III - 2010
Ta c
2 2 2
1 1 2(1 ) (1 2 )(1) y y y y + + = .
Mt khc
2 2
1 1 1 1 2 (2) y y y y + + + .
T (1) v (2), suy ra
2 2 2 2
2(1 ) (1 2 ) 2 y y y
t
2
y z = , ta c 0 1 z v
2 2
2(1 ) (1 2 ) 2 (4 10 7) 0 z z z z z z +
0 z (do
2
4 10 7 0 z z + > ).
Do 0 z = , suy ra 0 y = hay
2
2 0 x x =
0
2
x
x
=

.
Vy phng trnh c nghim l 0 x = v 2 x = .

2. MT S BI TON THI LP I TUYN HC SINH GII TNH
BC GIANG

Chn i tuyn ca tnh Bc Giang thi hc sinh gii quc gia cng c nhng bi ton
gii phng trnh v t. Sau y l mt s bi.

Bi 1 (Lp i tuyn HSG quc gia tnh Bc Giang nm hc 2004 2005)
Gii phng trnh
3 2 3 3
2 11 4 4 14 5 13 2 x x x x x x + + = + .

Bi 2 (Kim tra i tuyn HSG quc gia tnh Bc Giang nm hc 2004 2005)
Gii phng trnh
3 2 3 3 3 2
2 2 3 1 2 3 1 x x x x x x + + = .

Bi 3 (Lp tin i tuyn HSG quc gia tnh Bc Giang nm hc 2006 2007)
Gii phng trnh
4
8 4 2 3 3 x x x x + + + = + + .

Bi 4 (D tuyn ton QG gi B GD-T ca Bc Giang nm hc 2006 2007)
Gii phng trnh
2 2 2
2 3 2 1 3 3 x x x x x x + = + + .

Bi 5. (Kim tra i tuyn HSG quc gia tnh Bc Giang nm hc 2007 2008)
Gii phng trnh
2
2
2007 2008 2009
2007
x x x
x x
+
=
+
.

Bi 6. (Gio s dy i tuyn ton tnh Bc Giang nm hc 2004 2005)
Gii cc phng trnh sau:
1)
2
1 3 2 1 x x x x + + = + . 4)
2
1 5
8
2
x
x
+ = .

2)
3 4
7 80 x x x + + = + . 5)
4
3
2
8
x x = + .

3)
3 3
1 2(2 1) x x + = . 6)
2 3
2 4 3 4 x x x x + + = + .






=========================================================== 22
HI CC TRNG THTP CHUYN KHU VC DUYN HI V NG BNG BC B

Hi tho khoa hc mn Ton hc ln th III - 2010
3. MT S BI TON THI HC SINH GII CA MT S QUC GIA

Thc t bi ton gii phng trnh v t trong k thi hc sinh gii quc gia l khng
kh. Tuy nhin lm c vic ln th trc ht phi lm tt vic nh, do hc sinh
mun ot gii t khuyn khch tr ln phi lm tt bi ton ny. D bit vy nhng khng
phi hc sinh xut sc no cng vt qua c.

Bi 1 (1995 - Bng A. VMO)
Gii phng trnh
3 2 4
3 8 40 8 4 4 0 x x x x + + = .

HD: k 1 x .
Khi xt
3 2
( ) 3 8 40 f x x x x = + v
4
( ) 8 4 4 g x x = + trn on [ ) 1; + .
Ta c ( ) ( ) f x g x = . p dng BT C-si cho bn s khng m, ta c
4 4 4 4 4 4
4
1
( ) 2 .2 .2 (4 4) (2 2 2 (4 4)) 13(1)
4
g x x x x = + + + + + = + . ng thc xy ra khi v
ch khi
4
4 4 2 3 x x + = = .
Mt khc
3 2 2
3 8 40 13 ( 3)( 9) 0 x x x x x x + +

2
( 3) ( 3) 0(2) x x + .
ng thc xy ra khi v ch khi 3 x = .
T (1) v (2), ta c ( ) 13 ( ) g x x f x + . C hai ng thc u xy ra khi 3 x = , tha
mn iu kin.
Vy phng trnh c nghim duy nht l 3 x = .

Nhn xt: Ta c th s dng o hm xt s bin thin ca cc hm s ( ) f x v
( ) g x trn on [ ) 1; + , ta c
[ ) 1:
min ( ) (3) 13 f x f
+
= = v
[ ) 1:
max ( ) (3) 13 g x g
+
= = .
Hoc ta c th t
4
4 4 x y + = , vi 0 y sau dng o hm kho st s bin
thin ca hm s
12 8 4
( ) 24 16 512 2816 f y y y y y = + + ( '( ) 2( 2). ( ) f y y h y = vi
( ) 0 h y > ).
Bi 2 (1995 - Bng B. VMO)
Gii phng trnh
2 3
2 11 21 3 4 4 0 x x x + = .

HD: t
3
4 4 x y = .
Khi
3
4
4
y
x
+
= v suy ra
6 3
2
8 16
6
y y
x
+ +
= . T ta c phng trnh
6 3 3 6 3
1 11
( 8 16) ( 4) 3 21 0 14 24 96 0(1)
8 4
y y y y y y y + + + + = + =

2 4 3 2
( 2) ( 4 12 18 14) 0(2) y y y y y + + + + = .
Do 0 y th Vt(1) dng, do ta xt 0 y > , khi
4 3 2
4 12 18 14 0 y y y y + + + + > .
Nn t (2) ta thy 2 y = hay
3
4 4 2 x = , ta c 3 x = .Th li ng.
Vy phng trnh c nghim duy nht l 3 x = .
Bi 3 (2002 - Bng A. VMO)
Gii phng trnh 4 3 10 3 2 x x = .
HD: Cch 1 (p n)

=========================================================== 23
HI CC TRNG THTP CHUYN KHU VC DUYN HI V NG BNG BC B

Hi tho khoa hc mn Ton hc ln th III - 2010
k
74 10
27 3
x . Vi iu kin phng trnh cho tng ng vi phng trnh

2 2 2
4 3 10 3 4 4 9(10 3 ) (4 ) x x x x x x = + =

4 3 2
2
8 16 27 29 0
( 3)( 2)( 7 15) 0
x x x x
x x x x
+ + =
+ + =

3 x = (do k v
2
7 15 0 x x + > vi mi x tha mn k)
Vy phng trnh c nghim duy nht l 3 x = .
Cch 2: t 10 3x y = , suy ra
4
0
3
y (1) v
2 2
10 4
2 0
3 3
y y
x x

= = >
vi mi y tha mn (1).
Khi ta c
2 4 2
4 8 16
4 3 4 3
3 9
y y y
y y
+
= =

4 3
2
8 27 20 0
( 1)( 4)( 3 5) 0
y y y
y y y x
+ =
+ + =

1 y = .
Hay ta c 10 3 1 x = 3 x = .
Vy phng trnh c nghim duy nht l 3 x = .

Bi 4 (1998-CMO)
Gii phng trnh
1 1
1 x x
x x
= + .

Nhn xt: y l bi ton thi hc sinh gii ca Canada, c th ni l n gin, nh
nhng vi hc sinh tinh nhng cng y cm by vi mi hc sinh.
Tht vy, t k xc nh ca phng trnh ta phi dn n c 1 x > .
Vi k , phng trnh tng ng vi
1 1
1 x x
x x
=


2 2
1 1
1 x x
x x
| | | |
=
| |
| |
\ \
(do hai v khng m vi mi
1 x > )

2 2
( 1) 2 ( 1) 0 x x x x + =

2 2
( 1 ) 0 x x =

2
1 0 x x = . T suy ra
1 5
2
x
+
= .
Cng c th t
2 2
( 1) 2 ( 1) 0 x x x x + = , chuyn
2
2 ( 1) x x sang v phi ri bnh
phng hai v, sau t
1
2
x y = ta c phng trnh trng phng n
1
2
y > , gii
phng trnh ny tm c
5
2
y = . T suy ra
1 5
2
x
+
= nhng cch ny hi di.
Vy phng trnh c nghim duy nht l
1 5
2
x
+
= .

=========================================================== 24
HI CC TRNG THTP CHUYN KHU VC DUYN HI V NG BNG BC B

Hi tho khoa hc mn Ton hc ln th III - 2010
4. MT S BI TP T LM

Sau y l mt s bi tp t lm m chng ta c th s dng cc phng php trn.

Bi 1. Gii cc phng trnh sau:
1)
2 2 2
1 1 2 x x x x x x + + + = + .
2)
2 2
1 1 (1 2 1 ) x x x + = + .
3)
2
2
1 2
1
x x x
x x
+
=
+
.
4)
2
2 4 2 5 1 x x x x + = .
5)
3 2 3 2 3 3
3 2001 3 7 2002 6 2003 2002 x x x x x + + = .

Bi 2. Gii cc phng trnh sau:
1)
2 2 2
2 3 2 1 3 3 x x x x x x + = + + .
2)
42 60
6
5 7 x x
+ =

.
3) ( 2) 1 2 2 0 x x x + = .
4)
3 3 3 3
3 1 5 2 9 4 3 0 x x x x + + + = .
5)
2 2
4 4 10 8 6 10 x x x x = .

Bi 3. Gii cc phng trnh sau:
1)
2
(2004 )(1 1 ) x x x = + .
2) 3 3 x x x = + .
3) 5 5 x x x x = .
4)
4 3 3
16 5 6 4 x x x + = + .
5)
3 2 3
3 2 ( 2) 6 0 x x x x + + = .

Bi 4. Gii cc phng trnh sau:
1)
2 3
5 1 9 2 3 1 x x x x + = + .
2)
2
4
28 27
2. 27 24 1 6
3 2
x x x + + = + + .
3) 13 1 9 1 16 x x x + + = .
4)
3 3
86 5 1 x x + = .
5)
3 2 3
2 ( 4) 7 3 28 0 x x x x x + = .

Bi 5. Gii cc phng trnh sau:
1)
2 2
2
2 2 2 2
x x
x x
+
+ =
+ +
.
2)
2
2 2 4 4 2 9 16 x x x + + = + .

=========================================================== 25
HI CC TRNG THTP CHUYN KHU VC DUYN HI V NG BNG BC B

Hi tho khoa hc mn Ton hc ln th III - 2010
3)
2 3
2 5 2 4 2( 21 20) x x x x + = .
4)
3
3 2 x x x = + .
5)
2
4 3 2 3
1
2 2 2 1 ( )
x
x x x x x x
x

+ + + = + .

Bi 6. Gii cc phng trnh sau:
1)
3 3 3
6 6 6 x x + + = .
2)
4 1 5
2 x x x
x x x
+ = + .
3)
2 4 3 2
2 4 7 4 3 2 7 x x x x x x + + = + + .
4)
2 2 4 6
1 1 1 1 x x x x + + + = .
5)
2
2
2
1
3
x x
| |
=
|
\
.

Bi 7. Gii cc phng trnh sau:
1)
( ) ( )
2 2
3 2 1 1 1 3 8 2 1 x x x x + = + + + .
2)
2 3
2( 2) 5 1 x x + = + .
3)
6 4 2 2
64 112 56 7 2 1 x x x x + = .
4)
( )
2 3 3 2
1 1 (1 ) (1 ) 2 1 x x x x + + = + .
5)
( )
2
2 3 3
2 1
1 1 (1 ) (1 )
3 3
x
x x x

+ + = + .

Bi 8. Gii cc phng trnh sau:
1)
3 3
6 6 4 4 0 x x + = .
2)
2 3
2( 3 2) 3 8 x x x + = + .
3)
6 2 3 3
1 1 1 x x x + = .
4)
2 2 3
15 3 8 2 x x x + = + + .
5)
2 3 3 2 4
4 4 4
(1 ) (1 ) 1 (1 ) x x x x x x x x + + = + + .

Bi 9. Gii cc phng trnh sau:
1)
3 3
1 3 3 1 x x + = .
2)
2
35
12
1
x
x
x
+ =

.
3)
2 3
2 11 21 3 4 4 0 x x x + = .
4)
4 3 2 2
4 6 4 2 10 2 x x x x x x + + + + + + = .
5)
2 2 2
2 2 2
32
1 1 4 4
(2 3)
x x x x x
x x
+ + + + =
+
.

Bi 10. Gii cc phng trnh sau:

=========================================================== 26
HI CC TRNG THTP CHUYN KHU VC DUYN HI V NG BNG BC B

Hi tho khoa hc mn Ton hc ln th III - 2010
1)
2
3
1
1
x
x
x
+ =
+
.
2) ( 1) 1 5 1 4 4 0 x x x x + + = .
3)
4 2 2 2
10 14 19 (5 38) 2 x x x x + = .
4)
2 2
( 1) 2 3 1 x x x x + + = + .
5)
2 2
1
1 1 2
2
x x x = .

Bi 11. Gii cc phng trnh sau:
1)
1 3
1 0
4 2
x
x x
+
=
+ +
.
2)
3
3 2 0 x x x + = .
3)
3 3
8 4 6 1 1 0 x x x + = .
4)
( )
2 2 2
3 2 2 2 1 0 x x x x + + + = .
5)
2 2
3 5 12 5 0 x x x + + + = .

Bi 12. Gii cc phng trnh sau:
1)
2 3
2( 8) 5 8 x x + = + .
2)
2
4 3 4 3 10 3 x x x = .
3) ( 3) (4 )(12 ) 28 x x x x + + = .
4)
2 2 2 3
2 1 6 9 6 ( 1)(9 ) 38 10 2 x x x x x x x + + + + = + .
5)
2 2 2
7 22 28 7 8 13 31 14 4 3 3( 2) x x x x x x x + + + + + + + = + .

Bi 13. Gii cc phng trnh sau:

1)
4 2 2 2 2 2 2
2
1
4 16 9 2 2 x y x y x y y x
x
| |
+ + = +
|
\
.
2)
2 2 2 2 3 2
1 1 1 1
2 ... 2 3 3 1
4 4 4 4
x x x x x x x x + + + + + + = + + + .
Trong biu thc v tri c tt c 2008 du cn thc bc hai.











=========================================================== 27
HI CC TRNG THTP CHUYN KHU VC DUYN HI V NG BNG BC B

Hi tho khoa hc mn Ton hc ln th III - 2010
LM NGC BT NG THC
Nguyn c Vang (THPT chuyn Bc Ninh)

Trong bo ton s 377(thng 11 nm 2008) c bi ton sau:
Tm s thc k nh nht sao cho vi mi b s thc khng m x, y, z ta lun c:
{ } x z z y y x Max k xyz
z y x
+
+ +
, , .
3
3
.
Bt chc cch lm y, ti khai thc mt s bt ng thc quen bit, bng cch thm vo
v b mt lng ng bc ti thiu lm thay i s chnh lch.
Bi 1. Tm s thc k nh nht sao cho bt ng thc sau ng vi mi x, y khng
m:

2 2 2 2
. 2 y x k xy y x + + .
Bi 2. Tm s thc k nh nht sao cho bt ng thc sau ng vi mi x, y khng
m:
y x k y x y x + + + . ) ( 2
2 2
.
Bi 3. Tm s thc k nh nht sao cho bt ng thc sau ng vi mi x, y khng
m:
{ } x z z y y x Max k z y x z y x + + + + + , , . ) ( 3
2 2 2
.
Bi 4. Tm s thc k nh nht sao cho bt ng thc sau ng vi mi x, y:

4 4 4 4 4
y x . k )
2
y x
( 2 y x +
+
+
Bi 5. Tm s thc k nh nht sao cho bt ng thc sau ng vi mi x, y khng
m:

n n n n n
y x k
y x
y x +
+
+ . )
2
( 2 (vi n l s nguyn
dng)
Bi 6. Tm s thc k nh nht sao cho bt ng thc sau ng vi mi x, y, z:

{ }
2 2 2 2 2 2 2 2 2 2
, , . max ) ( ) ( 3 x z z y y x k z y x z y x + + + + +
Bi 7. Tm s thc k nh nht sao cho bt ng thc sau ng vi mi x, y, z:
{ }
2 2 2
2 1
2 2
2
2
1
. max ) ... ( ) ... (
j i n n
x x k x x x x x x n + + + + + + +
Bi 8. Tm s thc k nh nht sao cho bt ng thc sau ng vi mi x, y khng
m:

q k
n
n
n
k
k
x x Max k x x n x +

=
. ...
1
1
.
Bi 9. Tm s thc k nh nht sao cho bt ng thc sau ng vi mi x, y
(

2
; 0



=========================================================== 28
HI CC TRNG THTP CHUYN KHU VC DUYN HI V NG BNG BC B

Hi tho khoa hc mn Ton hc ln th III - 2010
) ( sin . cos cos
2
cos
2
y x k y x
y x
+ +
+

Bi 10. Tm s thc k nh nht sao cho bt ng thc sau ng vi mi a, b khng
m:

2 2
. )
2
( 2 ) ( ) ( b a k
b a
f b f a f +
+
+
trong f(x) = x
2
+ 2x +3.
HNG DN GII
Bi 1.
+) Gi s bt ng thc
2 2 2 2
. 2 y x k xy y x + + (*)
ng vi mi x, y khng m.
Cho x = 0, y = 1 suy ra 1 k .
+) Ta chng minh
2 2 2 2
2 . , , : 0 x y x y x y x y x y + + .
Tht vy, BT trn tng ng vi y . x y
2

BT ny ng v 0 y x .
Vy s thc k nh nht cn tm l . 1 k
0
=
Bi 2.
+) Gi s bt ng thc y x k y x y x + + + . ) ( 2
2 2
(*)
ng vi mi x, y khng m.
Cho x = 0, y = 1 suy ra 1 2 k .
+) Ta chng minh
2 2
2( ) . ( 2 1)( ), , : 0 x y x y x y x y x y + + + .
Tht vy, BT trn tng ng vi
xy y y ). 2 2 ( x 2 ) y x ( 2
2 2 2
+ +
BT ny ng v 0 y x .
Vy s thc k nh nht cn tm l . 1 2 k
0
=
Bi 3.
+) Gi s bt ng thc { } x z z y y x Max k z y x z y x + + + + + , , . ) ( 3
2 2 2

(*)
ng vi mi x, y khng m.
Cho x = 1, y = z = 0 suy ra 1 3 k .
+) Ta chng minh
2 2 2
3( ) ( 3 1)( ); , , : 0 x y z x y z x z x y z x y z + + + + + .
Tht vy, BT trn tng ng vi
zx ) 3 3 2 ( yz ) 3 2 ( y . x 3 z ) 1 3 ( 2 y
2 2
+ + +

=========================================================== 29
HI CC TRNG THTP CHUYN KHU VC DUYN HI V NG BNG BC B

Hi tho khoa hc mn Ton hc ln th III - 2010
BT ny ng v



+
2
2
2 2
z ) 3 3 2 ( zx ) 3 3 2 (
z ) 3 2 ( yz ) 3 2 (
z ) 1 3 ( y xy 3
.
Vy s thc k nh nht cn tm l . 1 3 k
0
=
Cch 2:
t f(x;y;z) = 0 z y x ); z x )( 1 3 ( z y x ) z y x ( 3
2 2 2
+ + .
Dng o hm, ch ra c . 0 ) z ; z ; z ( f ) ; z ; y ; y ( f ) z ; y ; x ( f =
Bi 4.
+) Gi s bt ng thc
q k n n
x x kMax x x x x n + + + + + . ... ) ... (
1
2 2
1
(*)
ng vi khi . 0 x ... x x
n 2 1

Cho 1 n k 0 x ... x , 1 x
n 2 1
= = = .
+) BT ) x x )( 1 n ( x ... x ) x ... x ( n
n 1 n 1
2
n
2
1
+ + + + + , vi
. 0 x ... x x
n 2 1

chng minh c bng cch dn bin nh cch 2 ca bi 3.
Vy s thc k nh nht cn tm l . 1 n k
0
=
Bi5.
+) Gi s bt ng thc
4 4 4 4 4
y x . k )
2
y x
( 2 y x +
+
+
ng vi mi x, y khng m.
Cho x = 0, y = 1 suy ra
8
7
k .
+) Dng o hm, ta chng minh c: ) y x .(
8
7
)
2
y x
( 2 y x
4 4 4 4 4
+
+
+ , vi
0 y x
Vy s thc k nh nht cn tm l .
8
7
k
0
=
Bi6.
+) Gi s bt ng thc
) x z , z y , y x ( Max . k ) z y x ( ) z y x ( 3
2 2 2 2 2 2 2 2 2 2
+ + + + +
ng vi x, y, z khng m.
Cho x = 1, y = z = 0 suy ra 2 k .
+) Dng o hm, ta chng minh c ) z x ( 2 ) z y x ( ) z y x ( 3
2 2 2 2 2 2
+ + + + +
vi 0 z y x
Vy s thc k nh nht cn tm l . 2 k
0
=
Bi 7

=========================================================== 30
HI CC TRNG THTP CHUYN KHU VC DUYN HI V NG BNG BC B

Hi tho khoa hc mn Ton hc ln th III - 2010
+) Cho x
1
= 1, x
2
= = x
n
= 0 1 k n
+) t f(x
1
;x
2
;;x
n
) = n(x
1
2
+ +x
2
n
) (x
1
++x
n
)
2
(n - 1)(x
2
1
x
n
2
),
vi 0 ....
2 1

n
x x x . Khi
0 ) x ;...; f(x ) x ;...; x ; x ; f(x ) x ;...; x ; f(x
n n n 3 2 2 n 2 1
.
Vy s thc k nh nht cn tm l . 1
0
= n k
Bi 8
+) Gi s bt ng thc
q k
n
n
n
k
k
x x Max k x x n x +

=
. ...
1
1

ng khi x
1
, , x
n
khng m.
Cho x
1
= x
2
= = x
n-1
=1, x
n
= 0 suy ra 1 n k .
+) Ta chng minh ) x x )( 1 n ( x ... x n x
n 1
n
n 1
n
1 k
k
+

=
vi 0 x ... x x
n 2 1

Tht vy, BT trn tng ng vi
1
n
n 1 n
1 n
2 k
k
x ) 2 n ( x ... x n x . n x + +

=

BT ng v

+ +

n
n 2 1 n
1 1 n 2
x ... x . x n nx
x ) 2 n ( x ... x

Vy s thc k nh nht cn tm l . 1 n k
0
=
Bi 9
Vi 0
2
y x

, ta c:

2
2
2 2 2 2
2
2cos (cos cos ) 4cos .sin cos
2 2 4 2
sin ( )
4sin .cos 4cos .cos
2 2 4 2
1

4cos .c
8
x y x y x y x y
x y
x y x y x y x y
x y

+ + +
+
= =

2 2
3 2 2
os
4

=
+

1 n k
Bi 10
+)Vi a > b > 0 ta c
2
1
) ( 2
)
2
( 2 ) ( ) (
2 2

+

+
+
b a
b a
b a
b a
f b f a f

+) D dng chng minh bt ng thc
2 2
.
2
1
)
2
( 2 ) ( ) ( b a
b a
f b f a f +
+
+ vi a, b
khng m.
Vy gi tr nh nht ca k l:
2
1
= k

=========================================================== 31
HI CC TRNG THTP CHUYN KHU VC DUYN HI V NG BNG BC B

Hi tho khoa hc mn Ton hc ln th III - 2010
CHNG MINH BT NG THC BNG CCH S DNG BT
NG THC SP XP LI V BT NG THC CHEBYSHEV
o Quc Huy - T Ton Tin
Trng THPT Chuyn Bin Ha H Nam

Bt ng thc l mt chuyn quan trng trong chng trnh bi dng hc sinh
gii Quc gia. Trong cc phng php chng minh bt ng thc th phng php p dng
bt ng thc c in thng xuyn c s dng, c rt nhiu bi ton chng minh bt
ng thc m li gii cp n vic s dng bt ng thc lin h gia Trung bnh cng -
Trung bnh nhn (AM-GM), bt ng thc Cauchy Schwarz, bt ng thc Holder, bt
ng thc Schur . Trong khun kh bi vit, ti xin cp n bt ng thc Sp xp li
v mt s bi tp s dng bt ng thc ny. Bn cnh , bi vit cng cp n mt
phng php s dng bt ng thc Chebyshev (coi nh h qu ca bt ng thc Sp xp
li) nh gi mt s bt ng thc 3 bin dng phn thc.
I. Bt ng thc Sp xp li:
Gi s
1 2
...
n
a a a v
1 2
...
n
b b b
( )
*
n l hai dy cc s thc. Ta t
1 1 2 2
1 2 1 1
...
...
n n
n n n
A a b a b a b
B a b a b a b

= + + +
= + + +

Gi
1 2
( , ,..., )
n
x x x l mt hon v ca
1 2
( , ,..., )
n
b b b , t
1 1 2 2
...
n n
X a x a x a x = + + +
Khi ta c bt ng thc sau A X B
Du ng thc xy ra khi v ch khi cc
i
a tt c bng nhau hoc cc
i
b tt c bng
nhau.
Chng minh:
Trc ht ta chng minh A X bng phng php qui np:
- Vi 1 n = , kt qu l hin nhin.
- Gi s bt ng thc ng cho n k = , vi 1 n k = + ta t
1 k i
b x
+
= v
1 k j
x b
+
= .
T bt ng thc ( )( )
1 1
0
k i k j
a a b b
+ +
ta thu c
1 1 1 1 i j k k k j i k
a b a b a b a b
+ + + +
+ + ,
nh vy trong X ta c th thay i
i
x v
1 k
x
+
thu c tng ln hn. Sau khi i ta p
dng gi thit qui np cho k thnh phn u tin ca tng X v suy ra A X .
Bt ng thc X B c suy ra t A X bng cch xt dy
1 1
...
n n
b b b


thay cho dy
1 2
...
n
b b b .

Vi k hiu nh trn, mt cch ngn gn ta coi A l tng cc ch s cng chiu, B
l tng cc ch s o chiu, cn X l tng cc ch s ty . Bt ng thc Sp xp li
cho ta: tng cng chiu tng ty tng o chiu.

=========================================================== 32
HI CC TRNG THTP CHUYN KHU VC DUYN HI V NG BNG BC B

Hi tho khoa hc mn Ton hc ln th III - 2010
Vic p dng bt ng thc Sp xp li quan trng nht ch bin i bt ng thc
cn chng minh v dng c cc v l tng ca tch cc phn t tng ng ca 2 dy m th
t ca chng lin quan vi nhau (cng th t hoc ngc th t). Chng hn hai dy
{ } , , a b c v
{ }
3 3 3
, , a b c c cng th t, cn vi , , 0 x y z > th hai dy { } , , x y z v
1 1 1
, ,
x y z x y z

`
+ + +
)
ngc th t.

II. S dng bt ng thc Sp xp li:
Mc d bt ng thc Sp xp li c pht biu cho cc s thc nhng trong cc
bi tp di y gi thit thng cho iu kin cc s dng hoc khng m, iu ny nhm
mc ch sp xp 2 dy cng chiu ca gi thit c tha ng. Bn cnh , nu khng c
g c bit tc gi xin khng trnh by trng hp xy ra du ng thc ( bi v n hon ton
nh pht biu trn, ng thc xy ra khi 1 trong 2 dy l dy dng ), ng thi tc gi xin
c s dng k hiu

thay th cho
cyc

trong cc bi ton chng minh bt ng thc


quay vng ca 3 bin. Ngoi cch p dng bt ng thc Sp xp li, c nhiu bi ton trong
s nhng bi di y hon ton c th gii bng nhng phng php khc, v bn cnh s
dng bt ng thc Sp xp li ta cn p dng mt s bt ng thc c in khc.

Bi ton 1:
Cho n s thc ( , 2) n n :
1 2
, ,...,
n
a a a tha mn
1 2
1 2
... 0
... 1
n
n
a a a
a a a
+ + + =

+ + + =


Chng minh rng
1 2
1
2 ...
2
n
n
a a na

+ + +

Bi gii:
K hiu
1 2
, ,...,
t
j j j v
1 2
, ,...,
k
s s s l cc phn t { } 1, 2,..., n sao cho
1 2 1 2
... 0 ...
t k
j j j s s s
a a a a a a
T gi thit ta suy ra
1 2
1
...
2
t
j j j
a a a + + + = v
1 2
1
...
2
k
s s s
a a a + + + = .
Khng gim tnh tng qut, ta c th gi s
1 2
2 ... 0
n
a a na + + + ( v nu tri li ta
dng php t '
i i
a a = ). Theo bt ng thc Sp xp li ta c:
1 1 1
1 1 1
1 2
1 2 ... 1 2 ... ( 1) ...
1
1 1 ... 1 ...
2
k k t
k k t
n s s s j j
s s s j j
a a na a a ka k a na
n
a a a na na

+ + + + + + + + + +

+ + + + + +

Bt ng thc cho c chng minh.

=========================================================== 33
HI CC TRNG THTP CHUYN KHU VC DUYN HI V NG BNG BC B

Hi tho khoa hc mn Ton hc ln th III - 2010

Bi ton 2:
Cho cc s thc , , 0 a b c tha mn 1 a b c + + = .
Chng minh rng
2
2
a b
b c
+



Bi gii :
Ta c
2 2
( ) ( )
1
a b a b a b c a a b
b c b c b c
+ + + + +
= = +
+ + +


bt ng thc cn chng minh tr thnh
( ) a a b
a b c
b c
+
+ +
+

(*)
Ch l 2 dy { } , , a b c v
1 1 1
, ,
b c c a a b

`
+ + +
)
c cng th t, theo bt ng thc Sp
xp li ta c
2
( ) ( )
a ab ca ab
a b c
b c b c b c b c
+ + = + +
+ + + +

. Vy (*) c chng minh.

Bi ton 3:
Cho , , 0. x y z >
Chng minh rng ( ) ( ) ( )
2 2 2
1
y x z y x z
x y z y z x z x y

( ( ( + + +



Bi gii:
t ( ) ( ) ( )
2 2 2
, , a x y z b y z x c z x y = + = + = + .
Ly lgarit t nhin 2 v, a bt ng thc cn chng minh v dng:
ln ln x c x a

, bt ng thc ny ng theo bt ng thc Sp xp li vi nhn xt
rng 2 dy { } , , x y z v { } , , a b c cng th t, ng thi hm lnt ng bin trn ( ) 0; + nn
dy { } ln , ln , ln a b c cng c th t nh 2 dy trn, ta suy ra iu phi chng minh.

Bi ton 4:
Cho tam gic nhn ABC .
Chng minh rng
3
1 sin sin
2
A B




=========================================================== 34
HI CC TRNG THTP CHUYN KHU VC DUYN HI V NG BNG BC B

Hi tho khoa hc mn Ton hc ln th III - 2010
Bi gii:
S dng nh l sin, bt ng thc cn chng minh tng ng vi:
( )
2
2 2
2
3
3 4 4 0 0
4
R ab
R R ab R ab R
R ab R


+

(*)
( y a,b,c,R theo th t l di cc cnh BC,CA,AB v bn knh ng trn ngoi
tip tam gic ABC ).
T bt ng thc quen thuc
2 2 2 2
9 a b c R + + m
2 2
3 2 ab ab ab a b = +
ta thu c:
( )
2 2
3 3R ab c ab (1).
Ta s chng minh
2
2 2
4 4
c ab
R ab R R ab R

+ +

(2).
V 2 dy { } , , a b c v
2 2 2
1 1 1
, ,
4 4 4 R bc R R ca R R ab R

`
+ + +
)
c cng th t
nn theo bt ng thc Sp xp li ta c
2 2 2
2 2 2 2
2 2
4 4 4 4
c a b ab
R ab R R ab R R ab R R ab R
+
+ + + +


Vy (2) c chng minh, kt hp vi (1) ta suy ra (*) c chng minh.
Bi ton 5:
Cho , , 0. a b c >
Chng minh rng
2 2 2
2 2 2
1 1 1
1 1 1
a b c a b c
b c a b c a
+ + +
+ + + +
+ + +


Bi gii:
Ta c
( ) ( )( ) ( )( ) ( ) ( ) ( )
( )( )
2 2 2 2 2 2 2 2 2 2 2 2
2 2 2 2 2 2
2 2
2 2
1 1 1 2
2 1 1
1 1
1 1
a b a b a b ab a b ab a b ab a b
a b a b a b a b
b a ab b a
a b
+ + + + + = + + + + +
+ + + + +
+ = = +
+ +
+ +


Theo bt ng thc Sp xp li
( ) ( )
2 2 2 2 2 2
2 2 2 2 2 2 2 2
1
1 1 1 1 1
a a a a b a
b b b b b b b b
+
= + + =
+ + + + +

.
T ta c:

=========================================================== 35
HI CC TRNG THTP CHUYN KHU VC DUYN HI V NG BNG BC B

Hi tho khoa hc mn Ton hc ln th III - 2010
2
2
2
2
2 2 2 2
2 2 2 2
1 1 2
1 1 1 1
1 2 1
1 1 1 1
a a b a
b b a b
a b a a
b a b b
| | | |
+ = + + +
| |
\ \
| | | |
+ + + +
+ + + = + | |
| |
+ + + +
\ \



hay
2
2
1
1
a a
b b
+

+

(pcm).

Bi ton 6:
Cho , , 0 a b c tha mn 1 a b c + + = .
Chng minh rng
2 2 2
4
3
9
a b c abc + + +

Bi gii:
Ta cn chng minh
2 2 2 3
4
( )( ) 3 ( )
9
a b c a b c abc a b c + + + + + + +
Khai trin ri a bt ng thc v dng ( )
3
5 3 3 a abc ab a b + +


- Theo bt ng thc Schur: c ( )
3
3 a abc ab a b + +

(1).
- Theo bt ng thc Sp xp li: c
3 2 2 2
a a b b c c a + +

v
3 2 2 2
a ab bc ca + +

,
cng tng v ta thu c ( ) ( )
3 3
2 4 2 a ab a b a ab a b + +

(2).
Cui cng, cng tng v ca (1) v (2) ta c pcm.

Bi ton 7:
Cho , , , 0 a b c d tha mn 4 a b c d + + + = .
Chng minh rng
2 2 2 2
4 a bc b cd c da d ab + + +

Bi gii:
Gi s ( ) , , , p q r s l hon v ca ( ) , , , a b c d sao cho p q r s .

=========================================================== 36
HI CC TRNG THTP CHUYN KHU VC DUYN HI V NG BNG BC B

Hi tho khoa hc mn Ton hc ln th III - 2010
Khi
( ) ( ) ( ) ( )
( ) ( ) ( ) ( ) ( )( )
( )( )
2 2 2 2
2
2 2
2 1 1
2 4 4 2
a bc b cd c da d ab a abc b bcd c cda d dab
p pqr q pqs r prs s qrs pq rs pr qs
pq rs pr qs p q r s
p s q r
+ + + = + + +
+ + + = + +
(
+ + + + + + | | | |
= + + (
(
| |
\ \
(


(bt ng thc u tin l bt ng thc Sp xp li, hai bt ng thc sau l bt ng
thc AM-GM)
l 4 p q r s a b c d + + + = + + + = , ta c pcm.

III.Bt ng thc Chebyshev dng mu s
Bt ng thc Chebyshev c in c th coi nh l h qu ca bt ng thc Sp xp li
(xem bi tp p dng 1 phn V), t dng c in ny ngi ta m rng bt ng thc
Chebyshev theo mt vi hng, sau y l mt dng m rng c nhiu ng dng chng
minh bt ng thc:

Bt ng thc Chebyshev dng mu s (cn gi l dng Engel) c pht biu nh sau:
a) Nu ta c
1 2
1 2
1 2
...
...
n
n
n
a a a
x x x
x x x

hoc
1 2
1 2
1 2
...
...
n
n
n
a a a
x x x
x x x

th ta c
( )
1 2 1 2
1 2 1 2
...
...
...
n n
n n
n a a a a a a
x x x x x x
+ + +
+ + +
+ + +

b) Nu ta c
1 2
1 2
1 2
...
...
n
n
n
a a a
x x x
x x x

hoc
1 2
1 2
1 2
...
...
n
n
n
a a a
x x x
x x x

th ta c
( )
1 2 1 2
1 2 1 2
...
...
...
n n
n n
n a a a a a a
x x x x x x
+ + +
+ + +
+ + +

(Chng minh 2 kt qu ny bng cch s dng trc tip bt ng thc Chebyshev)
Hai kt qu trn, kt hp vi vic thm cc biu thc ph hp, tr nn hiu qu trong
vic nh gi cc bt ng thc i 3 bin c cha phn thc, mc d chng ch l m rng
n gin t bt ng thc Chebyshev. lm r thm, chng ta xt mt vi v d sau:

Bi ton 1: Cho , , 0 a b c > tha mn
1 1 1
1
1 1 1 a b b c c a
+ +
+ + + + + +


=========================================================== 37
HI CC TRNG THTP CHUYN KHU VC DUYN HI V NG BNG BC B

Hi tho khoa hc mn Ton hc ln th III - 2010
Chng minh rng a b c ab bc ca + + + +

Li gii: Ta c
1 1 1
1 1 1
a b c
a b b c c a ab ac a bc ba b ca cb c
+ + = + +
+ + + + + + + + + + + +

Khng gim tnh tng qut, gi s c b a , th th
c ca ca b bc ba a ab ac + + + + + +
Li c
a b c
ab ac a bc ba b ca cb c

+ + + + + +
lun ng (v bt ng thc ny
a b c )
Do theo a) th

( )
( )
1 1 1
1 1 1
3
2
a b c
a b b c c a ab ac a bc ba b ca cb c
a b c
a b c ab bc ca
+ + = + +
+ + + + + + + + + + + +
+ +

+ + + + +

Kt hp vi gi thit ta suy ra
( )
( )
3
1
2
a b c
a b c ab bc ca
a b c ab bc ca
+ +
+ + + +
+ + + + +

Ta c iu phi chng minh, ng thc xy ra khi 1 a b c = = = .

Bi ton 2: Cho , , 0 a b c > tha mn 3 a b c + + = .
Chng minh rng
4 9 4 9 4 9
13
ab bc ca
ab a b bc b c ca c a
+ + +
+ +
+ + + + + +


Li gii:
Khng mt tnh tng qut, gi s a b c , khi t bt ng thc lun ng
( )( ) 1 0 a b c + ta suy ra ab a b ac a c + + + + . Tng t ta thu c
ab a b ac a c bc b c + + + + + +
Cng t bt ng thc lun ng ( )( )( ) 3 3 0 b c a a + ta thu c
4 9 4 9 ab ca
ab a b ca c a
+ +

+ + + +
, tng t ta c
4 9 4 9 4 9 ab ca bc
ab a b ca c a bc b c
+ + +

+ + + + + +


=========================================================== 38
HI CC TRNG THTP CHUYN KHU VC DUYN HI V NG BNG BC B

Hi tho khoa hc mn Ton hc ln th III - 2010
Do theo b) VT ca bt ng thc cn chng minh
( )
( )
( ) 12 81 12 81
2 6
ab bc ca ab bc ca
ab bc ca a b c ab bc ca
+ + + + + +
=
+ + + + + + + +

Ta ch cn chng minh
( )
3 4 27
13 3
6
ab
ab
ab
+

+


bt ng thc cui cng ny ng do
( )
2
3
3
a b c
ab bc ca
+ +
+ + = .

Bi ton 3: Cho tam gic nhn ABC .
Chng minh rng
1 1 1 3
1 tan tan 1 tan tan 1 tan tan 1 2 3 A B B C C A
+ +
+ + + + + + +


Li gii: Tng t Bi ton 1 ta c bt ng thc
( )
( )
1 1 1
1 tan tan 1 tan tan 1 tan tan
3 tan tan tan
tan tan tan 2 tan tan tan tan tan tan
A B B C C A
A B C
A B C A B B C C A
+ +
+ + + + + +
+ +

+ + + + +

T bt ng thc ( ) 3 ab bc ca abc a b c + + + + v ng thc tan tan A A =


c
( )
( )
( )
( )
3 tan tan tan
tan tan tan 2 tan tan tan tan tan tan
3 3
2 tan tan tan tan tan tan
3 tan tan
1
1 2
tan tan tan
tan
3 3
1 2 3
3 tan
1 2
tan
A B C
A B C A B B C C A
A B B C C A
A A
A B C
A
A
A
+ +
+ + + + +
=
+ +
+
+
+ +
= =
+
+


Ta c iu phi chng minh.

Bi ton 4: Cho , , 0 a b c > tha mn
2 2 2
1 a b c + + = .
Chng minh rng
( )
9
1 1 1 2
a b b c c a
ab bc ca a b c
+ + +
+ +
+ + + + +


=========================================================== 39
HI CC TRNG THTP CHUYN KHU VC DUYN HI V NG BNG BC B

Hi tho khoa hc mn Ton hc ln th III - 2010

Li gii: Bt ng thc cn chng minh tng ng vi
( )( )
2 2 2
2 2 2
2 3 2 2
3 0 0
1
a b a b c a b c ab ac bc
a b c ab ab
+ + + | | + +

|
+ + + +
\


Khng gim tnh tng qut, gi s 1 1 1 a b c ab ac bc + + +
Ta cn tip tc kim tra bt ng thc
( ) ( ) ( ) ( ) ( ) ( )
( )( )
( ) ( ) ( ) ( ) ( )
( ) ( )( )
2 2 2 2 2 2
2 2 3 3 3 2 2 2
3 2 2 2 2
2 2 3 2 2 2 2 2
2 2
3 2 2 3 2 2
1 1
2 3 2 0
2 2 3 3 3 2 2 0
2 1 2 1 2 3 3 0
2 2
a b c ab ac bc a c b ac ab bc
ab ac
b c a b c a b c abc b c a b c a b c
b c b c a a abc ab abc ac a b a c
b c b a c a abc a a a b c ab ac
b c b c b c
+ + + +

+ +
+ +
+ + + +
+ + + + + + +
+ + +
( )
2 2
2 2 0 abc ab ac + +

bt ng thc trn lun ng, tng t ta thu c
2 2 2 2 2 2 2 2 2
3 2 2 3 2 2 3 2 2
1 1 1
a b c ab ac bc a c b ac ab bc b c a bc ab ca
ab ac bc
+ + + + + +

+ + +

Do theo b) ta c
( ) ( )
2 2 2 2
2 2 2
3 3 2 2 12
3 2 2
0
1 3 3
a b c ab ac bc a ab
a b c ab ac bc
ab ab ab
+ +
+ +
=
+ + +



Ta c iu phi chng minh.

Bi ton 5: Cho cc s thc , , x y z sao cho 1 x y z + + = .
Chng minh rng
2 2 2
9
1 1 1 10
x y z
x y z
+ +
+ + +


Li gii: Ta c
2 2 2 2 2 2
1 1 1 1 1 1
x y z x y z
x y z x y z
+ + + +
+ + + + + +

Do ta ch cn chng minh bt ng thc trong trng hp , , 0 x y z > .
Khng mt tnh tng qut, gi s x y z , khi
2 2 2
1 1 1 x y z + + +

=========================================================== 40
HI CC TRNG THTP CHUYN KHU VC DUYN HI V NG BNG BC B

Hi tho khoa hc mn Ton hc ln th III - 2010
Tip tc, ta kim tra ( )( )
2 2
1 0
1 1
x y
x y xy
x y

+ +

V 1 2 1 x y z x y xy xy + + = + < nn bt ng thc trn ng.
Tng t ta c
2 2 2
1 1 1
x y z
x y z

+ + +

Do theo a) ta thu c
( )
( )
2 2 2 2 2 2
2
3 3 9
1
1 1 1 3 10
3
3
x y z x y z
x y z x y z
x y z
+ +
+ + =
+ + + + + +
+ + +

Ta c iu phi chng minh.

Bi ton 6: Cho , , 0 a b c > tha mn
1 1 1
1
1 2 1 2 1 2 ab bc ca
+ +
+ + +

Chng minh rng 3 a b c abc + +
Li gii:
Ta c
1 1 1
1 2 1 2 1 2 2 2 2
a b c
ab bc ca a abc b abc c abc
+ + = + +
+ + + + + +

Khng gim tnh tng qut, gi s a b c , th th 2 2 2 a abc b abc c abc + + +
li c
2 2 2
a b c
ab ac bc
a abc b abc c abc

+ + +
( lun ng )
do theo a) ta c
( ) 3 1 1 1
1 2 1 2 1 2 2 2 2 6
a b c a b c
ab bc ca a abc b abc c abc a b c abc
+ +
+ + = + +
+ + + + + + + + +

kt hp vi gi thit ta suy ra
( ) 3
1 3
6
a b c
a b c abc
a b c abc
+ +
+ +
+ + +

Ta c iu phi chng minh, ng thc xy ra khi 1 a b c = = =

Bi ton 7: Cho , , 0 a b c > tha mn 3 ab bc ca + + = .
Chng minh rng

=========================================================== 41
HI CC TRNG THTP CHUYN KHU VC DUYN HI V NG BNG BC B

Hi tho khoa hc mn Ton hc ln th III - 2010
( ) ( ) ( )
2 2 2
1 1 1 3
1 1 1 1 2 a b c b c a c a b abc
+ +
+ + + + + + +

Li gii: Bt ng thc cn chng minh tng ng vi
( ) ( ) ( )
( )
( )
( )
( )
( )
( )
( ) ( ) ( )
2 2 2
1 1 1 1 1 1
0
1 2 1 1 2 1 1 2 1
1 1 1
0
1 3 1 3 1 3
1 1 1
0
1 3 1 3 1 3
abc a b c abc b c a abc c a b
a bc b ac c ba
a bc b ac c ba
bc ac ba
abc bc abc ac abc ba
+ +
+ + + + + + + + +

+ +
+ + +

+ +
+ + +

Khng mt tnh tng qut, gi s a b c , khi
( ) ( ) ( ) 1 3 1 3 1 3 abc ba abc ac abc bc + + +
V ta cng c
( ) ( ) ( )
1 1 1
1 3 1 3 1 3
bc ac ba
abc bc abc ac abc ba


+ + +

(v
3 2 2 2
3 3 1 ab bc ca a b c abc = + + )
Do theo b) ta c
( ) ( ) ( )
( )
( )
3 3 1 1 1
0
1 3 1 3 1 3 3 9
ab bc ca bc ac ba
abc bc abc ac abc ba abc ab bc ca

+ + =
+ + + +

Ta c iu phi chng minh.
IV. Mt s bi tp p dng
Bi tp 1: (bt ng thc Chebyshev)
K hiu , A B ging nh trong bt ng thc Sp xp li.
CMR:
( )( )
1 2 1 2
... ...
n n
a a a b b b
A B
n
+ + + + + +

Bi tp 2:
Cho n s thc dng
1 2
, ,...,
n
c c c (vi
*
n ). Ta k hiu
( ) ( )
( ) ( ) ( ) ( )
1 2
2 2 2
1 2 1 2
1
1 2 1 2
... , ... ,
... , 1 1 ... 1
n n
n
n n
RMS c c c n AM c c c n
GM c c c HM n c c c
(
= + + + = + + +

= = + + + (


CMR: RHM AM GM HM
Bi tp 3:
Cho , , 0. a b c > CMR:
1
2
k k
a a
a b

+

( 2) k
Bi tp 4:

=========================================================== 42
HI CC TRNG THTP CHUYN KHU VC DUYN HI V NG BNG BC B

Hi tho khoa hc mn Ton hc ln th III - 2010
Cho ( ) , , 1; a b c + . CMR:
2 2 2
log log log log log log
ab bc ca
a bc ab c abc
c a b bc ca ab + + + +
Bi tp 5:
K hiu , , a b c ln lt l di cc cnh , , BC CA AB ca tam gic nhn ABC .
CMR: ( ) 4
cos cos cos
a b b c c a
a b c
C A B
+ + +
+ + + +
Bi tp 6:
Cho , , 0 a b c > . Chng minh cc bt ng thc sau:
a)
( ) ( ) ( )
2 2 2
0
a b b c b c c a c a a b
a b b c c a

+ +
+ + +

b)
3 3 3 3 3 3
2 2 2 2 2 2
a b b c c a
a b c
b c c a a b
+ + +
+ + + +
+ + +

c)
5 5 5 2 2 2
3 3 3 3 3 3
2
a b c a b c
a b b c c a
+ +
+ +
+ + +

Bi tp 7:
Cho , , 0 a b c > tha mn 3 a b c + + = .
CMR:
1 1 1 3
9 9 9 8 ab bc ca
+ +


Bi tp 8:
Cho , , , 0 a b c d tha mn 4 a b c d + + + = .
CMR:
1 1 1 1
1
5 5 5 5 abc bcd cda dab
+ + +


Bi tp 9:
Cho , , 0 a b c . CMR: ( )
2 2 2
3 8 8 8 a b c a bc b ca c ab + + + + + + +
Bi tp 10:
Chng minh rng vi , , 0 a b c v 2 0 k ta c bt ng thc
2 2 2
2 2 2 2 2 2 2 2 2
0
a bc b ca c ab
b c ka a c kb b a kc

+ +
+ + + + + +

V. Ti liu tham kho:
1. G.H. Hardy, J.E. Littlewood, G. Polya, Bt ng thc.
2. Phm Kim Hng, Sng to bt ng thc ( tp 1).
3. VIMF (Nhiu tc gi), Discovery Inequalities (Third version).
-------------------------------------------------------------------------------

=========================================================== 43
HI CC TRNG THTP CHUYN KHU VC DUYN HI V NG BNG BC B

Hi tho khoa hc mn Ton hc ln th III - 2010
TNH TUN HON TRONG DY S NGUYN
Tc gi: Ng Th Hi
Gio vin trng THPT chuyn Nguyn Tri, Hi Dng.

Dy s l mt lnh vc kh v rt rng. gii c cc bi ton loi ny khng ch
i hi ngi lm Ton phi s dng nhiu kin thc khc nhau ca Ton hc m cn phi
c kh nng sng to rt cao. Trong cc bi ton v dy s mt vn c quan tm nhiu
l tnh cht s hc ca dy s nh: tnh chia ht, tnh cht nguyn hay tnh chnh phng
Chng rt a dng v phong ph. Trong nhiu trng hp, dy s ch l v b ngoi cn
bn cht bi ton l mt bi s hc. Chnh v l , cc bi ton v s hc ni chung, cc
bi ton v tnh cht s hc ca dy s ni ring thng xut hin trong cc k thi hc sinh
gii quc gia v quc t, v n bao gm nhiu bi ton hay v kh. Trong khun kh ca bi
vit ny ti ch cp n mt kha cnh rt nhca dy s nguynl tnh tun hon, hi
vng rng y l mt ti liu tham kho tt cho cc em hc sinh kh v gii. Trc ht ta
hy xem nh l su y:
nh l: Cho dy s nguyn truy hi cp k ( k l s nguyn dng) ngha l

Nu dy b chn th n l dy tun hon k t lc no .
Chng minh:
Gi s dy b chn bi s nguyn dng M, ngha l .
Xt cc b k s C ti a
b khc nhau nn trong b u tin pha c 2 b trng nhau.
Chng hn

Ngha l
M nn
t th ta c
Vy dy tun hon vi chu k k t
H qu : Cho dy s nguyn tho mn
trong l cc s nguyn v m l s
nguyn dng ln hn 1. Gi l s d trong php chia cho m. Khi dy tun
hon.
Chng minh:
Theo gi thit ta c . Theo tnh cht ca ng d thc ta c

Theo cc xc nh ta c tc l dy b chn v truy hi tuyn tnh
cp k nn theo nh l trndy tun hon k t lc no , ngha l sao cho

Chn ta c



Vy . Tng t ta c = ,,
Do dy tun hon vi chu k T.

=========================================================== 44
HI CC TRNG THTP CHUYN KHU VC DUYN HI V NG BNG BC B

Hi tho khoa hc mn Ton hc ln th III - 2010
Sau y ti s a ra mt s v d in hnh v vic p dng nh l trn. Cc bi ton
nu ra y u s dng n tnh tun hon ca dy s d. Gi s l s d trong php
chia cho mt s nguyn dng m no . Khi dy b chn v cng c cng cng
thc truy hi vi dy nn theo h qu trn n l dy tun hon.
Bi 1:
Cho Vi l s d ca php chia cho
100. Tm s d trong php chia cho 8.
Bi gii:
Gi l s d trong php chia cho 4. Theo gi thit
nn

Mt khc tc l dy b chn do dy ny tun hon.
Ta tnh c
D kim tra tun hon chu k 6, ngha l
Li c . Do nn cng tnh chn l suy
ra
hay
Vy
M
. Do
chia ht cho 8.
Bi 2:
Cho dy , n=0,1,2, xc nh bi v

Chng minh rng: chia ht cho 20
Bi gii:
T cng thc truy hi ca dy ta thy ( .Gi l s d trong
php chia cho 4. Khi Hn na ( nn tng t
bi 1 dy tun hon chu k 6.
Ta c ( . V vy

tc l chia ht cho 4.
Mt khc vi ta c
Suy ra

Vy . Do
chia ht cho 20.
Bi 3:
Cho dy , n=,1,2,3 xc nh bi
Chng minh rng tn ti v s s hng ca dy chia ht cho 2005.
Bi gii:

=========================================================== 45
HI CC TRNG THTP CHUYN KHU VC DUYN HI V NG BNG BC B

Hi tho khoa hc mn Ton hc ln th III - 2010
Ta c chia ht cho 2005. Gi l s d trong php chia cho 2005. T
cng thc truy hi ca dy ta c

ng thi dy tun hon k t lc no , ngha l sao cho
.
Chn ta c

Vy . Tng t ta cng c
Do hay chia ht cho 2005 (pcm)
Bi 4:
Cho dy , n=,1,2,3 xc nh bi

Chng minh rng vi mi s nguyn dng tn tai v s s t nhin sao cho
cng chia ht cho .
Bi gii:
Xt dy , n=,1,2,3 xc nh nh sau
Ta tnh c Do
Gi l s d trong php chia cho m. Khi dy tun hon ngha l tn ti s
t nhin T>1 sao cho
Vy chia ht cho m vi hay
chia ht cho m vi .
Bi 5:
Gi l nghim dng ln nht ca phng trnh Xt dy xc
nh theo cng thc sau: Tm s d trong php chia
cho 17.
Bi gii:
t Ta c >0,
.
Do l hm lien tc trn R nn phng trnh c 3 nghim phn
bit:

t Khi l nghim ca phng trnh sai phn tuyn
tnh thun nht c pt c trng l Do ta c

Hay trong

( s dng nh l Vi-et)
V vy Do >0
Suy ra .

=========================================================== 46
HI CC TRNG THTP CHUYN KHU VC DUYN HI V NG BNG BC B

Hi tho khoa hc mn Ton hc ln th III - 2010
Li c nn

(do .
Vy =( . Cho nn
=
Vy
Gi l s d trong php chia cho 17. Khi dy tun hon v bng tnh ton
trc tip ta c

D kim tra tun hon chu k 16, ngha l l
Vy nn
hay chia 17d 6.
Cui cung ti xin nu thm 2 bi tp khc c th gii theo phng php ny bn c
tham kho
Bi 1:
Cho dy , n=,1,2,3 xc nh bi
Chng minh rng:
a) Mi s hng ca dy u l s nguyn dng.
b) C v s nguyn dng n sao cho c 4 ch s tn cng l 2003.
c) Khng tn ti s nguyn dng n sao cho c 4 ch s tn cng l 2004.
Hng dn: Bin i dn n
Bi 2:
Dy s nguyn , n=,1,2,3 xc nh bi
Chng minh c v s s hng ca dy chia ht cho 1986.



















=========================================================== 47
HI CC TRNG THTP CHUYN KHU VC DUYN HI V NG BNG BC B

Hi tho khoa hc mn Ton hc ln th III - 2010
NH L PASCAL V NG DNG
L c Thnh
GV THPT Chuyn Trn Ph Hi Phng

Trong bi vit chuyn ny ti mun cp n mt nh l c rt nhiu ng dng
a dng, l nh l Pascal v lc gic ni tip ng trn. Trong thc t p dng, khi thay
i th t cc im, hay l khi xt cc trng hp c bit ta s thu c rt nhiu kt qu
khc nhau.
Trc ht ta pht biu ni dung nh l:

nh l Pascal:
Cho cc im A, B, C, D, E, F cng thuc mt ng trn (c th hon i th t). Gi
P AB DE, Q BC EF, R CD FA = = = .
Khi cc im P, Q, R thng hng.
Chng minh:
Gi
X EF AB, Y AB CD, Z CD EF. = = =
p dng nh l Menelaus cho tam gic
XYZ i vi cc ng thng BCQ, DEP, FAR ,
ta c:
( )
( )
( )
CY BX QZ
1 1
CZ BY QX
FZ AX RY
1 2
FX AY RZ
EZ PX DY
1 3
EX PY DZ
=
=
=


Mt khc, theo tnh cht phng tch ca mt im i vi ng trn ta c:
( ) YC.YD YB.YA, ZF.ZE ZD.ZC, XB.XA XF.XE 4 = = =

Nhn (1),(2) v (3) theo v, ta c:
( )
QZ RY PX CY.BX.FZ.AX.EZ.DY
1
QX RZ PY CZ.BY.FX.AY.EX.DZ
QZ RY PX YC.YD ZF.ZE XB.XA
1 5
QX RZ PY YB.YA ZD.ZC XF.ZE
=
=

Th (4) vo (5), ta c
QZ RY PX
1.
QX RZ PY
=
Vy P, Q, R thng hng (theo nh l Menelaus).

ng thng PQR trn c gi l ng thng Pascal ng vi b im
A, B, C, D, E, F.
Bng cch hon v cc im A, B, C, D, E, F ta thu c rt nhiu cc ng thng
Pascal khc nhau, c th ta c ti 60 ng thng Pascal.
Z
Y
X
R
Q
P A
B
C
D
E
F

=========================================================== 48
HI CC TRNG THTP CHUYN KHU VC DUYN HI V NG BNG BC B

Hi tho khoa hc mn Ton hc ln th III - 2010

Chng hn hnh v bn minh ha trng hp cc
im ACEBFD.
Ngoi ra khi cho cc im c th trng nhau (khi
lc gic suy bin thnh tam gic, t gic, ng gic), v d
E F th cnh EF tr thnh tip tuyn ca ng trn ti
E , ta cn thu thm c rt nhiu cc ng thng Pascal
khc na.
Hnh v di y minh ha trng hp cc im
ABCDEE, ABCCDD, AABBCC:














Tip theo ta a ra cc bi ton ng dng nh l Pascal:

Bi ton 1: (nh l Newton)
Mt ng trn ni tip t gic ABCD ln lt tip xc vi cc cnh AB, BC, CD, DA
ti E, F, G, H.
Khi cc ng thng AC, EG, BD, FH ng quy.
R
Q
Y
P
A
B
C
D
E
R
Q
P
A
D
B
C
Q
R
P
B
C
A
P
Q
R
A
B
C
D
E
F

=========================================================== 49
HI CC TRNG THTP CHUYN KHU VC DUYN HI V NG BNG BC B

Hi tho khoa hc mn Ton hc ln th III - 2010
Li gii:
Gi O EG FH, X EH FG = = .
V D l giao im ca cc tip tuyn vi ng trn ti G, H, p dng
nh l Pascal cho cc im E, G, G, F, H, H , ta c:

EG FH O,
GG HH D,
GF HE X.
=
=
=

Suy ra O, D, Xthng hng.
p dng nh l Pascal cho cc im
E, E, H, F, F, G, ta c:

EE FF B,
EH FG X,
HF GE O.
=
=
=

Suy ra B, X, Othng hng.

T ta c B, O, Dthng hng.
Vy EG, FH, BD ng quy ti O.
Chng minh tng t i vi ng thng AC ta c iu phi chng minh.

Bi ton 2:
Cho tam gic ABC ni tip trong mt ng trn. Gi D, E ln lt l cc im chnh
gia ca cc cung AB, AC; P l im tu trn cung BC; DP AB Q, PE AC R = = .
Chng minh rng ng thng QR cha tm I ca ng trn ni tip tam gic ABC.
Li gii:
V D, E ln lt l im chnh gia ca cc
cung AB, AC nn CD, BE theo th t l cc ng
phn gic ca gc

ACB, ABC.
Suy ra I CD EB. =
p dng nh l Pascal cho su im
C, D, P, E, B, A, ta c:
CD EB I = ;
DP BA Q; =
PE AC R. =
Vy Q, I, R thng hng.




Bi ton 3: (Australia 2001)
X
O
C
D
A
B
G
E
H
F
I
R
Q
E
D A
B C
P

=========================================================== 50
HI CC TRNG THTP CHUYN KHU VC DUYN HI V NG BNG BC B

Hi tho khoa hc mn Ton hc ln th III - 2010
Cho tam gic ABC ni tip ng trn (O), ng cao nh A, B, C ln lt ct (O) ti
A, B, C. D nm trn (O), DA' BC A", DB' CA B", DC' AB C" = = = .
Chng minh rng: A, B, C, trc tm H thng
hng.
Li gii:
p dng nh l Pascal cho su im
A, A', D, C', C, B, ta c:

AA' C' C H,
A' D CB A",
DC' BA C".
=
=
=

Vy H, A", C" thng hng.
Tng t suy ra A, B, C, H thng hng.

Bi ton 4: (IMO Shortlist 1991)
P thay i trong tam gic ABC c nh. Gi P, P l hnh chiu vung gc ca P trn
AC, BC, Q, Q l hnh chiu vung gc ca C trn AP, BP, gi X P' Q" P"Q' = .
Chng minh rng: X di chuyn trn mt ng c nh.
Li gii:
Ta c:

0
CP' P CP"P CQ' P CQ"P 90 = = = =
Nn cc im C, P', Q", P, Q', P" cng thuc
mt ng trn.
p dng nh l Pascal cho su im
C, P', Q", P, Q', P" ta c:

CP' PQ' A,
P' Q" Q' P" X,
Q"P P"C B.
=
=
=

Vy A, X, B thng hng.
Vy X di chuyn trn ng thng AB c nh.

Bi ton 5: (Poland 1997)
Ng gic ABCDE li tha mn:

0
CD DE, BCD DEA 90 = = = . im F trong on
AB sao cho
AF AE
BF BC
=
Chng minh rng:

FCE ADE, FEC BDC = = .



Li gii:
H
C"
B"
A"
C'
B'
A'
B C
A
D
X
Q"
Q'
P"
P'
A
B C
P
R
Q
P
F
A
E
D
C
B

=========================================================== 51
HI CC TRNG THTP CHUYN KHU VC DUYN HI V NG BNG BC B

Hi tho khoa hc mn Ton hc ln th III - 2010
Gi P AE BC = , Q, R ln lt l giao im ca AD v BD vi ng trn ng
knh PD, G QC RE = .
p dng nh l Pascal cho su im P, C, Q, D, R, E, ta c:

PC DR B,
CQ RE G,
QD EP A.
=
=
=

Vy A, G, B thng hng.
Li c:

DAG
DBG
DAE
DBC
sin GQD
DA GQ
S AG DG.DA.sin GDQ DA.GQ DA.sin QRE
DG
BG S DB.GR
DG.DB.sin GDR sin GRD DB.sin RQC
DB GR
DG
S DA.sin ADE DA.DE.sin ADE AE
S BC
DB.sin BDC DB.DC.sin BDC
AG AF
F G
BG BF

= = = = =

= = = =
=

T d dng c

FCE ADE, FEC BDC = = .

Bi ton 6:
Cho tam gic ABC ni tip ng trn (O), A, B, C l trung im BC, CA, AB.
Chng minh rng tm ng trn ngoi tip cc tam gic AOA, BOB, COC thng
hng.
Li gii:
Gi A, B, C l trung im ca OA,
OB, OC. I, J, K l tm cc ng trn ngoi
tip cc tam gic AOA, BOB, COC. Khi
I l giao im ca cc trung trc ca OA v
OA, hay chnh l giao im ca BC v tip
tuyn ca ng trn (O;OA) ti A. Tng
t vi J, K.
p dng nh l Pascal cho su im
A", A", B", B", C", C" ta c:

A"A" B"C" I,
A"B" C"C" K,
B"B" C"A" J.
=
=
=

Vy I, J, K thng hng.
Bi ton 7: (China 2005)
Mt ng trn ct cc cnh ca tam gic ABC theo th t ti cc im
1 2 1 2 1 2
D , D , E , E , F , F .
1 1 2 2 1 1 2 2 1 1 2 2
D E D F L, E F E D M, FD F E N = = = .
Chng minh rng AL, BM, CN ng quy.
K
J
I
B"
A"
C"
C'
B'
A'
O
B
C
A

=========================================================== 52
HI CC TRNG THTP CHUYN KHU VC DUYN HI V NG BNG BC B

Hi tho khoa hc mn Ton hc ln th III - 2010
Li gii:












Gi
1 1 2 2 1 1 2 2 1 1 2 2
D F D E P, E D E F Q, FE F D R = = = .
p dng nh l Pascal cho su im
2 1 1 1 2 2
E , E , D , F , F , D ta c:

2 1 1 2
1 1 2 2
1 1 2 2
E E FF A,
E D F D L,
D F D E P.
=
=
=

Suy ra A, L, P thng hng.
Tng t B, M, Q thng hng, C, N, R thng hng.
2 1 1 2 1 2 2 1 1 2 1 2 2 1 1 2 1 2
E E D F CA D F X, F F E D AB E D Y, D D FE BC FE Z = = = = = =
p dng nh l Pascal cho su im
1 1 1 2 2 2
F , E , D , D , F , E ta c:

1 1 2 2
1 1 2 2
1 2 2 1
FE D F R,
E D F E Q,
D D E F Z.
=
=
=

Suy ra Q, R, Z thng hng.
Tng t P, Q, Y thng hng, Z, P, X thng hng.
Xt cc tam gic ABC, PQR c: X CA RP, Y AB PQ, Z BC QR = = = .
p dng nh l Desargues suy ra cc ng thng AP AL, BQ BM, CR CN ng
quy.

Bi ton 8: (nh l Brianchon)
Lc gic ABCDEF ngoi tip mt
ng trn.
Khi AD, BE, CF ng quy.


Li gii:
Z
N
M
R
Q
P
L
F2
F1
E2
E1
D2 D1
A
B
C
N
P
M
A
B
C
D
E
F
H
G
L
K
J
I

=========================================================== 53
HI CC TRNG THTP CHUYN KHU VC DUYN HI V NG BNG BC B

Hi tho khoa hc mn Ton hc ln th III - 2010
Ta s chng minh nh l ny bng cc v i cc thy rng Pascal v Brianchon l
hai kt qu lin hp ca nhau.
Gi cc tip im trn cc cnh ln lt l G, H, I, J, K, L. Khi GH, HI, IJ, JK, KL,
LG ln lt l i cc ca B, C, D, E, F, A.
Gi GH JK N, HI KL P, IJ LG=M = =
Theo Pascal cho lc gic GHIJKL ta c M, N, P thng hng.
M M, N, P ln lt l i cc ca AD, BE, CF nn suy ra AD, BE, CF ng quy ti
cc ca ng thng MNP.

Bi ton 9:
Cho tam gic ABC, cc phn gic v ng cao ti nh B, C l BD, CE, BB, CC.
ng trn ni tip (I) tip xc vi AB, AC ti N, M.
Chng minh rng MN, DE, BC ng quy.
Li gii:
Gi hnh chiu ca C trn BD l P,
hnh chiu ca B trn CE l Q.
D chng minh:



0
A
NMI ICP NMI PMI 180
2
= = + =

Nn M, N, P thng hng.
Tng t suy ra M, N, P, Q thng
hng.
p dng nh l Pascal cho su
im B', C', B, P, Q, C, ta c:

B' C' PQ S,
C' B QC E,
BP CB' D.
=
=
=

Vy S, E, D thng hng, hay l MN, DE, BC ng quy ti S.

Bi ton 10:
Cho tam gic ABC ni tip ng trn (O). Tip tuyn ca (O) ti A, B ct nhau ti S.
Mt ct tuyn quay quanh S ct CA, CB ti M, N, ct (O) ti P, Q.
Chng minh rng M, N, P, Q l hng
im iu ha.
Li gii:
V tip tuyn ME, MD ca (O) ct SA,
SB ti K, L.
p dng nh l Newton cho t gic
ngoi tip SKML ta c BE, AD, SM, KL
ng quy.
P
Q
S
C'
B'
N
M
I
E
D
A
B C
I
L
K
D
E
M
N Q
S
A
B
C
P

=========================================================== 54
HI CC TRNG THTP CHUYN KHU VC DUYN HI V NG BNG BC B

Hi tho khoa hc mn Ton hc ln th III - 2010
p dng nh l Pascal cho su im A, D, E, E, B, C, ta c:

AD EB I,
DE BC N',
EE CA M.
=
=
=

Vy I, N', M thng hng, hay N N' , tc l N DE .
Do DE l i cc ca M i vi (O) nn M, N, P, Q l hng im iu ha.

Bi ton 11: (nh l Steiner)
ng thng Pascal ca cc lc gic ABCDEF, ADEBCF, ADCFEB ng quy.
Li gii:
Gi
1 1 2
AB DE P , BC EF Q , AD BC P , = = =
2 3 3
DE CF Q , AD FE P , CF AB Q . = = =
p dng nh l Pascal cho su im
A, B, C, F, E, D, ta c:

1 3 1 3
1 2 1 2
2 3 2 3
PQ Q P AB FE P,
PQ Q P BC ED Q,
Q Q P P CF DA R.
= =
= =
= =

Vy P, Q, R thng hng.
p dng nh l Desargues suy ra cc ng
thng
1 1 2 2 3 3
PQ , P Q , P Q ng quy.
Hay ng thng Pascal ca cc lc gic
ABCDEF, ADEBCF, ADCFEB ng quy.

Bi ton 12: (nh l Kirkman)
ng thng Pascal ca cc lc gic ABFDCE, AEFBDC, ABDFEC ng quy.

Ta bit trn l c 60 ng thng Pascal. C 3 ng mt ng quy to ra 20
im Steiner. Trong 20 im Steiner c 4 im mt li nm trn mt ng thng to ra 15
ng thng Plucker. Ngoi ra 60 ng thng Pascal li c 3 ng mt ng quy to
ra 60 im Kirkman. Mi im Steiner li thng hng vi 3 im Kirkman trn 20 ng
thng Cayley. Trong 20 ng thng Cayley, c 4 ng mt li ng quy to ra 15 im
Salmon
kt thc xin a ra mt s bi ton khc p dng nh l Pascal:

Bi ton 13: (MOSP 2005)
Cho t gic ni tip ABCD, phn gic gc A ct phn gic gc B ti E. im P, Q ln
lt nm trn AD, BC sao cho PQ i qua E v PQ song song vi CD.
Chng minh rng AP BQ PQ + = .

R
Q
P
Q3
P3
Q2
P2
Q1
P1
A F
B
C
D
E

=========================================================== 55
HI CC TRNG THTP CHUYN KHU VC DUYN HI V NG BNG BC B

Hi tho khoa hc mn Ton hc ln th III - 2010
Bi ton 14:
Cc im P, Q trong tam gic ABC sao cho

0
BP CP, BQ CQ, ABP ACQ 180 = = + = .
Chng minh rng

BAP CAQ = .

Bi ton 15: (IMO Shortlist 2007)
Cho tam gic ABC c nh, cc trung im
1 1 1
A , B , C ca BC, CA, AB tng ng.
im P thay i trn ng trn ngoi tip tam gic. Cc ng thng
1 1 1
PA , PB , PC ct li
ng trn ti A, B, C tng ng. Gi s cc im A, B, C, A, B, C i mt phn bit
v cc ng thng AA, BB, CC to ra mt tam gic.
Chng minh rng din tch ca tam gic khng ph thuc vo v tr ca P.

Bi ton 16:
Hai tam gic ABC, ABC c cng ng trn ngoi tip. Cc cnh ca hai tam gic
ct nhau ti 6 im to ra mt hnh lc gic.
Chng minh rng cc ng cho ca hnh lc gic ng quy.

Bi ton 17: (IMO 2010)
im P nm trong tam gic ABC vi CA CB . Cc ng AP, BP, CP ct li ng
trn ngoi tip ti K, L, M. Tip tuyn ca ng trn ngoi tip ti C ct AB S. Gi s
SC SP = .
Chng minh rng MK ML = .

Bi ton 18: (MEMO 2010)
ng trn ni tip tam gic ABC tip xc cc cnh BC, CA, AB ti D, E, F tng ng.
K l i xng ca D qua tm ng trn ni tip. DE ct FK ti S.
Chng minh rng AS song song BC.


















=========================================================== 56
HI CC TRNG THTP CHUYN KHU VC DUYN HI V NG BNG BC B

Hi tho khoa hc mn Ton hc ln th III - 2010
HM S HC V MT S BI TON V HM S HC

Trng THPT Chuyn Hng Yn


Hm s hc l hm s c min xc nh l tp con ca tp s t nhin .
K hiu A l tp hp tt c cc s hc.
I.Mt s tnh cht chung ca cc hm s hc
1.Tch chp Dirichlet (gi l tch chp)
nh ngha 1. Cho f v g l hai hm s hc. Tch chp Dirichlet gi l tch chp ca
f v , g k hiu l f * g , xc nh bi
( ) ( ) ( ) *, *
|
N n
d
n
g d f n g f
n d
|

\
|
=


trong tng ly trn tt c cc s nguyn dng d m chia ht n .
V d 1. Ta xt hai hm s hc sau nu
( )

=
0
1
n
( ) 1 = n e vi mi . * N n
Khi vi mi A f ta c
( ) ( ) ( ) ( ) n f
d
n
d f n f a
n d
= |

\
|
=

|
* ) vi mi * N n . * f f =
( ) ( ) ( ) ( )

= |

\
|
=
n d n d
d f
d
n
e d f n e f b
| |
* ) vi mi * N n .
nh l 1. Vi A h g f , , ta c
( ) ( )
( ) . * * * )
. * * * * )
. * * )
h f g f h g f iii
h g f h g f ii
f g g f i
+ = +
=
=

2. Ton t ( ) A f R a f T
a
, v ( ) A f Lf
nh ngha 2. Cho , R a vi mi A f ta xc nh ton t f T
a
nh sau
( ) ( ) . * N n n n f n f T
a
a
=
nh ngha 3. Vi mi A f ,ta xc nh ton t Lf nh sau
( ) ( ) . * ln N n n n f n Lf =
V d 2. Vi hai hm s hc v e , ta c
( ) ( ) ( ) = = =
a
a
a
T N n n n n n T * .
( ) ( ) . * N n n n n e n e T
a a
a
= =
( ) ( ) . * 0 ln N n n n n L = =
( ) ( ) . * ln ln N n n n n e n Le = =
nh l 2. Cho A g f , ta c
( )
( ) ( )
( ) . )
. , * * )
. * * * )
Lg Lf g f L iii
R a g T f T g f T ii
Lg f Lf g g f L i
a a a
+ = +
=
+ =

iv) K hiu
k
f * l tch chp ca f vi chnh n k ln, ngha l
nu n =1
nu * N n , n>1

=========================================================== 57
HI CC TRNG THTP CHUYN KHU VC DUYN HI V NG BNG BC B

Hi tho khoa hc mn Ton hc ln th III - 2010
. * ... * * * ; * ; *
1 0
f f f f f f f
k
= = =

k ln
Khi vi
+
Z k th ( )
( )
. * * *
1
A f Lf kf f L
k k
=


3. Hm s Mobius
nh ngha 4. Cho , A f hm A g c gi l nghch o tch chp ca f nu
= g f * , k hiu . *
1
= f g
nh l 3. Cho A g f , , ta c
i)
1
*

f tn ti khi v ch khi ( ) . 0 1 f
ii) Nu tn ti
1
*

f th
1
*

f c xc nh duy nht theo quy np nh sau


( )
( )
( )
( )
( )

>

\
|
=
=
n d
d
N n n
d
n
f d f
f
n f
f
f
|
1
1 1
1
. , 2 *
1
1
*
,
1
1
1 *

c bit nu p l s nguyn t th ( )
( )
( )
.
1
*
2
1
f
p f
p f =


iii) Nu tn ti
1
*

f th ( ) ( ). * * *
1
Lf f f Lf =


iv) Nu tn ti
1 1
* , *

g f th cng tn ti ( )
1
* *

g f v c xc nh nh sau
( ) =
1
* * g f
1 1
* * *

g f
Nhn xt: T v d 1 v nh l 3 suy ra lun lun tn ti
1
*

v
1
*

e v xc nh
( )
( )
( )
( )
( )
( ) ( )

>

> = = |

\
|
=
= = =
n d
d
n n
d
n
d n
|
1
1 1
1
. 1 0 *
1
1
*
, 1 1
1
1
1 *


=
1
* .
i vi hm s hc e vic tm
1
*

e s kh hn , ta s xt di y.
nh ngha 5. Hm Mobius c nh ngha l nghch o tch chp ca hm e,
ngha l
1
*

= e .
Nhn xt: T v d 1 v nh ngha hm Mobius ta suy ra kt qu quan trng sau
( ) ( )( ) ( )

>
=
= = =
n d
n nu
n nu
n n e d
|
. 1 0
, 1 1
*
nh l 4. Ta c
i) Nu * N n th
( ) ( )

=
0
1
1
r
n
ii) Nu A F f , th
( ) ( ) ( ) ( ) . *
| |
N n
d
n
F d n f d f n F
n d n d
|

\
|
= =


nu n = 1
nu n c phn tch tiu chun l n = p
1
p
2
p
r
, r>1
nu tn ti s nguyn t p sao cho p
2
|n.

=========================================================== 58
HI CC TRNG THTP CHUYN KHU VC DUYN HI V NG BNG BC B

Hi tho khoa hc mn Ton hc ln th III - 2010
(Cng thc o ngc kin-Liuvin)
T nh l trn ta c
( ) ( ) ( ) ( ) ( ) ( ) ( ) ( ) . 1 1 5 . 3 15 ; 0 2 8 ; 1 1 2 ; 1 1
2 3 1
= = = = = = = =


4. Hm nhn tnh
nh ngha 6. Cho A f v f khng ng nht bng khng,
Hm f c gi l hm nhn tnh nu ( ) ( ) ( ) n f m f mn f = vi mi * , N n m tha mn
(m,n) =1. Hm f c gi l hm hon ton nhn tnh nu ( ) ( ) ( ) n f m f mn f = vi mi
* , N n m .
K hiu M l tp hp tt c cc hm nhn tnh.
Nhn xt:
T nh ngha trn, gi s
k
k
p p p n

...
2 1
2 1
= l s phn tch tiu chun ca n, ta c kt
qu sau
Nu f l hm nhn tnh th ( ) ( ).
1

=
=
k
i
i
i
p f n f


Nu f l hm hon ton nhn tnh th ( ) ( ) ( )

p f p f = vi mi * N v p
nguyn t.
chng minh hai hm nhn tnh bng nhau ch cn chng minh chng bng nhau
trn mi ly tha ca cc s nguyn t.
chng minh hai hm hon ton nhn tnh bng nhau ch cn chng minh chng
bng nhau trn tp cc s nguyn t.
Vi f l hm nhn tnh, nu m = n = 1 th (m,n) = 1 suy ra ( ) ( ) ( ), 1 1 1 f f f = do vy
( ) 0 1 = f hoc ( ) 1 1 = f
Nu ( ) 0 1 = f th vi mi n cng c (n,1) = 1 ( ) ( ) ( ) 1 f n f n f = =0, nn f l
hm ng
nht bng khng, vy nu f l hm nhn tnh th ( ) 1 1 = f .
Nu
2 1
, f f l cc hm nhn tnh, hm tch f ca chng c nh ngha bi:
( ) ( ) ( ) *
2 1
N n n f n f n f = , k hiu
2 1
f f f = th f cng l hm nhn tnh.
Ta d dng kim tra c
- Vi mi * N n hm ( )
n
a a f = vi mi * N a l mt hm nhn tnh.
- Hm l hm nhn tnh.
- Hm e l hm hon ton nhn tnh.
nh l 5. ( tnh cht c bn ca hm nhn tnh)
Nu * , 1 N n n > v
k
k
p p p n

...
2 1
2 1
= l s phn tch tiu chun ca n th vi
mi hm nhn tnh f ta c
( ) ( ) ( ) ( ) ( ) ( ) ( ) ( ) 1 , ... 1 ... ... 1
|
1 1
1 k
k k
n d
p f p f p f p f d f

+ + + + + + =


trong tng v tri ly trn tt c cc c s dng d ca n.
Chng minh
Nu ta khai trin v phi (1), th ta s c mt tng gm cc s hng c dng
( ) ( ) ( )
k
k
p f p f p f

...
2 1
2 1
, trong
i i
0 vi mi i = 1,2,3,..,k.
Theo gi thit f l hm nhn tnh nn ( ) ( ) ( ) ( )
k k
k k
p p p f p f p f p f

... ...
2 1 2 1
2 1 2 1
=
nhng ( ) k i p p p
i i k
k
,..., 3 , 2 , 1 , 0 ...
2 1
2 1
=

chnh l mt c d ca n v mi c d ca n
u c dng . V vy v phi ca (1) l tng ca nhng s hng c dng ( ) d f , trong d
chy khp ch mt ln tt cc c dng ca n, chnh l v tri ca (1) ( pcm).

=========================================================== 59
HI CC TRNG THTP CHUYN KHU VC DUYN HI V NG BNG BC B

Hi tho khoa hc mn Ton hc ln th III - 2010
Nhn xt: Vn dng nh l trn i vi hm nhn tnh ( )
n
a a f = vi mi * N a v
k
k
p p p a

...
2 1
2 1
= l s phn tch tiu chun ca a th ta c
( ) ( ). ... 1 ... ... 1
2
|
1
2
1 1
1 k
n
k
n
k
n
k
n d
n n n n
p p p p p p d

+ + + + + + + + =


Nu n = 0 v k hiu d(a) l cc s c dng ca a th
( ) ( )( ) ( ). 1 ... 1 1 1
2 1
| |
0
+ + + = = =
k
a d a d
d a d
Nu n = 1 v k hiu ( ) a l tng cc c dng ca a th ( ) .
|

=
a d
d a
Do ( ) ( ) ( ) .
1
1
...
1
1
... 1 ... ... 1
1
1
1
1 2
1
2
1 1
1
1

= + + + + + + + + =
+ +
k
k
k k k
p
p
p
p
p p p p p p a
k
k



nh l 6. Cho R a v M f ta c
i)
1 *
f tn ti
ii) M f T
a
v nu f l hm hon ton nhn tnh th f T
a
cng l hm hon ton nhn
tnh.
iii) ( ) .
1 * 1 *
= f T f T
a a

Chng minh.
i) Ta c ( ) 1 1 = f suy ra
1 *
f tn ti (theo nh l 3 )
ii) Nu R a v * , N n m tha mn (m,n) = 1 th
( ) ( )( ) ( ) ( ) ( ) ( ). n f T m f T n n f m m f mn mn f mn f T
a a
a a a
a
= = =
v vy ta c M f T
a
.
Nu f l hm hon ton nhn tnh th ( ) ( ) ( ) * , N n m n f m f mn f = suy ra
( ) ( )( ) ( ) ( ) ( ) ( ). n f T m f T n n f m m f mn mn f mn f T
a a
a a a
a
= = =
hay f T
a
l hm hon ton nhn tnh.
iii) Do M f theo i) tn ti
1 *
f , v vy ta c
( ) = = =


a a a a
T f f T f T f T
1 * 1 *
* * ( ) .
1 * 1 *
= f T f T
a a

Da vo cc kt qu trn ta hon ton c th chng minh nh l sau
nh l 7. Cho . , A g f
i) Nu M g f th M g f * ,
ii) Nu . * M g th M g f v f
iii) Nu M f th .
1 *
M f


II.Mt s hm s hc thng gp
1. Tng cc c
nh ngha 7. Cho n l s nguyn dng, vi mi s thc ta gi hm

l tng ly
tha ca cc c dng ca n, ngha l ( ) . *
|
0
N n d n
n d
d
=

>


Khi 1 = ta vit ( )

>
= =
n d
d
N n d n n
|
0
1
*, ) ( ( ) n cn c gi l tng cc c
dng ca n.
Khi 0 = ta vit ( )

>
= =
n d
d
N n n d n
|
0
0
*, 1 ) ( trong d(n) l s cc c dng
ca n.

=========================================================== 60
HI CC TRNG THTP CHUYN KHU VC DUYN HI V NG BNG BC B

Hi tho khoa hc mn Ton hc ln th III - 2010
nh l 8. Ta c
i) . * R e T e =


ii)

l hm nhn tnh.
iii)Vi 1 , > n N n v n c phn tch tiu chun l
r
r
p p p n

...
2 1
2 1
= ta c cc kt qu
sau
Nu 0 th ( )
( )

=
+

=
r
i i
i
p
p
n
1
1
1


Nu 0 = th ( ) ( )

=
+ =
r
i
i
n
1
1


Chng minh
i) Nu
*
N n th
( )( ) ( ) ( ) ( ) ( ), *
| | | |
n d d d e d e T d e T
d
n
e n e T e
n d n d n d n d



= = = = |

\
|
=


suy ra . * e T e

=
ii) Do e T e

, l hm nhn tnh suy ra e T e

* l hm nhn tnh theo nh l 7, v


vy e T e

* = l hm nhn tnh.
iii) V

l hm nhn tnh nn ta ch cn tnh ( )


1

i
p vi , 1 r i
Nu 0 = th ( ) ( ) . 1
|
0
1

= =
i
i
p
i
k
i i
p d p


M
i
i
p

ch c cc c l
i
i i i
p p p

,..., , , 1
2
do

( ) ( ) ( ) 1
1
0
+ = =
i i i
i
p d p



Nu 0 th

( ) ( )
1
1
... 1
) 1 (
2
0
1

= + + + + = =
+
=

i
i
i i i
t
t
i i
p
p
p p p p p
i
i
i

V vy ta c ( )
( )

= +

=
+
0 1
0
1
1
) 1 (
1

nu
nu
p
p
p
i
i
i
i
i

T ta c kt qu cn chng minh
V d3: Tnh ( ) ( ) ( ) ( ) ( ). 10 ; 10 ; 18 ; 18 ; 18
2 0 2 0

Gii
( ) ( )
( ) ( )
( )
( )
( ) . 130
1 5
1 5
1 2
1 2
) 5 . 2 ( 10
4 ) 1 1 )( 1 1 ( ) 10 ( ) 5 . 2 ( 10
455
1 3
1 3
1 2
1 2
) 3 . 2 ( 18
. 42
1 2
1 2
1 3
1 3
3 . 2 18
6 ) 1 2 )( 1 1 ( 3 . 2 ) 18 ( 18
2
4
2
4
2 2
0 0
2
6
2
4
2
2 2
2 2
2
2
0 0
=

= =
= + + = = =
=

= =
=

= =
= + + = = =





d
d

2. S cc c
nh ngha 8. Cho k, n l cc s nguyn dng, ta gi hm d
k
(n) l cch vit n thnh
tch ca k nhn t, trong th t ca cc nhn t cng c tnh.
Nhn xt

=========================================================== 61
HI CC TRNG THTP CHUYN KHU VC DUYN HI V NG BNG BC B

Hi tho khoa hc mn Ton hc ln th III - 2010
d
1
(n)= 1; d
2
(n)= d(n) l s cc c dng ca n.
Theo nh ngha th d
k+1
(n) l s cch vit n nh l tch ca (k+1) nhn t hay l s
cch vit n c dng = n a
1
a
2
a
k+1
. Nu c nh a
k+1
|n th
1 + k
a
n
c th vit nh l
tch ca k nhn t v c
|
|

\
|
+1 k
k
a
n
d cch. V vy ta c

=
+
n i
k k
k i d n d
|
1
1 ) ( ) (
nh l 9. Cho * N k , ta c
i)
k
k
e d
*
=
ii) d
k
l hm nhn tnh
iii)Nu

=
=
r
i
i
i
p n
1

l s phn tch tiu chun ca n th vi mi 1 k ta c


( )

=

+
=
r
i
k
k k
i
C n d
1
1
1

Chng minh
i) Chng minh quy np theo k.
Nu k = 1 th d
1
(n) = 1 = e(n) suy ra d
1
=e
*1
.
Nu k = 2 th

= =
n d
n d n d
|
2
1 ) ( ) (
Ta li c ( ) , 1 ) ( ) ( * ) (
| |
2 *

= |

\
|
= =
n d n d
d
n
e d e n e e n e . * ) ( ) (
2 *
2
N n n e n d ra suy =
V vy ta c d
2
= e
*2
.
Gi s vi mi 1 1 k m ta c d
m
= e
*m
.
Khi ta c
( )
( ) ( )
( ) ) ( ) ( * ) (
) ( ) ( ) (
* 1 *
|
1 *
|
1 *
|
1
n e n e e i e
i
n
e
i e i d n d
k k
n i
k
n i
k
n i
k k
= = |

\
|
=
= =



hay d
k
= e
*k
* N k (pcm).
ii)V e l hm nhn tnh suy ra d
k
= e
*k
l hm nhn tnh (theo nh l 7).
iii) S dng kt qu: Nu m,n l cc s nguyn khng m th ) 1 (
1
1
0
+
+ +
=
+
=
m
n m
m
i
n
n i
C C
V d
k
l hm nhn tnh nn ta ch cn chng minh ( ) ,
1
1

+
=
k
k k
C p d

(2) vi p nguyn t,
* N . Tht vy ta c
Nu k = 1 th ( ) , 1
1
1
0
1

+
= = =
k
k
C C p d

suy ra (2) ng.


Nu k = 2 th ( ) ( ) , 1
1
1
1
1 2

+ +
= = + = =
k
k
C C p d p d


suy ra (2) ng.
Gi s vi mi k m 1 m ( ) ,
1
1

+
=
m
m m
C p d

khi ta c
( ) ( )
k
k
i i
k
k i
i
k k
C C p d p d
+
= =

+ +
= = =

0 0
1
1 1
( theo (1)) nn (2) ng vi k+1.
Vy (2) ng vi mi * N k
V vy nu

=
=
r
i
i
i
p n
1

l s phn tch tiu chun ca n th ( )



=

+
=
r
i
k
k k
i
C n d
1
1
1

3. Hm le ( ) n

=========================================================== 62
HI CC TRNG THTP CHUYN KHU VC DUYN HI V NG BNG BC B

Hi tho khoa hc mn Ton hc ln th III - 2010
nh ngha 9. Cho * N n , ta gi hm ( ) n l s cc s nguyn dng nguyn t vi n
v nh hn n, tc l ( ) . 1
1 ) , (
1

=

=
n d
n d
n
nh l 10. Ta c
i) ( ) . ) (
|

=
n k
k
n
k n
ii) . *
1
e T =
iii) l hm nhn tnh
iv) Nu n l s nguyn dng th ( )

|
|

\
|
=
n p
p
n n
|
1
1 .
Chng minh
i) T nhn xt ca nh ngha 5 ta c vi mi n>1 th
( ) =

0
|n d
d ( ) ( )

= =
=

= = =
n
d
n k
d k n d k
n
d
n d
n d
k k n
1
|
| ) , |( 1
1 ) , (
1
. ) ( 1 1
C nh k l l c ca n, ta phi ly tng vi nhng gi tr ca d tha mn n d 1 m
d chia ht cho k. Nu d=qk th do n d 1 suy ra
k
n
q 1 v
( ) ( ) ( )

= =
= = =
k n
q n k n k n k
k n
q
k
n
k k k n
/
1 | | |
/
1
. 1 ) ( (pcm)
ii). Theo i) ta c
( ) ( ) ( ) ( ) ( ) = |

\
|
= |

\
|
= =

e T
k
n
e T k
k
n
k
n
e k
k
n
k n
n k n k n k
1 1
| | |
* e T
1
* = .

iii). Theo ii) ta c e T
1
* = . M e T
1
, l hm nhn tnh nn l hm nhn tnh.
iv) V l hm nhn tnh xc nh ( ) n ta ch cn tnh gi tr ca ( ),

p vi
* N v p nguyn t. Ta c
( ) ( ) ( ) ( ) ( )

=
|
|

\
|
= = + = = =

=

n p i
i i
p k
p
n n
p
p p p p p p p p p
k
p
k p
i
\
1 1 0
0 |
).
1
1 ( ) (
1
1

III. Mt s bi tp v hm s hc
Bi 1. Cho * N n , ta c
a)

=
n d
n d
|
. ) ( ( H thc Gauss)
b) ( )

= |

\
|
n d
n
i
n
d i
|
. ) (
Gii
a) Do e T
1
* = v e(n)= 1 vi mi * N n nn ta c
( ) ( )
. ) )( ( ) (
) ( * * ) ( * ) ( ) (
1 *
1
|
1
| |

= = =
= = |

\
|
=


e v n n e T d
n e T e n e
d
n
e d d
n d
n d n d



b) Ta c ( )( ) ( )( ) ( )( ) ( )

= = = = |

\
|
n d
n n e e T n e e e T n d
i
n
d i
|
1 1
. * * * * * ) ( (theo nh l
8)

=========================================================== 63
HI CC TRNG THTP CHUYN KHU VC DUYN HI V NG BNG BC B

Hi tho khoa hc mn Ton hc ln th III - 2010
Nu n l s nguyn t
nu tn ti s nguyn t p sao cho p
2
|n,
nu n c phn tch tiu chun l n = p
1
p
2
p
r
,
r 2
nu n l s nguyn t
nu tn ti s nguyn t p sao cho p
2
|n,
nu n c phn tch tiu chun l n = p
1
p
2
p
r
,
r 2
Bi 2. t ( ) ( )

=
n d
N n d n
|
* . Chng minh rng
( )

=
1
0
1
n
Gii
S dng nh l 4 v cng thc tnh ( ) n , ta xt cc trng hp sau
Nu n l s nguyn t th ( ) ( ) ( ) 1 ) ( 1
|
= = =

n d n
n d

Nu tn ti s nguyn t p sao cho p
2
|n, th m p n

=
vi ( ) 1 , , 2 = p m v N suy ra
( ) ( ) ( ) 0 ) (
| |
= = =




p d
n d n d
p p d n
Nu n c phn tch tiu chun l n = p
1
p
2
p
r
,( r 2) th
( ) ( ) ( )

< < < =

= =
r i i i
r i i i
r i i
i i
r
i
i
n d
r
r
p p p p p p p p p d n
1 2 1
2 1
2 1
2 1
... 1
2 1
1 1 |
) ... ( ) ... ( )... ( ) ( ) ( 1

( ) ( ) ( ) ( ) ( ) ( )
( ) ( )
) ... 2 (
) 1 ( 3 2
2 1
1 3 2 1
1
1 1 ... 1 1 1
r
r r r
r
r
r
r r r r
rC C C
rC C r C C C
n
n
+ + +

=
=


M
( ) ( )
1
2
1 2 1
1 2 ... 2

= = + + +
r
r
r r
r r r
n ra suy r rC C C

V 2 r nn ( ) 1 1 r . Ta c 2 2
1

r
hay ( ) 1 = n
Do trong mi trng hp ta u c
( )

=
1
0
1
n
Bi 3
Cho * , N n M f
a) Nu n l, chng minh rng ( ) ( ) ( ). 1
| |
/

=
n d n d
d n
d f d f
b) Nu n chn, n = 2
s
m vi 1 s , m l, chng minh
rng ( ) ( ) ( ) ( ) ( ). 2 2 1
| | |
/

=
m k
s
n d n d
d n
k f f d f d f
c)Tnh gi tr ca tng S = ( ) ( ) ( )


n d
d n
d f
|
/
. 1 1
Gii
a)Nu n l, d|n suy ra d l v n|d cng l. Khi ta c ( ) 1 1
|
=
d n
v
( ) ( ) ( )

= =
n d n d n d
d n
d f d f d f
| | |
|
) ( ) ( 1 (pcm).
b)Nu n chn, n = 2
s
m vi 1 s v m l th c
Vi d|n suy ra d c dng 1 ,..., 2 , 1 , 2 = = s i m d
i
hoc , 2 k d
i
= trong i = 0,1,,s
v k|m.
Ta c

=========================================================== 64
HI CC TRNG THTP CHUYN KHU VC DUYN HI V NG BNG BC B

Hi tho khoa hc mn Ton hc ln th III - 2010
( ) ( ) ( ) ( ) ( ) ( )
( ) ( ) ( )
( )
( )
( ) ( ) ( ) ( )
( ) ( ) ( )( ) a dung p k f k f m f
k f k f m f
k f m f
k f m f d f
m k
s
s
i m k
i
s
i
i
m k
s k m
s
i m k
i
s
i
i
s
i m k
i
s
i
i
s
i m k
i k n
s
i
i m n
n d
d n
k
m
i s
i s
i i




+ =
+ + =
+ =
+ =

=
=

=
=

|
1
0 |
1
1
|
/
1
0 |
1
1
0 |
2
1
1
2
0 |
2 |
1
1
2 /
|
/
2 2 2
2 1 2 2
2 1 2 1
2 1 2 1 1

( ) ( ) ( )
( ) ( ) ( )


= =
+ =
=

=
m k
s s
n d
s
i m k
s
m k
i
s
i
i
k v k f f d f
k f k f m f
| |
0 | |
1
1
, 1 , 2 ) ( 2 2
2 2 2 2

suy ra ( ) ( ) ( ) ( )

=
m k
s
n d n d
d n
k f f d f d f
| | |
/
. 2 2 ) ( 1 (pcm)
c) Nu n l theo cu a) ta c ( ) ( ) ( ) ( ). 2 1 1
| |
/

= =
n d n d
d n
d f d f S
Nu n chn, n = 2
s
m, s 1,m l th theo cu b) ta c
( ) ( )

= =
m k
s
n d
d n
n d
k f f d f d f S
| |
/
|
) ( 2 2 ) ( 1 ) (
Do ta c

m k
s
n d
k f f
d f
S
|
|
) ( ) 2 ( 2
) ( 2

Nhn xt: T bi ton trn ta c cc kt qu sau
1. Nu = f , s dng h thc Gauss (bi 1) ta c
( ) ( )

0
1
|
/
n
d
n d
d n

2. Nu ( ) * N n n n f = th
( )

+
n d m k
s
n d
n d
d n
k d
d
d
| |
1
|
|
/
2
1
Bi 4. Chng minh cc ng thc sau
( ) ( )
( )
( )
( ) . * ) ( )
. * ) ( * ) ( )
. * ) ( * ) ( )
* / ) ( / ) ( )
2
| |
3
2
3
2
2 2
|
2
|
2
N n k d k d d
N n n d n d c
N n n d n d b
N n k n d k k n k d a
n k n k
n k n k

|
|

\
|
=
=
=
=



Gii
a)Theo nh ngha tch chp, ng thc cn chng minh tng ng vi vic chng
minh
nu n l
nu n chn, n = 2
s
m, s 1,m l
nu n l
nu n chn
nu n l
nu n chn, n = 2
s
m, s 1,m l

=========================================================== 65
HI CC TRNG THTP CHUYN KHU VC DUYN HI V NG BNG BC B

Hi tho khoa hc mn Ton hc ln th III - 2010
( )( ) ( )( ) n d n d * *
2 2
= (1)
Do , d l cc hm nhn tnh nn d v d d * * , ,
2 2 2 2
cng l hm nhn tnh. V
vy chng minh (1) ta ch cn chng minh ( )( ) ( )( ), * *
2 2
p d p d = vi p nguyn t
v 1
Tht vy ta c
( )( ) ( ) ( )
( ) ( ) ( ) ( )
( ) ( ) 1 2 1 1 1
1
*
2 2
1 2 2
0
2 2
+ = + + =
+ =
=


p p d p d
p p d p d
i
i i

( )( ) ( ) ( )
( ) ( ) ( ) ( )
( ) ( ) 1 2 1 1 1
1
*
1 2 2
0
2 2
+ = + + =
+ =
=


p d p p d
p d p p d
i
i i

suy ra ( )( ) ( )( ). * *
2 2
p d p d = (pcm)
b) Gi s
r
r
p p p n

...
2 1
2 1
= l s phn tch tiu chun ca n.
Khi
r
r
p p p n
2 2
2
2
1
2
...
2 1
= v ( ) ( )

=
+ =
r
i
i
n d
1
2
1 2
Mt khc theo cu a) ta c ( )( ) ( )( )

= =
+ = =
r
i
i
r
i
i
p d n d
1 1
2 2
) 1 2 ( * *
1



( ) ) ( * ) (
2 2
n d n d = (pcm)
c) Do
2
3
2
* , d d l hm nhn tnh. chng minh ng thc cho ta ch cn chng
minh ( )( )

p d p d
2
3
2
* ) ( = vi mi p nguyn t v 1 .
( )
( ) ( ) ( ) ( ) ( ) ( ) ( )
( ) , 1 1 2
1 ) * (
. 1 ) (
2 2 2
1
2
2
2
3
2
3
0
2
3
2
3
2 2
+ = + + = + =
+ = =
+ =
+ +

C C
p p d p d p p d p d
p d
i
i
i i

suy ra ( )( )

p d p d
2
3
2
* ) ( = ( ) . * ) ( * ) (
2
3
2
N n n d n d =
d) Ta c
( ) ( ) ) ( * / ) ( ) (
3
|
3
|
3
n e d k n e k d k d
n k n k
= =


( )( ) ( ) ( ) ( ) n e d n e d k n e k d k d
n k n k
2 2
2
|
2
|
* * ) / ( ) ( ) ( = =
|
|

\
|
=
|
|

\
|


Do ng thc cn chng minh tng ng vi phi chng minh
( )( ) ( ) ( ) * * *
2 3
N n n e d n e d =
hay ( )( ) ( ) ( ), * *
2 3
p e d p e d = vi mi p nguyn t v 1 .
Tht vy ta c

=========================================================== 66
HI CC TRNG THTP CHUYN KHU VC DUYN HI V NG BNG BC B

Hi tho khoa hc mn Ton hc ln th III - 2010
( )( ) ( ) ( ) ( ) ( )
( )
( ) ( )
( )( ) ( ) ( ) ( )
( ) ( )
( )( ) ( ) ( )
4
2 1
4
2 1
1 ... 2 1
1 *
4
2 1
1 ... 2 1
1 *
2 2 2
2
2
0
2
0
2 2
3 3 3
0
3
0
3
0
3 3
+ +
= |

\
| + +
= + + + + =
|

\
|
+ = |

\
|
=
+ +
= + + + + =
+ = = =


= =

= = =





i i
i i
i i
i
i
i i
i p e p d p e d
i p d p e p d p e d

( )( ) ( ) ( ), * *
2 3
p e d p e d = vy ta c pcm ( )
2
| |
3
) (
|
|

\
|
=

n k n k
k d k d
Bi tp t luyn
Bi 1. Cho M f , chng minh rng
a) f l hm hon ton nhn tnh khi v ch khi . *
1 *
f f f =


b) ( ) ( ) ( ) ( ) p p f d f d
n p n d
, 1
| |

= nguyn t.
c) ( ) ( ) ( ) ( ) p p f d f d
n p n d
, 1
| |
2

= nguyn t.
d).Nu n l s nguyn dng, k hiu ( ) n w l s cc c nguyn t phn bit ca n th
( )
( ) n w
n d
d 2
|
2
=


Bi 2. Gi ) (
0
n l k hiu tng cc c l ca s nguyn dng n . Chng minh rng
a) . ) 1 ( ) (
/
||
0
d n
d n
n d

=
b) ), 2 / ( 2 ) ( ) (
0
n n n = vi n l s chn.
c) ) (
0
n l hm nhn tnh.
Bi 3. Chng minh rng
a) . * , 2 ) ( N n n n d
b) . 2 , ) ( ) ( 2 + n N n n n n
c) . * , 2 ) ( ) (
2 / ) 1 (
N n N k n n d n n n
k k
k
k

+

d) . * ), ( ) ( N n n d n n














=========================================================== 67
HI CC TRNG THTP CHUYN KHU VC DUYN HI V NG BNG BC B

Hi tho khoa hc mn Ton hc ln th III - 2010
MT S BI TON S HC
TRONG CC K THI OLYMPIC TON

Trn Xun ng
(THPT Chuyn L Hng Phong Nam nh)

Trong k thi Olympic ton Quc t ln th 49 c t chc ti Ty Ban Nha c bi
ton sau (bi ton 1) m tc gi ca n l Kestutis Cesnavicius (Lithuania) (Litva).
Bi ton 1: Chng minh rng tn ti v s s nguyn dng n sao cho
2
1 n + c c
nguyn t ln hn 2 2 n n +
Bi ton ny l bi ton kh nht ca ngy thi th nht. Li gii ca bi ton 1 c
pht trin t li gii ca cc bi ton n gin hn sau y:
Bi ton 2: Chng minh rng tn ti v s s nguyn dng n sao cho n
2
+ 1 khng l
c ca n!.
( thi chn i tuyn ca Innxia d thi Ton Quc t nm 2009) .
Li gii ca bi ton 2:
B : Tn ti v s s nguyn t dng 4k + 1 (k N
*
)
Chng minh: Gi A l tp hp gm tt cc s nguyn t dng 4k+1 (k N
*
) ,
Khi A rng v 5 A. Gi s A l tp hu hn. Gi p
0
l phn t ln nht ca A p
0

5 .
Gi s p
1
, p
2
p
n
l tt c cc s nguyn t nh hn p
0
.
t
2 2 2
0 1
4 ... 1
n
a p p p = + khi ? a N
*
, a > 1. Gi s q l c nguyn t ca a
q p
i
, i {0,1,2 , n}. Mt khc (2p
0
p
1
p
n
)
2
+ 1 0 (modq)
- 1 l s chnh phng (modq) v q l.
Suy ra q
q
q
q

= =
|
|

\
|

2 :
2
1
1 ) 1 ( 1
1
2
1
) 4 (mod 1 q c dng 4k + 1 (k N
*
).
Mt khc q> p
0
. iu ny mu thun vi cch chn p
0
. Vy tn ti v s s nguyn t dng
4k + 1 (kN
*
).
Chng ta chuyn sang vic gii bi ton 2. Gi s p l s nguyn t dng 4k + 1 (k
N
*
) 1 1 ) 1 (
1
2
1
= =
|
|

\
|
p
p
l s chnh phng (modp)
n
p
{ 0,1,2 . ,p - 1} sao cho
2
p
n ) (mod 1 p
2
p
n +1: p v n
p
! khng chia ht
cho p n
p
! khng chia ht cho
2
p
n + 1. Ta c:
2
p
n + 1 p n
p
1 p . V tn ti v

=========================================================== 68
HI CC TRNG THTP CHUYN KHU VC DUYN HI V NG BNG BC B

Hi tho khoa hc mn Ton hc ln th III - 2010
s s nguyn t p dng 4k + 1 (kN
*
) nn tn ti v s s nguyn dng n sao cho n
2
+ 1
khng l c ca n!
Bi ton 3: Chng minh rng tn ti v s s nguyn dng n sao cho c nguyn t
ln nht ca n
2
+ 1

ln hn 2n
(Tp ch Animath ca Php nm 2006)
Li gii ca bi ton 3: Gi s p l s nguyn t dng 4k + 1 (k N
*
)
Suy ra 1 1 ) 1 (
1
2
1
= =
|
|

\
|
p
p
l s chnh phng (modp)
x {0,1,2, ,p - 1} sao cho x
2
- 1(modp).
Ta c: q
2
(p- q)
2
(modp) (q Z)
q {0,1,2, ,
2
1 p
} sao cho q
2
-1 (modp).
Tht vy gi s
2
1 p
< x < p x
2
1 + p
. t q = p x, ta c:
q
2
= ( p x)
2
x
2
- 1 (modp) v 0 < q
2
1 p
. Ta c: q
2
+1 M p v
p 2q +1 > 2q. Suy ra c nguyn t ln nht ca q
2
+1 ln hn 2q.V c v s s
nguyn t dng 4k + 1(k N
*
) nn tn ti v s s nguyn dng n sao cho n
2
+1 c c
nguyn t ln hn 2n.
Sau y l cc li gii ca bi ton 1
Li gii th nht ca bi ton 1: Xt s nguyn t p dng 4k + 1 (k N
*
)
1 1 ) 1 (
1
2
1
= =
|
|

\
|
p
p
l s chnh phng (modp)
x {0,1,2, p - 1} sao cho x
2
- 1(modp).
V x
2
(p- x)
2
(modp) (x Z) x {0,1,2 ,
2
1 p
} sao cho
x
2
-1 (modp). {0,1,2, ,
2
1 p
} sao cho
|

\
|

2
1 p
2
-1 (modp)
t m =
|

\
|

2
1 p
m {0,1,2, ,
2
1 p
} v m
2
-1 (modp)
Gi s p > 20. N?u 0
+
4
3 1 4p
0 < 2 +1
+
2
1 1 4p

(2 +1)
2
<p - 4 V m
2
-1 (modp) nn 4m
2
-4 (modp)
Mt khc 4m
2
= (p 1 -2 )
2
(2 +1)
2
(modp) (2 +1)
2
-4 (modp)

=========================================================== 69
HI CC TRNG THTP CHUYN KHU VC DUYN HI V NG BNG BC B

Hi tho khoa hc mn Ton hc ln th III - 2010
iu l iu v l v 0 < (2 +1)
2
< p 4
Vy >
+
4
3 1 4p
p > 2m + m 2 . V m
2
+1 M p nn m
2
p -1 m
1 p . V tn ti v s s nguyn t p dng 4k + 1 (k N
*
) nn tn ti v s s nguyn
dng n sao cho c nguyn t ln nht ca n
2
+ 1 ln hn 2 2 n n + .
Li gii th 2 ca bi ton 1: Gi s n l s nguyn, n 24. Gi s p l c nguyn
t ca (n!)
2
+ 1. Hin nhin p > n. Gi s x (0,
2
p
) l s d trong php chia n ! hoc n!
cho p. Khi ? 0 < x< p x < p. Ta c? x
2
+ 1 chia ht cho p. Tht vy tn ti m Z sao cho n!
= mp + x hoc n! = mp + x. Trong c hai trng hp ta u c (n!)
2
+1 = (mp+x)
2
+1 x
2

+1 = (n!)
2
+ 1 m
2
p
2
2mpx x
2
+1M p . T suy ra p l c ca p
2
- 2px + 4x
2
+ 4 = (p
2x)
2
+ 4
p (p 2x)
2
+ 4 p 2x + 4 p
p - 4 2x + 4 p - 4 2x + 20 4> 2x
p 2x + 4 p > 2x + x 2 T y suy ra iu phi chng minh
Bi ton sau l bi ton tng qut ca bi ton 1
Bi ton 4: Chng minh rng tn ti v s s nguyn dng n sao cho n
2
+ 1 c c
nguyn t ln hn 2n + 2 n
Bi ton 5: Chng minh rng vi mi s nguyn n 3, tn ti cp s nguyn dng
l (x
n
, y
n
) sao cho
n
n n
y x 2 7
2 2
= +
( thi Olympic Ton ca Bungari nm 1996)
Li gii: Vi n = 3 , chn x
3
= y
3
= 1
Gi s vi n 3 , tn ti cp s nguyn dng l (x
n
,y
n
) sao cho
n
n n
y x 2 7
2 2
= + . Ta
chng minh rng mi cp .
(X=
2
7
,
2
n n
n n
y x
Y
y x
=
+
) , (X=
2
7
,
2
n n
n n
y x
Y
y x +
=

) tho mn
1 2 2
2 7
+
= +
n
Y X
Tht vy
2 2
2
7
2
7
|

\
|
+
|

\
|
n n n n
y x y x m
= 2 (
2 2
7
n n
y x + ) = 2. 2
n
= 2
n+1
V x
n
, y
n
l nn x
n
= 2k + 1, y
n
= 2l + 1 (k, l Z) 1
2
+ + =
+
l k
y x
n n


=========================================================== 70
HI CC TRNG THTP CHUYN KHU VC DUYN HI V NG BNG BC B

Hi tho khoa hc mn Ton hc ln th III - 2010
v l k
y x
n n
=

2
. iu chng t rng mt trong cc s
2
,
2
n n
n n
y x y x +
l l .
V vy vi n +1 tn ti cc s t nhin l x
n+1
v y
n+1
tho mn
n
n n
y x 2 7
2
1
2
1
= +
+ +
+1
Bi ton 6 : Chng minh rng vi mi s nguyn dng n, phng trnh x
2
+ 15y
2
=
4
n
c t nht n nghim t nhin (x,y)
( thi chn hc sinh gii Ton Quc gia nm hc 2009 2010)
Li gii: Trc ht ta chng minh rng vi mi s nguyn n 2 tn ti cp s nguyn
dng l (x
n
, y
n
) sao cho sao cho
n
n n
y x 4 15
2 2
= +
Tht vy vi n = 2 , chn x
2
= 1 , y
2
= 1
Gi s vi n 2 tn ti cp s nguyn dng l (x
n
, y
n
) sao cho sao cho
n
n n
y x 4 15
2 2
= + . Ta chng minh rng mi cp
(X=
2
,
2
15
n n
n n
y x
Y
x y +
=

), (X=
2
,
2
15
n n
n n
x y
Y
x y
=
+
) tho mn
1 2 2
4 15
+
= +
n
Y X
Tht vy
2 2
2
15
2
15
|

\
|
+ |

\
|
n n n n
x y x y m
= 4 (
2 2
15
n n
y x + ) = 4. 4
n
= 4
n+1
V x
n
, y
n
l nn x
n
= 2k + 1, y
n
= 2l + 1 (k, l Z) 1
2
+ + =
+
l k
y x
n n

v k l
k l x y
n n
=
+ +
=

2
) 1 2 ( ) 1 2 (
2
. iu chng t rng mt trong cc s
2
,
2
n n n n
x y y x +
l l . V vy vi n +1 tn ti cc s t nhin l x
n+1
v y
n+1
tho mn
1 2
1
2
1
4 15
+
+ +
= +
n
n n
y x


Tr li bi ton 6:
Vi n = 1, phng tnh
n
y x 4 15
2 2
= + c 1 nghim t nhin l (x,y) = (2,0)
Vi n = 2, phng tnh
n
y x 4 15
2 2
= + c 2 nghim t nhin l (x,y)= (4,0); (1,1)
Gi s vi n 2, phng tnh
n
y x 4 15
2 2
= + c n nghim t nhin l (x
1
,y
1
), (x
2
,y
2
), ,
(x
n
, y
n
) khi (x,y) = ( 2x
k
, 2y
k
) (1 k n) l cc nghim t nhin ca phng trnh
n
y x 4 15
2 2
= +
+1
. Theo chng minh trn phng trnh
n
y x 4 15
2 2
= +
+1
li c 1 nghim t
nhin l. Vy phng tnh
n
y x 4 15
2 2
= +
+1
c t nht n+1 nghim t nhin. Bi ton 6
c gii quyt.

=========================================================== 71
HI CC TRNG THTP CHUYN KHU VC DUYN HI V NG BNG BC B

Hi tho khoa hc mn Ton hc ln th III - 2010
Bi ton 7: Tm tt c cc cp s nguyn dng (x,y) sao cho
y x
y x

+
2 2
l s nguyn
v l c ca 1995.
( thi Olympic ton Bungari nm 1995)
Li gii : Trc ht ta chng minh
B : Cho s nguyn t p = 4q + 3 (q N). Gi s x, y l cc s nguyn sao cho x
2
+
y
2
chia ht cho p, Khi x v y chia ht cho p. Tht vy nu x M p th y M p .
Gi s x khng chia ht p y khng chia ht cho p
Theo nh l nh Phecma ta c x
p-1
1 (modp) x
4q+2
1 (modp). Tng t y
4q+2

1 (modp) . Ta c x
2
+ y
2
M p x
2
-y
2
(modp)
(x
2
)
2q+1
(-y
2
)
2q+1
(modp) x
4q+2
- y
4q+2
(modp) 1 - 1 ( modp) p = 2
(v l). B c chng minh.
p dng b vo bi ton 7: Gi s tn ti cc s nguyn dng x,y sao cho x> y ,
y x
y x

+
2 2
l s nguyn v
y x
y x

+
2 2
l c ca 1995 . t k =
y x
y x

+
2 2
th x
2
+y
2
= k( x y) v
k l c ca 1995 = 3.5.7.19.N?u k M 3 th k= 3 k
1
(k
1
N
*
) (k
1
khng chia ht cho 3) x
2

+ y
2
M 3 x M 3 v y M 3 x = 3x
1
, y = 3y
1
(x
1
, y
1
N
*
, x
1
> y
1
) ) (
1 1 1
2
1
2
1
y x k y x = + .
N?u k = 1 th x
2
+ y
2
= x y . l iu v l v x
2
+ y
2
x + y > x y (v x,y 1 )
Nu k = 5 th x
2
+ y
2
= 5(x y) (2x - 5)
2
+ (2y +5)
2
= 50 x = 3 , y = 1 hoc x = 2
, y = 1
Nu k = 7 , tng t nh trn, tn ti k
2
N
*
sao cho k = 7 k
2
(k
2
khng chia ht cho
7) x = 7x
2
, y = 7y
2
(x
2
, y
2
N
*
, x
2
> y
2
) v ) (
2 2 2
2
2
2
2
y x k y x = +
Nu k M 19 th tn ti k
3
N
*
sao cho k = 19k
3
(k
3
khng chia ht cho 19 ), x = 19x
3
,
y = 19y
3
(x
3
, y
3
N
*
, x
3
> y
3
) v ) (
3 3 3
2
3
2
3
y x k y x = +
Vy tt c cc cp s nguyn dng (x,y) cn tm c dng (3c, c), (2c, c), (c, 2c), (c,
3c) trong c {1,3,7,19,21,57,133,399} .
Bi ton 8: Tm tt c cc cp s nguyn dng (x,y) sao cho s A =
y x
y x

+
2 2
l s nguyn v l c ca 2010.
( thi Olympic Ton khu vc duyn hi ng bng Bc B nm hc 2009 2010)
Li gii: Trn c s li gii ca bi ton 7 ta ch cn tm cc nghim nguyn dng
ca cc phng trnh : ) (
2 2
y x k y x = + vi k { 2,5, 10}. Phng tnh x
2
+ y
2
= 2 (x- y)
khng c nghim nguyn dng . Tht vy gi s x,y N
*
, x > y v x
2
+ y
2
= 2 (x- y)

=========================================================== 72
HI CC TRNG THTP CHUYN KHU VC DUYN HI V NG BNG BC B

Hi tho khoa hc mn Ton hc ln th III - 2010
x
2
+ y
2
2x +y
2
> 2(x y). l iu v l . Phng trnh x
2
+ y
2
= 5 (x- y) c cc
nghim nguyn dng l (x,y) = (3,1), (2,1). Phng trnh x
2
+ y
2
= 10 (x- y) (x-5)
2
+
(y+5)
2
= 50 c cc nghim nguyn dng l (x,y) = (6,2) ; (4,2) .
Vy tt c cc cp s nguyn dng (x, y) tho mn bi l (3c, c), (2c, c), (c, 2c) ,
(c, 3c) , (6c, 2c) , (4c, 2c) , ( 2c, 6c), (2c, 4c) trong
c {1,3,6,7,201}
Cui cng l mt s bi ton dnh luyn tp
Bi ton 9: Chng minh rng vi mi s nguyn dng n, phng tnh
7x
2
+ y
2
= 2
n+2
lun c nghim nguyn dng.
Bi ton 10: Chng minh rng vi mi s nguyn dng n, phng trnh x
2
+ 15y
2
=
4
n
c ng n nghim t nhin .
Bi ton 11: Cho s nguyn dng n. Gi S
n
l tng cc bnh phng ca cc h s
ca a thc f(x) = (1+x)
n
.
Chng minh rng S
2n
+1 khng chia ht cho 3
( thi chn i tuyn Vit Nam d thi Olympic Ton Quc t nm 2010)

Bi ton 12: Chng minh rng tn ti v s s nguyn dng n sao cho
2
n
+2 chia ht cho n .
Bi ton 13: Chng minh rng tn ti v s s nguyn dng n sao cho tt c cc c
nguyn t ca n
2
+ n + 1 khng ln hn n .
( thi chn i tuyn Ukraina d thi Olympic ton quc t nm 2007)
Bi ton 14: Vi mi s nguyn dng n > 1, k hiu p(n) l c nguyn t ln nht
ca n. Chng minh rng tn ti v s s nguyn n > 1 sao cho:
p(n) < p(n+1) < p(n+2) .
Bi ton 15: Cho cc s nguyn a,b tho mn a>b > 0 . Chng minh rng tn ti v
s s nguyn dng n sao cho a
n
+ b
n
chia ht cho n .
Bi ton 16: Chng minh rng tn ti v s s nguyn t p c tnh cht sau: Tn ti
v s nguyn dng n sao cho p 1 khng chia ht cho n v n! +1 chia ht cho p.
( thi chn i tuyn ca Mnva d thi Olympic ton Quc t nm 2007).
Bi ton 17: Chng minh rng tn ti v s s nguyn dng n sao cho 5
n-2
1
chia ht cho n.
( thi Olympic ton ca Braxin nm 2008)



=========================================================== 73
HI CC TRNG THTP CHUYN KHU VC DUYN HI V NG BNG BC B

Hi tho khoa hc mn Ton hc ln th III - 2010
NH L LAGRANGE V NG DNG
ng nh Sn
Chuyn Lng Vn Ty Ninh Bnh

1. NH L LAGRANGE
1.1. NH L ROLLE
nh l: Nu ( ) f x

l hm lin tc trn on [ ; ] a b , c o hm trn khong ( ; ) a b
v ( ) ( ) f a f b = th tn ti ( ; ) c a b

sao cho '( ) 0 f c = .
Chng minh:
V ( ) f x

lin tc trn [a; b] nn theo nh l Weierstrass ( ) f x

nhn gi tr ln nht
M v gi tr nh nht m trn [a; b].
- Khi M = m ta c ( ) f x

l hm hng trn [a; b], do vi mi ( ; ) c a b

lun c
'( ) 0 f c = .
- Khi M > m, v ( ) ( ) f a f b =

nn tn ti c (a; b)

sao cho ( ) f c m = hoc
( ) f c M = , theo b Fermat suy ra '( ) 0 f c = .
H qu 1: Nu hm s ( ) f x

c o hm trn (a; b) v ( ) f x

c n nghim (n l s
nguyn dng ln hn 1) trn (a; b) th '( ) f x

c t nht n - 1 nghim trn (a; b).
H qu 2: Nu hm s ( ) f x

c o hm trn (a; b) v '( ) f x

v nghim trn (a;
b) th ( ) f x c nhiu nht 1 nghim trn (a; b).
H qu 3: Nu ( ) f x c o hm trn (a; b) v '( ) f x

c nhiu nht n nghim (n l
s nguyn dng) trn (a; b) th ( ) f x

c nhiu nht n + 1 nghim trn (a; b).
Cc h qu trn c suy ra trc tip t nh l Rolle v n vn ng nu cc nghim
l nghim bi (khi ( ) f x

l a thc).
Cc h qu trn cho ta tng v vic chng minh tn ti nghim cng nh xc
nh s nghim ca phng trnh, v nu nh bng mt cch no ta tm c tt c cc
nghim ca phng trnh (c th do m mm) th ngha l khi phng trnh c gii.
T nh l Rolle cho php ta chng minh nh l Lagrange, tng qut hn, ch cn ta
n ti ngha ca o hm (trung bnh gi tr bin thin ca hm s).
1.2. NH L LAGRANGE (Lagrange's Mean Value Theorem)

=========================================================== 74
HI CC TRNG THTP CHUYN KHU VC DUYN HI V NG BNG BC B

Hi tho khoa hc mn Ton hc ln th III - 2010
nh l: Nu ( ) f x l hm lin tc trn on
[ ; ] a b , c o hm trn khong ( ; ) a b th tn ti
( ; ) c a b

sao cho
( ) ( )
'( )
f b f a
f c
b a

.
Chng minh:
Xt hm s:
( ) ( )
( ) ( )
f b f a
F x f x x
b a

.
Ta c: F(x) l hm lin tc trn on [ ; ] a b , c
o hm trn khong ( ; ) a b

v ( ) ( ) F a F b = .
Theo nh l Rolle tn ti ( ; ) c a b

sao cho '( ) 0 F c = .
M
( ) ( )
'( ) '( )
f b f a
F x f x
b a

, suy ra
( ) ( )
'( )
f b f a
f c
b a

.
nh l Rolle l mt h qu ca nh l Lagrange (trong trng hp ( ) ( ) f a f b = )
ngha hnh hc:




nh l Lagrange cho
php ta c lng t s
( ) ( ) f b f a
b a


do n cn c gi l nh l Gi tr trung bnh
(Mean Value Theorem). T cho ta tng chng minh cc nh l v s bin thin ca
hm s, t nn mng cho nhng ng dng ca o hm.
nh l: Cho hm s ( ) f x c o hm trn khong ( ; ) a b .
- Nu '( ) 0, ( ; ) f x x a b > th ( ) f x

ng bin trn ( ; ) a b .
- Nu '( ) 0, ( ; ) f x x a b < th ( ) f x

nghch bin trn ( ; ) a b .
- Nu '( ) 0, ( ; ) f x x a b = th ( ) f x

l hm hng trn ( ; ) a b .
Chng minh:
Gi s '( ) 0, ( ; ) f x x a b > v
1 2 1 2
, ( ; ), x x a b x x < , theo nh l Lagrange, tn ti
1 2
c (x ; x )

sao cho
2 1
2 1
( ) ( )
'( )
f x f x
f c
x x

.
M
1 2
'( ) 0 ( ) ( ) ( ) f c f x f x f x > < ng bin trn (a; b).
Cho hm s ( ) f x tha mn cc gi thit ca
nh l Lagrange, th (C), A(a;f(a)), B(b;f(b)).
Khi trn (C) tn ti im C(c;f(c)), c (a; b) m
tip tuyn ca (C) ti C song song vi ng thng AB.

Joseph Louis Lagrange (1736 -
1813)

=========================================================== 75
HI CC TRNG THTP CHUYN KHU VC DUYN HI V NG BNG BC B

Hi tho khoa hc mn Ton hc ln th III - 2010
Nu trong gi thit ca nh l Lagrange ta thm vo gi thit '( ) f x ng bin hoc
nghch bin trn [a; b] th ta c th so snh
( ) ( ) f b f a
b a

vi '( ), '( ) f a f b .
C th: '( ) f x ng bin trn [a;b]
( ) ( )
'( ) '( )
f b f a
f a f b
b a

< <


'( ) f x nghch bin trn [a;b]
( ) ( )
'( ) '( )
f b f a
f a f b
b a

> >


T y cho ta tng ng dng nh l Lagrange chng minh bt ng thc v nh
gi cc tng hu hn.
Cng tng t nu trong gi thit ca nh l Lagrange ta thm vo gi thit '( ) f x
ng bin hoc nghch bin trn [a; b] th ta c th so snh
( ) ( ) f c f a
c a

vi
( ) ( ) f b f c
b c


vi [ ; ] c a b cho ta tng chng minh rt nhiu bt ng thc, nh bt ng thc
Jensen
Ngoi ra nh l Lagrange cn c pht biu di dng tch phn nh sau:
nh l: Nu ( ) f x l hm lin tc trn on [a; b] th tn ti im ( ; ) c a b tha
mn: ( ) ( )( )
b
a
f x dx f c b a =


nh l Lagrange dng tch phn c p dng chng minh mt s bi ton lin
quan n tch phn v gii hn hm s.
2. MT S NG DNG
2.1. CHNG MINH S TN TI NGHIM CA PHNG TRNH
Bi ton 1. Chng minh rng phng trnh acosx + bcos2x + ccos3x lun c nghim
vi mi b cc s thc a, b, c.
Li gii:
Xt
bsin2x sin3x
( ) asinx+ '( ) osx+bcos2x+ccos3x, x .
2 3
c
f x f x ac R = + =
M
0 0
(0) ( ) 0 (0; ), '( ) 0 f f x f x = = = , suy ra iu phi chng minh.
Nhn xt: Bi ton trn c dng tng qut:
Cho hm s f(x) lin tc trn [a; b], chng minh rng phng trnh f(x) = 0 c t nht
mt nghim trn (a; b).
Phng php gii:
Xt hm F(x) tha mn F(x) lin tc trn [a; b], F(x) = f(x).g(x) vi mi x thuc (a;
b), g(x) v nghim trn (a;b) v F(a) = F(b). Theo nh l Rolle suy ra iu phi chng minh.

=========================================================== 76
HI CC TRNG THTP CHUYN KHU VC DUYN HI V NG BNG BC B

Hi tho khoa hc mn Ton hc ln th III - 2010
Bi ton 2. Cho s thc dng m v cc s thc a, b, c tha mn:
0
2 1
a b c
m m m
+ + =
+ +
.
Chng minh rng ax
2
+ bx + c = 0 c nghim thuc (0; 1).
Hng dn: Xt hm s
2 1
. . .
( )
2 1
m m m
a x b x c x
f x
m m m
+ +
= + +
+ +
.
Tng t ta c bi ton tng qut hn.
Bi ton 3. Cho s thc dng m, s nguyn dng n v cc s thc
0 1
, ,...,
n
a a a

tha mn:
1 0
... 0
1
n n
a a a
m n m n m

+ + + =
+ +
.
Chng minh rng
1
1 1 0
... 0
n n
n n
a x a x a x a

+ + + + =

c nghim thuc (0; 1).
Hng dn: Xt hm s
1 1 0
( ) ...
1
m n m n m n n
a a a
f x x x x
m n m n m
+ +
= + + +
+ +

Bi ton 4.(nh l Cauchy)
Nu cc hm s ( ), ( ) f x g x l cc hm s lin tc trn on [ ; ] a b , c o hm trn
khong ( ; ) a b v '( ) g x khc khng trn khong ( ; ) a b th tn ti ( ; ) c a b

sao cho
( ) ( )
'( )
( ) ( )
f b f a
f c
g b g a

.
Li gii: Theo nh Lagrange lun tn ti
0
( ; ) x a b sao cho
0
( ) ( )
'( )
g b g a
g x
b a

( ) ( ) g a g b .
Xt hm s
( ) ( )
( ) ( ) ( )
( ) ( )
f b f a
F x f x g x
g b g a

, ta c: F(x) l hm lin tc trn on


[ ; ] a b , c o hm trn khong ( ; ) a b

v
( ) ( ) ( ) ( )
( ) ( )
( ) ( )
f a g b f b g a
F a F b
g b g a

= =

.
Theo nh l Rolle tn ti ( ; ) c a b

sao cho '( ) 0 F c = .
M
( ) ( )
'( ) '( )
( ) ( )
f b f a
F x f x
g b g a

, suy ra
( ) ( )
'( )
( ) ( )
f b f a
f c
g b g a

.
Nhn xt: nh l Lagrange l h qu ca nh l Cauchy (trong trng
hp ( ) g x x = )
Bi ton 5: Cho a + b c = 0. Chng minh rng: asinx+9bsin3x+25csin5x = 0 c t
nht 4 nghim thuc [0; ].

=========================================================== 77
HI CC TRNG THTP CHUYN KHU VC DUYN HI V NG BNG BC B

Hi tho khoa hc mn Ton hc ln th III - 2010
Nhn xt: Bi ton ny cng tng t cc bi ton trn. chng minh ( ) f x c t
nht n nghim ta chng minh F(x) c t nht n + 1 nghim vi F(x) l mt nguyn hm ca
( ) f x trn (a;b) (c th phi p dng nhiu ln)
Li gii: Xt hm s: ( ) 3 5 f x asinx bsin x csin x = , ta c:
'( ) os 3 os3 5 os5 f x ac x bc x cc x = , ''( ) 9 3 25 5 f x asinx bsin x csin x = + + .
Ta c
1 2 3
3 3 3
(0) ( ) ( ) ( ) 0 (0; ), ( ; ), ( ; )
4 4 4 4 4 4
f f f f x x x

= = = = sao
cho
1 2 3 4 1 2 5 2 3 4 5
(0) '( ) ' ( ) '( ) 0 ( ; ), ( ; ) | ''( ) ''( ) 0 f f x f x f x x x x x x x f x f x = = = = = =
m ''(0) ''( ) 0 f f = = iu phi chng minh.
Bi ton 6. Cho a thc P(x) v Q(x) = aP(x) + bP(x) trong a, b l cc s thc, a
0. Chng minh rng nu Q(x) v nghim th P(x) v nghim.
Li gii: Ta c degP(x) = degQ(x)
V Q(x) v nghim nn degQ(x) chn. Gi s P(x) c nghim, v degP(x) chn nn
P(x) c t nht 2 nghim.
- Khi P(x) c nghim kp x = x
0
ta c x
0
cng l mt nghim ca P(x) suy ra Q(x) c
nghim.
- Khi P(x) c hai nghim phn bit x
1
< x
2
.
Nu b = 0 th hin nhin Q(x) c nghim.
Nu b 0 : Xt ( ) ( )
a
x
b
f x e P x = ta c: ( ) f x c hai nghim phn bit x
1
< x
2

a a a a
b b b b
1 1
'( ) ( ) '( ) ( ( ) '( )) ( )
x x x x
a
f x e P x e P x e aP x bP x e Q x
b b b
= + = + =
V ( ) f x c hai nghim suy ra '( ) f x c t nht 1 nghim hay Q(x) c nghim.
2.2. GII PHNG TRNH
Bi ton 7: Gii phng trnh: 3 5 2.4
x x x
+ =
(1)

Li gii:
Nhn xt: 0; 1 x x = =

l nghim ca phng trnh (1).
Gi x
0
l nghim ca phng trnh cho. Ta c:
0 0 0 0 0 0 0
3 5 2.4 5 4 4 3 (1a)
x x x x x x x
+ = =


=========================================================== 78
HI CC TRNG THTP CHUYN KHU VC DUYN HI V NG BNG BC B

Hi tho khoa hc mn Ton hc ln th III - 2010
Xt hm s
0 0
( ) ( 1)
x x
f t t t = + , ta c (1a) (4) (3) f f =

V f(t) lin tc trn [3; 4] v c o hm trong khong (3; 4), do theo nh l Rolle
tn ti c (3; 4) sao cho:
0 0
0 1 1
0
0
0
'( ) 0 [( 1) ]=0
1
x x
x
f c x c c
x

=
= +


Vy phng trnh (1) c hai nghim x = 0 v x = 1.
Bi ton 8: Gii phng trnh:
x
5 3 2x (2)
x
=

Li gii:
Nhn xt: 0; 1 x x = =

l nghim ca phng trnh (2).
Gi x
0
l nghim ca phng trnh cho, ta c:
0 0
x
0 0
5 5 3 3x (2a)
x
x =


Xt hm s:
0
0
( )
x
f t t tx = , khi : (2a) (5) (3) f f =

V ( ) f t

lin tc trn [3; 5] v c o hm trn (3; 5), do theo nh l Lagrange
lun tn ti c (3; 5)

sao cho:
0
0 1
0
0
0
'( ) 0 ( 1)=0
1
x
x
f c x c
x

=
=


Vy phng trnh (1) c hai nghim x = 0 v x = 1.
Bi ton 9. Gii phng trnh: 3 19 4 . 2 3 + = + x
x x
(3).
Li gii:
(5) 0 3 19 4 . 2 3 = + x
x x
.
Xt hm s: 3 19 4 . 2 3 ) ( + = = x x f y
x x
ta c: 19 4 ln 4 . 2 3 ln 3 ) ( ' + =
x x
x f
R x x f
x x
> + = , 0 ) 4 (ln 4 . 2 ) 3 (ln 3 ) ( ' '
2 2
hay ''( ) f x v nghim, suy ra '( ) f x c
nhiu nht 1 nghim, suy ra ( ) f x c nhiu nht 2 nghim.
M 0 ) 2 ( ) 0 ( = = f f do (3) c ng hai nghim 2 , 0 = = x x .
Bi ton 10. Gii phng trnh:
cos
(1 cos )(2 4 ) 3.4cos (4)
x
x x + + =
Li gii:
t cos , ( [-1;1]) t x t =

=========================================================== 79
HI CC TRNG THTP CHUYN KHU VC DUYN HI V NG BNG BC B

Hi tho khoa hc mn Ton hc ln th III - 2010

(3) (1 )(2 4 ) 3.4 (1 )(2 4 ) 3.4 0
t t t t
t t + + = + + =
Xt hm s: ( ) (1 )(2 4 ) 3.4
t t
f t t = + +
2
'( ) 2 4 ( - 2)4 ln4, ''( ) 2.4 ln4 ( - 2)4 ln 4
t t t t
f t t f t t = + + = +
Ta c:
2
''( ) 0 2
ln 4
f t t = = + ''( ) f t c mt nghim duy nht
'( ) f t c nhiu nht hai nghim ( ) f t c nhiu nht ba nghim.
Mt khc d thy
1
(0) ( ) (1) 0
2
f f f = = = , do ( ) f t c ba nghim
1
0, ,1
2
t = .
Kt lun: Nghim ca phng trnh (4) l:
2 , 2 , 2 ,
2 3
x k x k x k k Z

= + = + =

2.3. CHNG MINH BT NG THC
Bi ton 11. Cho hai s thc dng a, b tha mn a < b. Chng minh rng:
ln
b a b b a
b a a

< <
Li gii:
Xt hm s
1
( ) ln '( ) , (0; ). f x x f x x
x
= = +
Theo nh l Lagrange lun tn ti c (a; b) sao cho
( ) ( )
'( )
f b f a
f c
b a

hay
1 ln ln
ln
b a a b b
c b a c a

= =

m
1 1 1
0 a b c
b c a
< < < < < ln
b a b b a
b a a

< < .
Bi ton 12. Chng minh rng:
1
1 1
(1 ) (1 ) , (0; ).
1
x x
x
x x
+
+ < + +
+

Li gii:
Ta c:
1
1 1
(1 ) (1 ) [ln( 1) - ln ] ( 1)[ln( 2) - ln( 1)]
1
x x
x x x x x x
x x
+
+ < + + < + + +
+


=========================================================== 80
HI CC TRNG THTP CHUYN KHU VC DUYN HI V NG BNG BC B

Hi tho khoa hc mn Ton hc ln th III - 2010
t ( ) [ln( 1) - ln ] f x x x x = +

Ta c:
1
'( ) ln( 1) ln 1 ln( 1) ln
1 1
x
f x x x x x
x x
= + + = +
+ +

p dng nh l Lagrange i vi hm s: y = lnt trn [x; x+1], th tn ti
c (x; x+1) sao cho:
1
'( ) ln( 1) ln ln( 1) ln . f c x x x x
c
= + = +
M
1 1 1
0 1
1
x c x
x c x
< < < + > >
+

1 1 1
ln( 1) ln ln( 1) ln 0
1 1
x x x x
x x x
> + > + >
+ +

'( ) 0, (0;+ ) f x x > hm s ( ) f x ng bin trn (0;+ ).

T (1) suy ra: '( ) 0, (0; ) f x x > + ( ) f x

ng bin trn (0; ). +

Suy ra: ( 1) ( ), (0; ) f x f x x + > + iu phi chng minh.

Nhn xt: Trong v d trn thc cht ca vn l ta i chng minh hm s
1
( ) (1 )
x
F x
x
= +

ng bin trn (0; ) +

v ta i chng minh hm s ( ) ln ( ) f x F x =

ng
bin trn (0; ) + , n y bi ton tr v ging nh v d 1. Tng t ta chng minh c
hm s
1
1
( ) (1 )
x
G x
x
+
= + nghch bin trn (0; ). +

Ta c th chng minh bi ton 12 bng cch khc.

Xt hm s: ( ) ln(1 ) F x x = +
Vi mi cp s thc x, y bt k tha mn 0 < x < y, theo nh l Lagrange, lun tn
ti
0 0
(0; ), ( ; ) x x y x y tha mn:
0 0
( ) (0) ( ) ( )
'( ) , '( )
0
f x f f y f x
f x f y
x y x

= =


hay
0 0
1 ln(1 ) 1 ln(1 ) ln(1 )
;
1 1
x y x
x x y y x
+ + +
= =
+ +
.
M
0 0
1 1
1 1 x y
>
+ +
ln(1 ) ln(1 ) ln(1 )
ln(1 ) ln(1 ).
x y x
y x x y
x y x
+ + +
> + > +


Vy vi mi cp s thc x, y bt k tha mn 0 < x < y, lun c
ln(1 ) ln(1 ), y x x y + > + thay x bi
1
y
v y bi
1
x
ta c:

=========================================================== 81
HI CC TRNG THTP CHUYN KHU VC DUYN HI V NG BNG BC B

Hi tho khoa hc mn Ton hc ln th III - 2010
1 1 1 1 1 1
ln(1 ) ln(1 ) (1 ) (1 )
y x
x y y x y x
+ > + + > +
Bi ton 13. (Bt ng thc Jensen)
Cho hm s ( ) f x c o hm cp hai trn (a; b) v ''( ) 0, ( ; ) f x x a b > .
Chng minh rng:
1 2 1 2
1 2
( ) ( )
( ), , ( ; )
2 2
f x f x x x
f x x a b
+ +

Li gii:
ng thc xy ra khi
1 2
x x = .
Khi
1 2
x x < , theo nh l Lagrange, tn ti
1 2 1 2
1 2
( ; ), ( ; )
2 2
x x x x
c x d x
+ +
tha mn
1 2 1 2
1 2
2 1 2 1
( ) ( ) ( ) ( )
2 2
'( ) , '( )
2 2
x x x x
f f x f x f
f c f d
x x x x
+ +

= =

.
M ''( ) 0, ( ; ) '( ) f x x a b f x >

ng bin trn (a; b)
1 2 1 2 1 2 1 2
1 2
( ) ( )
'( ) ( ) ( ) ( ) ( ) ( ) ( ).
2 2 2 2
x x x x f x f x x x
f c f d f f x f x f f
+ + + +
< < >

Bi ton 14. (Bt ng thc Bernoulli)
Vi mi s thc x tha mn x > -1, chng minh rng (1 ) 1 .
n
x nx + +

Li gii:
- Khi x > 0: xt ( ) (1 )
n
f t t = + , theo nh l Lagrange ta c (0; ) a x

tha mn
1
( ) (0) '( ) (1 ) 1 (1 ) (1 ) 1
n n n
f x f xf a x nx a nx x nx

= + = + > + > +
- Khi -1< x < 0: xt ( ) (1 )
n
f t t = + , theo nh l Lagrange ta c ( ; 0) a x

tha mn
1
( ) (0) '( ) (1 ) 1 (1 ) (1 ) 1
n n n
f x f xf a x nx a nx x nx

= + = + > + > +
Vy (1 ) 1 , (-1; )
n
x nx x + + + . ng thc xy ra khi v ch khi x = 0.
Bi ton 15. Cho hm s ( ) f x c o hm cp hai trn R, ''( ) 0, f x x R

( ''( ) 0 f x = c s nghim m c). Chng minh rng:
*
1
( ) (0) '( ) ( 1) (1),
n
i
f n f f i f n f n N
=
< < +

.
Li gii:
V ''( ) 0, f x x R ( ''( ) 0 f x = c s nghim m c) '( ) f x ng bin trn
R.
Theo nh l Lagrange, lun tn ti ( ; 1)
i
x i i + sao cho:

=========================================================== 82
HI CC TRNG THTP CHUYN KHU VC DUYN HI V NG BNG BC B

Hi tho khoa hc mn Ton hc ln th III - 2010
'( ) ( 1) ( ),
i
f x f i f i i R = + .
V '( ) f x ng bin trn R '( ) '( ) '( 1)
i
f i f x f i < < +
'( ) ( 1) ( ) '( 1), f i f i f i f i i R < + < + .
*
1 1
'( ) [ ( 1) ( )] ( 1) (1),
n n
i i
f i f i f i f n f n N
= =
< + = +


v
*
1 1
'( ) [ ( ) ( 1)] ( ) (0),
n n
i i
f i f i f i f n f n N
= =
> =


Nhn xt: Nu ''( ) 0, f x x R th bt ng thc cn chng minh s i chiu.
Bi ton 16. Chng minh rng:
*
1
1
1 ln ln( 1),
n
i
n n n N
i
=
+ > > +

.
Li gii:
Xt
1
( ) ln '( ) f x x f x
x
= = v '( ) f x

nghch bin trn (0 : ) +
Tng t bi ton trn ta c:
*
1
( ) (1) '(1) '( ) ( 1) (1),
n
i
f n f f f i f n f n N
=
+ > > +


*
1
1
1 ln ln( 1),
n
i
n n n N
i
=
+ > > +


Bi ton 17: Cho s nguyn dng k, tm
5
2
5 4
1
1 1
5
k
i i =
(
(

(trong [x] l s nguyn ln


nht khng vt qu x).
Li gii:
Xt hm s
5
( ) 5 f x x = , ta c:
5 4
1
'( ) '( ) f x f x
x
= nghch bin trn (0; ) + .
Suy ra
5
2
5 5 5 5 5 5
5 4
1 1
1 1
( ) (0) '( ) ( 1) (1) 2 2 1 1 2 1
5
k
n
k k k
i i
f n f f i f n f
i = =
> > + > > + >


5
10
5 4
1
1 1
2 .
5
k
k
i i =
(
=
(


Nhn xt: T ba bi ton trn ta nhn thy nh gi tng
*
1
( ),
n
i
f i n N
=


( ( ) f x ng bin hoc nghch bin trn (0 : ) + ), chng ta phi xt hm s ( ) F x l mt
nguyn hm ca ( ) f x trn (0 : ) + v gii quyt tng t bi ton trn.

=========================================================== 83
HI CC TRNG THTP CHUYN KHU VC DUYN HI V NG BNG BC B

Hi tho khoa hc mn Ton hc ln th III - 2010
T vic c lng c tng
*
1
( ),
n
i
f i n N
=


ta c th ngh n bi ton tm gii hn
1
lim ( ) ( ),
n
i
g n f i
=


ta nghin cu cc bi ton sau.
Bi ton 18. Tnh
1
0
1 i
lim os
2n
n
i
c
n

.
Li gii:
Xt
2
( ) sin '( ) os '( ) 0, [ ; ]
2 2
n x x
f x f x c f x x n n
n n

= =
'( ) f x ng bin trn [ ; ] n n . Suy ra
1
( ) (0) '( ) ( 1) (1)
n
i
f n f f i f n f
=
< < +


1 1
0 0
2 i 2 2 1 i 2 1
os os sin os os sin
2n 2n 2n 2n 2n 2n
n n
i i
n n
c c c c
n n



= =
< < < <


M
2 2 1 2
lim( os ) lim( sin )
2n 2n
c
n


= =
1
0
1 i 2
lim os
2n
n
i
c
n

=
=

(Nguyn l kp).
Bi ton 19. Cho phng trnh:
1
1
.
n
i
n
i nx =
=
+


Chng minh rng: Vi mi s nguyn dng n phng trnh c duy nht mt
nghim dng. K hiu nghim l x
n
, tm limx
n
.
Li gii:
Xt
1
1
( )
n
i
f x n
i nx =
=
+


Ta c:
3
1
1
'( ) 0, (0; ) ( )
2 ( )
n
i
f x x f x
i nx =
= < +
+


lin tc, nghch bin trn
[0; ) + .
M
1
1
(0) 0, lim ( ) 0 ( ) 0
n
x
i
n
f n n f x n f x
i n
+
=
= > = = < =

c 1 nghim
dng duy nht.
Xt hm s ( ) 2
n n
F x x nx = + , ta c
1
'( ) '( )
n n
n
F x F x
x nx
=
+

nghch bin trn
(0; ) + .
1
( ) (0) '( ) ( 1) (1)
n
n n n n n
i
F n F F i F n F
=
> > +



=========================================================== 84
HI CC TRNG THTP CHUYN KHU VC DUYN HI V NG BNG BC B

Hi tho khoa hc mn Ton hc ln th III - 2010
1
1
2( ) 2( 1 1 )
n
n n n n
i
n
n nx nx n nx nx
i nx
=
+ > > + + +
+


2( ) 2( 1 1 )
n n n n
n nx nx n n nx nx + > > + + +
1 1 1
1 1
2
n n n n
x x x x
n n
+ > > + + +
1 1 2
1 2 1 1
n n n n n n
x x x x x x
n n n
+ + < < + + + + < + + +
2
2 1 2 lim( 1 ) 2
n n n n
x x x x
n
< + + < + + =
n
1 3 9
lim( 1 ) lim limx
2 4 16
n n n
x x x + = = = .
Bi ton 20: Cho cc s thc dng a, b, c, d tha mn
2 2 2 2 2
4 4 4 4 4
a b c d e
a b c d e
+ + = +

+ + = +


Chng minh rng
3 3 3 3 3
. a b c d e + + < +

Nhn xt: Trong bi ton ny t gi thit
2 2 2 2 2
4 4 4 4 4
a b c d e
a b c d e
+ + = +

+ + = +

, ta nhn thy ngay


gi thit ca nh l Rolle vi hm s ( ) ( (2) (4) 0)
x x x x x
f x a b c d e f f = + + = = , khi
ta phi chng minh (3) 0 f < . V ( ) f x

lin tc v (3) 0 f < , suy ra tn ti khong( ; ) 3 m n

sao cho ( ) 0, ( ; ) f x x m n < , do bi ton tr thnh xt du ca ( ) f x , v th ta cn kim
sot c cc nghim ca ( ) f x .

Li gii:
Khng mt tnh tng qut ta c th gi s 1, a b c d e =
Nu
2 2 2 2
1 1 ( 0) d d x x b a e x = + + = +
4 4 4 4 4 4 2 2 2 2 4
( ) 1 (1 ) a b c d e a e x a x e + + = + + + + = + +
2 2 2
2 2 2 2 2 2 2 2 2
2
0
( 1) ( ) 0 1 1
x e a b
e a x a a e e a e a
e a
e a
= = +

+ = = + = +



=
=


( Mu thun
) 1 d <
Tng t ta c 1 a b d e < <
Xt hm s ( ) 1 (2) (4) 0
x x x x
f x a b d e f f = + + = =


=========================================================== 85
HI CC TRNG THTP CHUYN KHU VC DUYN HI V NG BNG BC B

Hi tho khoa hc mn Ton hc ln th III - 2010
Gi s ( ) f x c nghim
0
2; 4. x

Theo nh l Rolle, tn ti
1 2
x x < tha mn:
1 2
'( ) '( ) 0 f x f x = =

hay
1 1 1 1
ln ln ln ln ,
x x x x
a a b b d d e e + = +

2 2 2 2
ln ln ln ln
x x x x
a a b b d d e e + = +
2 2 2 2
1 1 1 1
ln ln ln ln
ln ln ln ln
x x x x
x x x x
a a b b d d e e
a a b b d a e b
+ +
=
+ +

M
2 2 1 2 1 2
1 0 ln ln ln ln
x x x x x x
a b d e a a b b a b a b b

< < > + +



2 2
2 1
1 1
ln ln
ln ln
x x
x x
x x
a a b b
b
a a b b

+

+

v
2 2
2 2 2 2 1 1 2 1
1 1
ln ln
ln ln ln ln 0
ln ln
x x
x x x x x x x x
x x
d d e e
d d e e d d d e b d
d d e e

+
+ + <
+

2 2 2 2
1 1 1 1
ln ln ln ln
ln ln ln ln
x x x x
x x x x
a a b b d d e e
a a b b d a e b
+ +
<
+ +
(Mu thun).
Vy ( ) f x ch c hai ngim x = 2, x = 4 v '( ) f x

c 1 nghim duy nht, v n thuc
(2; 4). V ( ) f x

lin tc nn ( ) f x

mang cng mt du trn mi khong
( ; 2), (2; 4), (4; ). +

M (0) 1 0 ( ) 0, ( ; 0) ( ) 0, (2; 4) f f x x f x x = > > < (v
nu ( ) 0, (2; 4) f x x >

th x = 2 l nghim ca '( ) f x ) (3) 0 f < (iu phi chng
minh).
nh l Lagrange cn c s dng gii quyt mt s bi ton v bt ng thc
i xng, nhm mc ch lm gim s bin. Nu cn chng minh bt ng thc i xng n
bin
1 2
, ,...,
n
a a a th ta xt a thc
1 2
( ) ( )( )...( )
n
f x x a x a x a = , suy ra ( ) f x c n
nghim, do '( ) f x

c n 1 nghim
1 2 1
, ,...,
n
b b b

, v da vo nh l Vite ta a v
chng minh bt ng thc i xng vi n 1 bin
1 2 1
, ,...,
n
b b b

.
Bi ton 21. Cho a < b < c, chng minh rng:
2 2 2 2 2 2
3 3 3 a a b c a b c ab bc ca b a b c a b c ab bc ca c < + + + + < < + + + + + <

Li gii:
Xt hm s: ( ) ( )( )( ) ( ) ( ) ( ) 0 f x x a x b x c f a f b f c = = = =

Theo nh l Lagrange tn ti
1 2
a x b x c < < < < sao cho:
1
( ) ( ) ( ) '( ) f a f b a b f x = ,

=========================================================== 86
HI CC TRNG THTP CHUYN KHU VC DUYN HI V NG BNG BC B

Hi tho khoa hc mn Ton hc ln th III - 2010
1 1 2
( ) ( ) ( ) '( ) '( ) '( ) 0 f c f b c b f x f x f x = = =
2
'( ) 3 2( ) f x x a b c x ab bc ca = + + + + +
2 2 2
1
3
a b c a b c ab bc ca
x
+ + + +
=

2 2 2
2
3
a b c a b c ab bc ca
x
+ + + + +
=
Do , t
1 2
a x b x c < < < < . Suy ra:
2 2 2
3 3 a a b c a b c ab bc ca b < + + + + <
2 2 2
3 a b c a b c ab bc ca c < + + + + + <

Bi ton 22. Cho cc s thc khng m a, b, c, d. Chng minh rng:
3
4 6
abc bcd cda dab ab bc cd da ac db + + + + + + + +

Li gii:
Xt ( ) ( )( )( )( ) f x x a x b x c x d = .
t
, , , p a b c d q ab bc cd da ac bd r abc bcd cda dab s abcd = + + + = + + + + + = + + + =
4 3 2 3 2
( ) '( ) 4 3 2 f x x px qx rx s f x x px qx r = + + = +
Ta c ( ) ( ) ( ) ( ) 0 f a f b f c f d = = = = , theo nh l Rolle suy ra '( ) 0 f x =

c ba
nghim (nu a = b th a l nghim ca f(x)).
Suy ra tn ti , , w 0 u v tha mn '( ) 4( )( )( w) f x x u x v x =

3 2
4 4( ) 4( ) 4 x u v w x uv vw wu x uvw = + + + + +

=========================================================== 87
HI CC TRNG THTP CHUYN KHU VC DUYN HI V NG BNG BC B

Hi tho khoa hc mn Ton hc ln th III - 2010
3
4
1
2
1
4
u v w p
uv vw wu q
uvw r

+ + =

+ + =

.M
2 3 3
3
( )
3 3 6 4
uv vw wu uv vw wu q r
uvw uvw
+ + + +

3
4 6
abc bcd cda dab ab bc cd da ac db + + + + + + + +

ng thc xy ra w u v a b c d = = = = = .
2.4. TM GII HN DY S
nh l Lagrange c s dng gii quyt mt s bi ton v gii hn dy s, vi
cc dy s xc nh bi hm s ( ) f x

v dy s xc nh bi nghim ca mt phng trnh
( ) 0
n
f x = , ni chung ( ), f x

( )
n
f x

l cc hm s c o hm v n iu trn tp xc nh
ca chng, o hm ca chng c th c lng c bi mt bt ng thc. Do nu tm
c gii hn l a, ta c th so snh c hiu ( ) ( ),
n
f x f a

( ) ( )
n n n
f x f a vi
n
x a v c
th c lng c x
n
.
Bi ton 23. Cho dy s thc (x
n
) xc nh bi:
1
*
1
2
2007
3 ,
1
n
n
n
x
x
x n N
x
+
=

= +


Tm gii hn ca dy s khi n tin dn ti dng v cng.
Li gii: Ta c
*
3, .
n
x n N >
Xt f(x) =
1
3
2

+
x
x
, ta c:
2 3
1
'( )
( 1)
f x
x
=

1
'( ) , ( 3; )
2 2
f x x < + .

=========================================================== 88
HI CC TRNG THTP CHUYN KHU VC DUYN HI V NG BNG BC B

Hi tho khoa hc mn Ton hc ln th III - 2010
Nu (x
n
) c gii hn th gii hn l nghim ln hn 3 ca phng trnh
( ) f x x = . Ta c:
2
( ) 3
1
x
f x x x
x
= = +

2
2
2
( 3)
1
x
x
x
=



2 2 2
( 3 ) 2( 3 ) 3 0 x x x x =
2
2
3 1
3 3
x x
x x

=

=


3 15
.
2
x
+
=
t
3 15
2
a
+
= , theo nh l Lagrange, lun tn ti ( ; )
n n
c x a

hoc ( ; )
n
a x tha
mn: ( ) ( ) '( ) .
n n n
f x f a f c x a =
1 1
1 1
( ) ( ) '( ) ... ( )
2 2 2 2
n
n n n n n
x a f x f a f c x a x a x a
+
= = < < <
M
1
1
lim( ) 0
2 2
n
x a = , do limx
n
= a =
2
15 3 +
.
Nhn xt:
Trong bi ton trn vic gii phng trnh ( ) f x x = khng nht thit phi trnh by,
ta ch cn chn c nghim tha mn ca n l c.
Bi ton trn c dng tng qut:
Cho dy s thc (x
n
) xc nh bi:
1
*
1
( ),
n n
x a
x f x n N
+
=

. Chng minh rng:


a) Nu ( ) f x

l hm s c o hm trn khong D cha a v '( ) 1, f x b x D < < th
(x
n
) c gii hn hu hn khi n tin dn n dng v cng.
b) Nu ( ) f x

l hm s c o hm trn khong D cha a, ( ) 0 f a v
'( ) 1, f x b x D > > th |x
n
| tin dn n dng v cng khi n tin dn n dng v cng.
Phng php gii

=========================================================== 89
HI CC TRNG THTP CHUYN KHU VC DUYN HI V NG BNG BC B

Hi tho khoa hc mn Ton hc ln th III - 2010
a) - Nu phng trnh ( ) f x x = gii c (tm c nghim) th ta gii quyt bi
ton tng qut tng t bi ton trn v khi ta tm c gii hn ca dy s khi n tin dn
ti dng v cng.
- Nu phng trnh ( ) f x x = kh gii th ta gii quyt bi ton tng qut bng cch
s dng tiu chun Cauchy. Bi ton sau y l mt v d c th.
b) Tng t a.
- Khi
0 0 0 0
: , ( ) a D a a f a a =

lun tn ti
0
( ; )
n n
c x a

hoc
0
( ; )
n
a x tha mn:
0 0
( ) ( ) '( )
n n n
f x f a f c x a =
1 1 0 0 0
( ) ( ) ... lim
n
n n n n n
x a x a f x f a b x a b a a x
+ +
+ = > > > = +

- Khi phng trnh f(x)=x v nghim, ta c f(x)-x > 0 xD hoc f(x)-x < 0 xD suy
ra x
n
tng hoc gim. Nu x
n
c gii hn th gii hn l nghim ca phng trnh f(x) = x,
do
lim
n
x = +
Bi ton 24. (D b VMO 2008)
Cho s thc a v dy s thc (x
n
) xc nh bi:
x
1
= a v x
n+1
= ln(3+cosx
n
+ sinx
n
) 2008,
*
. n N
Chng minh rng dy s (x
n
) c gii hn hu hn khi n tin dn n dng v cng.
Li gii:
t f(x) = ln(3+sinx+cosx) 2008, ta c:
cos sin
'( ) , R
3 sin cos
x x
f x x
x x

=
+ +
.
M 2 | cos sin | , 2 | sin cos | + x x x x , suy ra: . 1
2 3
2
| ) ( ' | < =

q x f
Theo nh l Lagrange : vi mi cp hai s thc x, y (x < y), lun tn ti ( ; ) z x y

tha mn: f(x) f(y) = f(z)(x-y).
T suy ra |f(x) f(y)| q|x y| vi mi x, y thuc R.
p dng tnh cht trn vi m > n N, ta c :

=========================================================== 90
HI CC TRNG THTP CHUYN KHU VC DUYN HI V NG BNG BC B

Hi tho khoa hc mn Ton hc ln th III - 2010
|x
m
x
n
| = |f(x
m-1
) f(x
n-1
)| q|x
m-1
- x
n-1
| q
n-1
|x
m-n+1
x
1
| q
N-1
|x
m-n+1
x
1
|.
Mt khc dy (x
n
) b chn v q < 1 nn vi mi > 0 tn ti N ln sao cho:
q
N-1
|x
m-n+1
x
1
| < .
Nh vy dy (x
n
) tho mn tiu chun Cauchy, do (x
n
) hi t.
Bi ton 25. (VMO 2007)
Cho s thc a > 2 v
10 10 1
( ) ... 1
n n n
n
f x a x x x x
+
= + + + + + .
a) Chng minh rng vi mi s nguyn dng n, phng trnh ( )
n
f x a = lun c
ng mt nghim dng

duy nht. K hiu nghim l x
n
.
b) Chng minh rng dy (x
n
) c gii hn bng
1 a
a

khi n dn n v cng.
Li gii:
t ( ) ( )
n n
F x f x a = , ta c ( )
n
F x lin tc, ng bin trn [0; ) + v
10
(0) 1 0, (1) 1 0.
n n
F a F a n a = < = + + >

Suy ra phng trnh ( )
n
f x a = lun c ng
mt nghim x
n
dng

duy nht.
t
9 *
1
( ) ( 1)[( 1) 1] ( ) ,
n
n n n
a
b f b b a a a f b a b x n N
a

= = + > >
Theo nh l Lagrange, lun tn ti ( ; )
n n
c x b tha mn:
( ) ( ) '( )( )
n n n n n
f b f x f c b x = .
M '( ) 1
n
f c > nn
9
n
( ) ( ) ( 1)[( 1) 1] limx
n
n n n n
b x f b f x b a a b < = =
9
n
( 1)[( 1) 1] limx
n
n
b b a a x b b < < = (v (0;1) b ).
Nhn xt:
Bi ton trn s kh khn hn nhiu nu bi khng cho trc gii hn ca dy s.
Khi cu hi t ra l gii hn bng bao nhiu?
Ta c th tr li cu hi nh sau:

=========================================================== 91
HI CC TRNG THTP CHUYN KHU VC DUYN HI V NG BNG BC B

Hi tho khoa hc mn Ton hc ln th III - 2010
Trc ht gii hn ca dy s phi thuc khong (0; 1), gi s gii hn ca dy s l
b ta c:
10 10
1 1 1
( ) ( 1) lim ( )
1 1 1
n
n n
f b b a b f b
b b b
= + + =

(v (0;1) b ).
M
1 1
( )
1
n n
a
f x a a b
b a

= = =

.
Trong bi ton dng trn dy s xc nh l dy nghim thuc (a; b) ca phng
trnh ( ) 0
n
f x = , vi gi thit ( )
n
f x l hm s ng bin hoc nghch bin trn (a;
b), '( )
n
f x c <

vi mi s nguyn dng n v s thc dng x thuc (a; b), khi gii bi ton
dng ny ni chung ta iu kh khn nht l xc nh c gii hn ca dy s.
Bi ton 26: (VMO 2002)
Xt phng trnh
2
1
1 1
1 2
n
i
i x
=
=

, vi n l s nguyn dng.
a) Chng minh rng vi mi s nguyn dng n, phng trnh nu trn c mt nghim
duy nht ln hn 1; k hiu nghim l x
n
.
b) Chng minh rng lim 4
n
x = .
Li gii:
a) Xt
2
1
1 1
( )
1 2
n
n
i
f x
i x
=
=

, ta c: ( )
n
f x lin tc v nghch bin trn (1; ). +
M
1
1
lim ( ) , lim ( )
2
n n
x x
f x f x
+
+
= + = ( ) 0
n
f x = c mt nghim duy nht ln hn 1.
b) Vi mi s nguyn dng n ta c:
1
(4) 0 (4) ( ) 4
2(2 1)
n n n n n
f f f x x
n
= < < <
+

Theo nh l Lagrange, lun tn ti ( ; 4)
n n
c x tha mn:
(4) ( ) '( )(4 )
n n n n n
f f x f c x = .
M
1 9
'( ) 4 9( (4) ( )) 4
9 2(2 1)
n n n n n n
f c x f f x x
n
< < <
+


=========================================================== 92
HI CC TRNG THTP CHUYN KHU VC DUYN HI V NG BNG BC B

Hi tho khoa hc mn Ton hc ln th III - 2010
9
4 4 lim 4.
2(2 1)
n n
x x
n
< < =
+

3. BI TP T GII
1. Gii cc phng trnh sau.
a) x x x 3 ) 1 2 ( log ) 1 3 ( log
3 2
= + + +
b) 2008 2010 2.2009
x x x
+ =
c) (4 2)(2 ) 6
x
x + =
2. Chng minh nu hm s ( ) f x

c o hm cp 2 trn on [a; b] v
'( ) '( ) f a f b =

th bt phng trnh
4
4
''( ) ( ) '( )
( )
f x f a f b
a b

c t nht mt nghim.
3. Tm
1
1 1
2
1 sin
2
n
i
Lim
i
n
n

=
| |
|
|
|
+
\


4. Cho dy s thc (x
n
) xc nh bi:
1
2
1
ln 1 2010, 1
n n
x a
x x n
+
=

= +

.
Chng minh rng x
n
c gii hn.
5. Cho phng trnh:
1
1
1.
n
i
i nx
=
=
+


Chng minh rng: Vi mi s nguyn dng n phng trnh c duy nht mt
nghim dng. K hiu nghim l x
n
, tm limx
n
.
6. Chng minh 1
b a
a b + > vi mi , 0 a b > .
7. Cho a thc P(x) v Q(x) = aP(x) + bP(x) + cP(x) trong a, b, c l cc s
thc tha mn a 0 v b
2
4ac > 0. Chng minh rng nu Q(x) v nghim th P(x) v
nghim.
8. Cho s thc a khc khng, a thc ( ), deg ( ) 1 P x P x n = v a
thc
2 ( )
( ) ( ) '( ) "( ) ... ( )
n n
Q x P x aP x a P x a P x = + + + + . Chng minh rng nu ( ) P x v nghim
th ( ) Q x cng v nghim.









=========================================================== 93
HI CC TRNG THTP CHUYN KHU VC DUYN HI V NG BNG BC B

Hi tho khoa hc mn Ton hc ln th III - 2010
T S KP V PHP CHIU XUYN TM

Trng THPT chuyn Thi Bnh Thi Bnh

Li ni u:
Php chiu xuyn tm v t s kp l mt phn rt p ca hnh hc.Ti liu nh ny xin
a ra mt s v d s dng php chiu xuyn tm v t s kp trong gii ton hnh hc.

Phn 1: S lc v l thuyt:

nh l 1. Cho t gic ABCD vi cc im cho S, O, K. Gi s SO ct AD ti M, ct
BC ti N. Khi .


nh l 2. Cho hai hng : Khi
song song hoc ng quy.



nh l 3. Cho hai chm v :
Gi s Khi thng hng.

=========================================================== 94
HI CC TRNG THTP CHUYN KHU VC DUYN HI V NG BNG BC B

Hi tho khoa hc mn Ton hc ln th III - 2010


nh l 4. Cho chm bn ng thng Khi

H qu. Cho t gic ABCD ni tip ng trn (O). Khi , vi mi im M trn (O), t
s khng i.

nh ngha 1. Php chiu xuyn tm.
Cho (d). S ngoi (d). Vi mi im M, SM ct (d) ti M(M khng thuc ng thng
qua S song song (d)). Vy MM l php chiu xuyn tm vi tm chiu S ln (d)
Tip theo ta s pht biu mt nh l quan trng v php chiu xuyn tm

nh l 5. Php chiu xuyn tm bo ton t s kp
Phn 2:Cc v d:

1.Cho hai tam gic ABC v c
Chng minh rng thng hng khi v ch khi ng quy hoc song song.

=========================================================== 95
HI CC TRNG THTP CHUYN KHU VC DUYN HI V NG BNG BC B

Hi tho khoa hc mn Ton hc ln th III - 2010

Gii. Gi s ct ti . Ta c

Do nn thng
hng.
2.Cho t gic ABCD, O l giao im ca hai ng
cho. ng thng d i qua O ct cc ng thng AB,
BC, CD, DA ti P, N, Q, M. Chng minh rng
(MNPO)=(MNOQ)


Gii.

Ln lt chiu tm A v D ln BC.
Ch : C th yu cu chng minh O l trung im ca MN khi v ch khi O l
trung im ca PQ.

3.Cho tam gic ABC v cc im M, N trn cnh BC sao cho M nm gia B v N. Gi
l tm cc ng trn ni tip v bng tip gc A ca tam gic ABM; l tm
ng trn bng tip v ni tip gc A ca tam gic CAN. Chng minh rng v
ng quy.
S
M
Q
N
P
O
D
C
B
A

=========================================================== 96
HI CC TRNG THTP CHUYN KHU VC DUYN HI V NG BNG BC B

Hi tho khoa hc mn Ton hc ln th III - 2010

Gii. Ch rng . Gi . S dng php
chiu tm O ta suy ra
4.(nh l Papuyt) Cho hai ng thng v . Trn cho ba im , trn
cho ba im . Gi s
Chng minh rng thng hng.
Gii. Ln lt chiu xuyn tm v ln , ta c T
suy ra ng quy v ta c iu phi chng minh.

5.Cho t gic ABCD, Trn cnh AB ly im P, trn cnh
BC ly im M. Gi s AM ct CD ti N, CP ct AD ti Q, MP ct QN ti E. Chng minh
rng S, K, E thng hng.

=========================================================== 97
HI CC TRNG THTP CHUYN KHU VC DUYN HI V NG BNG BC B

Hi tho khoa hc mn Ton hc ln th III - 2010


Gii. S dng php chiu xuyn tm C, ta c . Suy ra

Do , , , MA NA MB NS S MK ND K MP NQ E = = = nn thng hng.
Li gii 2. Chiu xuyn tm v ta c


Suy ra ng qui.

=========================================================== 98
HI CC TRNG THTP CHUYN KHU VC DUYN HI V NG BNG BC B

Hi tho khoa hc mn Ton hc ln th III - 2010
6.Cho t gic ABCD, O l giao ca hai ng cho. Mt ng thng d i qua O ct AD,
BC, AB, CD ti M, N, P, Q. Gi s v
. Chng minh rng X, Y, Z, T thng hng.


Cch gii 1. Gi Ta chng minh Gi giao im
ca XO vi AD, BC l R v G ; giao im ca KO vi AD v BC l E v F. Ta c
Do nn thng hng.
Cch gii 2. S dng php chiu tm A ln CD v php chiu tm B ln AD ta suy ra


=========================================================== 99
HI CC TRNG THTP CHUYN KHU VC DUYN HI V NG BNG BC B

Hi tho khoa hc mn Ton hc ln th III - 2010

Suy ra EA, CM, KS ng quy. Vy
Cch gii 3. S dng t s kp ca chm
Ta c : . Suy ra K, S, X thng
hng.
7.Cho tam gic v im nm trong tam gic. Gi s

ng thng i qua , song song vi ct ti Chng minh rng
l trung im ca

Gii. Ta c Do
8.Cho tam gic ABC, ng cao AH. Trn AH ly im M, BM ct AC ti K, CM ct AB
ti E. Chng minh rng AH l phn gic ca gc

=========================================================== 100
HI CC TRNG THTP CHUYN KHU VC DUYN HI V NG BNG BC B

Hi tho khoa hc mn Ton hc ln th III - 2010

Gii. p dng tnh cht ta c . T suy ra iu phi chng
minh.
9. (Iran) Cho tam gic v im nm trong tam gic. Gi s

K Chng minh rng l phn gic ca gc

Gii. Ta c Do ta c pcm.
10. Cho tam gic v im nm trong tam gic. Gi s

K ng thng i qua , song song vi ct ti .
Chng minh rng l phn gic ca gc


=========================================================== 101
HI CC TRNG THTP CHUYN KHU VC DUYN HI V NG BNG BC B

Hi tho khoa hc mn Ton hc ln th III - 2010

11. Cho tam gic , cc ng cao . Gi l giao im ca vi ,
l trung im ca . ng thng i qua song song vi ct ti .
Chng minh rng bn im cng thuc mt ng trn.

Gii. Ch rng . S dng h thc Macloranh ta c iu phi chng
minh.
12. Cho tam gic trc tm . ng thng i qua ct ti
; ng thng i qua , vung gc vi ct ti . Cc
im chia theo cng t s . Chng minh rng thng
hng.

=========================================================== 102
HI CC TRNG THTP CHUYN KHU VC DUYN HI V NG BNG BC B

Hi tho khoa hc mn Ton hc ln th III - 2010
Y
X
C
2
B
2
A
2
C
1
B
1
A
1
H
C
B
A

Gii. chng minh thng hng ta da vo nhn xt sau :
C
0
B
0
A
0
C
2
B
2
A
2
C
1
B
1
A
1

Cho hai hng v tha mn : Trn
ly , trn ly , trn ly sao cho

Khi , thng hng.
Tr li bi ton, ta chng minh Tht vy, ta c

trong song song vi
Ta chng minh

Qua , k cc ng thng song song vi . Ta c :

iu ny ng, do hai chm trn c cc ng tng ng vung gc.

13.(Bi ton con bm) Cho t gic ABCD ni tip ng trn (O), .
Mt ng thng i qua K ct cc cnh AB, CD ti M v N, ct ng trn (O) ti P v Q.
Chng minh rng K l trung im ca PQ khi v ch khi K l trung im ca MN.

=========================================================== 103
HI CC TRNG THTP CHUYN KHU VC DUYN HI V NG BNG BC B

Hi tho khoa hc mn Ton hc ln th III - 2010
O
K
Q P
N M
D
C
B
A
Gii. Ta c
. T c pcm.
14.(nh l Pascal). Ta c
. T suy ra iu phi chng
minh.
O
F
E
Z
Y
X
B'
A'
C'
C
B
A

Ch . Xt cc trng hp c bit (cho cc nh trng nhau) ta c cc kt qu
th v.

15.Cho tam gic SAB v im O nm trong tam gic. Cc ng thng BO, AO ct SA,
SB ti M v N. Mt ng thng qua O ct cc on MA, NB ti P v Q. Chng minh rng


Gii. iu phi chng minh tng ng vi

Xt php chiu tm O ta c Do

Suy ra hoc .
Gi s . Khi , (pcm).
16. (China TST 2002). Cho t gic li ABCD. Cho
E AB CD, F AD BC, P AC BD = = = . O l chn ng vung gc h t P xung EF.
Chng minh rng AOD=BOC.
Bg: BD ct ti T .(ETFS)=-1.Php chiu tm B ln SC suy ra (APCS)=-1.
Php chiu xuyn tm E ln BD,(BPDT)=-1.
O(BPDT)=-1 m OP vung gc vi OT suy ra OP l phn gic gc BOD.
O(APCS)=-1 m OP vung gc vi OS suy ra OP l phn gic gc AOC.C PCM.

Q
P
N
M
O
B A
S

=========================================================== 104
HI CC TRNG THTP CHUYN KHU VC DUYN HI V NG BNG BC B

Hi tho khoa hc mn Ton hc ln th III - 2010
17.(Bucharest 2006) Cho tam gic ABC cn ti A. M l trung im BC. Tm qu tch
cc im P nm trong tam gic tha mn gc BPM v CPA b nhau.
Bg:
AP ct ng trn ngoi tip tam gic BPC v BC ln lt ti D v S.Gi thit suy ra
gc BPS=CPM.p dng nh l Steiner cho hai ng ng gic PS v PM ca tam gic
BPC suy ra SB/SC=PB
2
/PC
2
.Li c SB/SC=DB/DC. PB/PC.Suy ra PBDC l t gic iu
ha.Dn n tip tuyn ti B v Cv PD ng quy ti A.
Nu A trng Ath P thuc cung BIC (I l tm ni tip tam gic ABC)
Nu A khc A th P thuc ng thng AM.
18.Cho t gic ABCD. E AB CD, F AD BC, G AC BD = = = .
EF AD, AB M, N = . Chng minh rng (EMGN) 1 = .
Chng minh.

Xt php cc php chiu:
A: E B, G C, M F, N N ( ) ( ) EGMN BCFN =
D: E C, G B, M F, N N (EGMN) (CBFN) =
( ) BCFN (CBFN)
1
(BCFN)
(BCFN)
=
=

(BCFN) 1 = (do (BCFN) 1 )
Vy ( ) EGMN 1 = (d.p.c.m)
Kt lun :Qua mt s v d trn phn no cho thy v p ca php chiu xuyn tm v
t s kp trong gii ton hnh hc.Ti liu cn s si ,chng ti knh mong nhn c s th
tt v mong nhn c gp ca qu ng nghip tp ti liu ny phong ph hn.
Ti liu tham kho:
1) Harmonic_division Virgil Nicula.
2) T s kp.Phm Minh Phng
3) Ti liu gio khoa chuyn ton 10 Hnh hc.Nguyn Minh H.
4) Hng im iu ha.Kim Lun.Nguyn nh Thnh Cng.L Nam Trng.







=========================================================== 105
HI CC TRNG THTP CHUYN KHU VC DUYN HI V NG BNG BC B

Hi tho khoa hc mn Ton hc ln th III - 2010
MT S DNG TON V DY S V GII HN

Trn Ngc Thng - THPT Chuyn Vnh Phc

1 Gii hn dy s

1.1 Dy s

nh ngha 1.1. Dy s (thc) l mt hm s xc nh trn tp con ca tp s t nhin

Vi M , thay cho k hiu
: u M
( ) n u n a
ta thng dng k hiu ( ) ,{ } , ( )
n n n n n n
u u u
M M
hay { }
n n
u

nh ngha 1.2. Cho dy ( )
n n
u


Dy ( )
n
u c gi l dy (n iu) tng nu
1 n n
u u n
+

Dy ( )
n
u c gi l dy (n iu) gim nu
1 n n
u u n
+

Dy ( )
n
u c gi l dy (n iu) tng nghim ngt nu
1 n n
u u n
+
<
Dy ( )
n
u c gi l dy (n iu) gim nghim ngt nu
1 n n
u u n
+
>
Nhn xt.
Nu ( ) , ( )
n n
x y th ( )
n n
x y +
Nu ( ) , ( )
n n
x y th ( )
n n
x y +
Nu ( )
n
x th ( )
n
x . V nu ( )
n
x th ( )
n
x
Nu hai dy dng ( ), ( )
n n
x y cng tng (gim) th ( )
n n
x y tng (gim).
Mt dy c th khng tng, cng khng gim. V d ( 1)
n
n
x n =
nh ngha 1.3. Cho dy s ( )
n n
x

.
Dy ( )
n
x c gi l b chn trn, nu tn ti hng s M sao cho
n
x M n
Dy ( )
n
x c gi l b chn di, nu tn ti hng s m sao cho
n
x m n
Dy ( )
n
x va b chn trn, va b chn di c gi l b chn.
nh l 1.1. Dy ( )
n
x b chn khi v ch khi tn ti ghng s 0 c sao cho | |
n
u c n

1.2 Gii hn ca dy s
nh ngha 1.4. Dy s ( )
n
u c gi l hi t v , k hiu lim
n
n
u

= , nu vi mi
0 > cho trc ty , tm c ch s
0
n sao cho vi mi
0
n n u c | |
n
u <
V d 1.1. Chng minh rng

1. lim
n
c c

=
2.
1
lim 0
n
n

=
3.
1
lim 1
n
n
n

+
=
nh l 1.2. (Tnh duy nht ca gii hn) Gii hn ca mt dy hi t l duy nht

=========================================================== 106
HI CC TRNG THTP CHUYN KHU VC DUYN HI V NG BNG BC B

Hi tho khoa hc mn Ton hc ln th III - 2010

nh l 1.3. (Tnh th t ca dy hi t) Cholim
n
n
x

= l v a . Khi
Nu a < l th
0 0
( : )
n
n n n a x <
Nu a > l th
0 0
( : )
n
n n n a x >
nh l 1.4. (Chuyn qua gii hn trong bt ng thc) Cho lim
n
n
x

= l va .
Khi
Nu
0 0
( : )
n
n n n x a th a l
Nu
0 0
( : )
n
n n n x a th a l
nh l 1.5. (nh l gii hn kp gia) Cho ba dy s ( ), ( ), ( )
n n n
x y z tha mn

0 0
:
n n n
n n n z x y
cc dy ( ), ( )
n n
y z cng hi t n l
Khi dy ( )
n
x hi t v lim
n
n
x

= l
nh l 1.6. (Tnh cht i s ca dy hi t) Cho hai dy hi t
( ), ( )
n n
x y v lim ; lim
n n
n n
x a y b

= = . Khi
Dy ( )
n
x hi t v lim( )
n
n
x a

=
Dy (| |)
n
x hi t v lim| | | |
n
n
x a

=
Dy ( )
n n
x y + hi t v lim( )
n n
n
x y a b

+ = +
Dy ( )
n n
x y hi t v lim( )
n n
n
x y a b

=
Dy ( )
n
kx hi t v lim( )
n
n
kx ka

=
Dy ( )
n n
x y hi t v lim( )
n n
n
x y ab

=
Vi 0 b th dy
1
n
y
| |
|
\
c xc nh t mt ch s no , hi t v
1 1
lim
n
n
y b

| |
|

\

Vi 0 b th dy
n
n
x
y
| |
|
\
c xc nh t mt ch s no , hi t v
lim
n
n
n
x a
y b

| |
|
=
\

V d 1.2. Tm cc gii hn sau

2
2
3 2
lim
3 2
n
n n
n n

+
+


2
3 2
2 3 1
lim
3 4 5
n
n n
n n

+
+


2 1 2
lim
n
n n
n
+
+


2 2
lim( 2 1 1)
n
n n n n
+
+ + +

=========================================================== 107
HI CC TRNG THTP CHUYN KHU VC DUYN HI V NG BNG BC B

Hi tho khoa hc mn Ton hc ln th III - 2010

0
0
(3 1)
lim
(2 3)
n
k
n
n
k
k
k
=
+
=
+
+



1.3 Du hiu hi t ca dy s

1.3.1 Tiu chun Weiersstrass

nh l 1.7. Mt dy s n iu v b chn th hi t

C th, mt dy n iu tng v b chn trn th hi t, mt dy n iu gim v b
chn di th hi t.

V d 1.3. Cho cc dy s ( ), ( )
n n
x y c xc nh nh sau
1 1 1 1
0, 0, , , 1.
2
n n
n n n n
x y
x a y b x x y y n
+ +
+
= > = > = =
Chng minh rng cc dy s ( ), ( )
n n
x y hi t v lim lim
n n
x y = .

Li gii. Ta xt hai trng hp sau:
(i) Nu a b th bng quy np ta ch ra c dy ( )
n
x l dy gim b chn di bi a ,
cn dy ( )
n
y l dy tng b chn trn bi a . Do theo nh l 1.7 tn ti lim , lim
n n
x y v
t gi thit chuyn qua gii hn ta clim lim
n n
x y = .
(ii) Nu a b tng t nh trng hp (i).

V d 1.4. Cho dy s ( )
n
x c xc nh nh sau
1 2 2 1
1, 2, , 1
n n n
x x x x x n
+ +
= = = + .
Chng minh rng dy s cho c gii hn v tm gii hn .

Li gii. D thy bng quy np ta ch ra c ( )
n
x l dy s tng v b chn trn bi 4.
Do theo nh l 1.7 ta c tn ti lim
n
x a = . T ng thc
2 1 n n n
x x x
+ +
= + chuyn qua
gii hn ta c 2 a a = nhng do 0 a > nn ch ly 4 a = . Vylim 4
n
a = .

Bi tp tng t

Bi tp 1.5. Cho dy s ( )
n
x xc nh bi
1 1
2, 2 , 1, 2,
n n
x x x n
+
= = + = Chng
minh rng dy s cho hi t v tm lim
n
n
x

.
Bi tp 1.6. Cho dy s tha mn iu kin
( )
1
1
0 1, 1
4
n n n
x x x
+
< < > .
Chng minh dy s trn hi t v tm gii hn .
Bi tp 1.7. (nh l Cantor) Cho hai dy s thc ( ), ( )
n n
a b tha mn cc iu kin sau:
[ ] [ ]
1 1
; , ,
n n n n n n
a b a b a b
+ +
vi mi n v ( ) lim 0
n n
b a = .

=========================================================== 108
HI CC TRNG THTP CHUYN KHU VC DUYN HI V NG BNG BC B

Hi tho khoa hc mn Ton hc ln th III - 2010
Khi tn ti s thc c sao cho
[ ] { }
0
,
n n
n
a b c

=
=
I
v lim lim
n n
a b c = = .
Bi tp 1.8. (VMO 2005). Cho dy s thc ( ), 1, 2, 3...
n
x n = c xc nh bi:
1
x a = v
3 2
1
3 7 5
n n n n
x x x x
+
= + vi 1, 2, 3,... n =
trong a l mt s thc thuc on
4
0,
3
(
(

.
Chng minh rng dy s ( )
n
x c gii hn hu hn v tm gii hn .
Bi tp 1.9. (VMO 2002B). Xt phng trnh
2 2
1 1 1 1 1
... ... 0
2 1 4 x x x x k x n
+ + + + + + =

,
trong n l tham s nguyn dng.
1. Chng minh rng vi mi s nguyn dng n , phng trnh nu trn c duy nht
nghim trong khong ( ) 0,1 ; k hiu nghim l
n
x .
2. Chng minh rng dy s
n
x c gii hn hu hn khi n +.
Bi tp 1.10. Cho s thc a . Cho dy s ( ),
n
x n , c xc nh bi:
0
x a = v
1
sin
n n n
x x x
+
= + vi mi n .
Chng minh rng dy s ( )
n
x c gii hn hu hn khi n + v tnh gii hn .

1.3.2 Tiu chun Cauchy

nh ngha 1.5. Dy ( )
n
x c gi l dy Cauchy nu tha mn iu kin
0, : , , , ,
m n
N m n m n N x x > <
nh l 1.8. Dy s ( )
n
x hi t khi v ch khi ( )
n
x l dy Cauchy.

V d 1.11. Cho hm s : f tha mn iu kin
( ) ( ) f x f y q x y , vi mi , x y ,
trong ( ) 0,1 q l hng s cho trc. Vi c cho trc v xc nh dy
( ), 0,1, 2, 3...
n
x n = nh sau:
0 1
, ( ), 0,1, 2,...
n n
x c x f x n
+
= = = . Chng minh rng dy s ( )
n
x
hi t v gii hn ca dy s l nghim ca phng trnh ( ) f x x = .

Li gii Trc ht ta chng minh dy ( )
n
x l mt dy Cauchy. Tht vy, vi
, , m n n m > ta c:
( ) ( )
1 1 1 1 0
...
m
n m n m n m n m
x x f x f x q x x q x x

=
(1)

Mt khc ta c
( )
1
0 1 1 0 1 0 1 0
1
... ... 1
1
n
n
n n n
q
x x x x x x q x x x x
q

+ + + + =

.
T y suy ra
0 n
x x b chn vi mi n . Kt hp vi (1) ta thu c vi mi 0 >
tn ti N sao cho vi mi , m n N th
n m
x x < . Nn dy ( )
n
x l mt dy Cauchy
suy ra n hi t.

=========================================================== 109
HI CC TRNG THTP CHUYN KHU VC DUYN HI V NG BNG BC B

Hi tho khoa hc mn Ton hc ln th III - 2010
T iu kin ca hm f d dng chng minh c f lin tc v do t ng thc
1
( )
n n
x f x

= chuyn qua gii hn ta c gii hn ca dy ( )


n
x l nghim ca phng trnh
( ) f x x = .

Bi tp tng t

Bi tp 1.12. Cho : f tha mn iu kin vi mi 0 > u tn ti 0 > sao
cho: nu x y < + th ( ) ( ) f x f y < . Xt dy s xc nh nh sau:
0 1
, ( ), 0,1,...
n n
x x f x n
+
= = Chng minh rng dy ( )
n
x hi t.
Bi tp 1.13. Cho : f tha mn iu kin ( ) ( ) ( ( )) x f x x f x , trong
: l hm lin tc v b chn di. Ly
0
x v lp dy
1
( ), 0,1, 2,...
n n
x f x n
+
= =
Chng minh rng dy s ( )
n
x hi t.
Bi tp 1.14. Cho : f tha mn iu kin
( )
( ) ( ) ( ) ( ) f x f y k x f x y f y + , vi mi , x y , trong
1
2
k < . Xt dy s xc
nh nh sau:
1 1
, ( ), 1
n n
x x f x n
+
= . Chng minh rng dy ( )
n
x hi t v gii hn ca
dy l nghim duy nht ca phng trnh ( ) f x x = .
Bi tp 1.15. Cho : f tha mn iu kin: c s , 0 1 k k < sao cho
{ }
( ) ( ) max , ( ) , ( ) , . f x f y k x y f x x y f y x y
Xt dy s xc nh nh sau:
1 1
, ( ), 1
n n
x x f x n
+
= . Chng minh rng dy ( )
n
x hi
t v gii hn ca dy l nghim duy nht ca phng trnh ( ) f x x = .

1.3.3 Nguyn l kp

nh l 1.9. Cho ba dy s ( ), ( )
n n
a b v ( )
n
c tha mn: N sao cho
n n n
a b c n N v lim lim
n n
a c a = = . Khi
n
limb a = .

V d 1.16. (Canada 1985) Dy s ( )
n
x tha mn iu kin
1
1 2 x < < v
2 *
1
1
1 , .
2
n n n
x x x n
+
= +
Chng minh rng dy s cho hi t. Tm lim
n
n
x



Li gii. Ta s chng minh bng quy np bt ng thc sau:
1
2 , 3
2
n n
x n < . Tht
vy ta kim tra c ngay bt ng thc ng vi 3 n = . Gi s bt ng thc ng vi
3 n , tc l
1
2
2
n n
x < . Khi ta c
( ) 1
1
1 1
2 2 2 2 2 2 2 2 2
2 2
1 1 1 1
2 .
2 2 2 2
n n n n n
n n n
x x x x x
x
+
+
= +
< < =


=========================================================== 110
HI CC TRNG THTP CHUYN KHU VC DUYN HI V NG BNG BC B

Hi tho khoa hc mn Ton hc ln th III - 2010
Do bt ng thc ng n 1 n + . Mt khc do
1
lim 0
2
n
= nn t bt ng thc trn
v nguyn l kp ta c lim 0
n
x = .

V d 1.17. Cho dy cc hm s { } ( )
n
P x xc nh nh sau
2
0 1
( )
( ) 0, ( ) ( ) , 0;
2
n
n n
x P x
P x P x P x n x
+

= = + .
Tm lim ( )
n
n
P x

.
Li gii Trc ht ta chng minh bng quy np bt ng thc
sau: 0 ( ) ,
n
P x x n .
(1)
Tht vy, vi [0,1] x suy ra 2 0 x x nn
1
0 ( )
2
x
P x x = . Nh vy (1) ng
vi n=1. Gi s (1) ng n $n$. Xt hm s
( )
2
1
( )
2
f t t x t = + vi [0,1] t . D thy hm
s ( ) f t ng bin trn [0,1] . theo gi thit quy np ta c 0 ( ) 1
n
P x x vi mi
[0,1] x
(2)
nn
1
( ) ( ( )) ( )
n n
P x f P x f x x
+
= = vi mi [0,1] x . Mt khc, t (2) ta c
2
1
( ) 0 ( ) ( ) 0
n n n
x P x P x P x
+
. Vy
1
0 ( )
n
P x x
+
. Do (1) ng n 1 n + nn
theo nguyn l quy np ta c (1) ng vi mi n .
Tip theo ta chng minh
2
( )
1
n
x P x
n

+
vi [0,1], x n .
(3)
Tht vy ta c
1
1
( )
( ) ( ) 1
2
n
n n
x P x
x P x x P x

(
+
(
=
(



1 1
0
1
( ) 1 ( ( ) 0)
2
2
... ( ) 1 1
2 2 2
1
2 2
2 2 2
.
1 1 1 1
n n
n n
n
n
x
x P x do P x
x n x x
x P x
n
n x x
n
n
n n n n n

+
(
(

(


( (
(
=
( (


( | |
+
( |
| | ( \
= <
|
(
+ + + +
\
(
(


T ta thu c bt ng thc
2
0 ( )
1
n
x P x
n
<
+
vi mi [0,1] x n .
Do
2
lim 0
1 n
=
+
nn theo nguyn l kp ta c lim ( )
n
P x x = , vi mi [0,1] x .


=========================================================== 111
HI CC TRNG THTP CHUYN KHU VC DUYN HI V NG BNG BC B

Hi tho khoa hc mn Ton hc ln th III - 2010
V d 1.18. Cho { } , , ( , ) 1; 1, 2,... a b a b n ab ab = + +

. K hiu
n
r l s cp s
( , ) u v

sao cho n au bv = + . Chng minh rng
1
lim
n
n
r
n ab

= .

Li gii Xt phng trnh au bv n + = (1). Gi
0 0
( , ) u v l mt nghim nguyn dng
ca (1). Gi s ( , ) u v l mt nghim nguyn dng khc
0 0
( , ) u v ca (1). Ta c
0 0
, au bv n au bv n + = + = suy ra
0 0
( ) ( ) 0 a u u b v v + = do tn ti k nguyn dng sao
cho
0 0
, u u kb v v ka = + = . Do v l s nguyn dng nn
0
0
1
1
v
v ka k
a

. (2)
Ta nhn thy s nghim nguyn dng ca phng trnh (1) bng s cc s k nguyn
dng cng vi 1. Do
0 0
1 1
1 1
n
v u n
r
a ab b a
( (
= + = +
( (

. T ta thu c bt ng
thc sau:
0 0
1 1
1.
n
u u n n
r
ab b a ab b a
+
T suy ra
0 0
1 1 1 1 1
.
n
u r u
ab nb na n ab nb na n
+
T y p dng nguyn l kp ta c ngay
1
lim
n
n
r
n ab

= .

V d 1.19. Tm gii hn ca dy s ( )
n
x bit
1 2 1 3 1 1 ( 1) 1
n
x n n = + + + + + L
Li gii 1.
Xt hm s ( ) 1 1 (1 ) f x x x = + + + L ta chng minh
1
( ) 2( 1)
2
x
f x x
+
+ . T
1 1
2 2
2 (1 ) ( ) 2 (1 )
n n
x f x x

+ + .
T , thay 2 x = c
1 1
2 2
32 32
n n
n
x

< < . T , theo nguyn l kp, suy ra


lim 3
n
n
x

= .

Li gii 2. Vi1 1 m n , t 1 1 (1 ) 1 1 ( 1) 1
m
a m m n n = + + + + + + L ta c
2 2 2 2
1 1
2 2
1
1 ( 1) 2
( 1) ( ( 2))
m m m m
m m
a ma a m ma m m
a m m a m
+ +
+
= + + =

+ = +
.
Suy ra
1 2
1
| |
| ( 1) | | 2 |
| ( 1) | 2
m m
m m
m
m a a m
a m a m
a m m
+ +
+

+ +
+ + +
.
T
2 1
1 1
| 3| | | | 1 ( 1) 1 | 0 ( )
1 1
n
n n
a a n n n n n
n n


< + +
+ +

Li gii 3. rng

=========================================================== 112
HI CC TRNG THTP CHUYN KHU VC DUYN HI V NG BNG BC B

Hi tho khoa hc mn Ton hc ln th III - 2010
n& 3 1 24 1 2 16 1 2 1 3 25 1 2 1 3 1 4 36 = + = + = + + = + + + bng quy np,
d dng chng minh c
2
1 2 1 3 1 1 ( 2) 3 n n + + + + + = L
Suy ra 3
n
x
(1)

Nhn xt. Cho 1 > . Khi 1 1 0 x x x + + .
p dng nhn xt trn vi , 2 x n n = = + c
2
1 ( 2) 2 1 n n n n + + + + .
T
2 4
1 ( 1) 1 ( 2) 1 2( 1) 1 2 1 ( 1) 1 n n n n n n n n n + + + + + + + + + .
Do , bng quy np, thu c
2
3 ( 2)
n
n
n x

+ (2)

T (1),(2) v nguyn l kp, suy ra lim 3
n
n
x

= .

Bi tp tng t

Bi ton 1.20. Cho , 2 > , dy s ( )
n
a
+
tha mn iu kin
1 2 1
... , 2.
n n
a a a a n

= + + +
Chng minh rng lim 0
n
a
n
= .
Bi tp 1.21. Cho dy s dng ( )
n
a tha mn iu kin
3 *
1 1 2
... , .
n n
a a a a n
+
+ + +
Chng minh rng vi mi
1
2
> ta lun c lim 0
n
a
n

= .
Bi tp 1.22. (VMO 2002A). Xt phng trnh
2 2
1 1 1 1 1
... ...
1 4 1 1 1 2 x x k x n x
+ + + + + =

,
trong n l tham s nguyn dng.
1. Chng minh rng vi mi s nguyn dng n , phng trnh nu trn c duy nht
nghim ln hn 1; k hiu nghim l
n
x .
2. Chng minh rng dy s
n
x c gii hn bng 4 khi n +.
Bi tp 1.23. (Matxcva 2000). K hiu
n
x l nghim ca phng trnh
1 1 1
... 0
1 x x x n
+ + + =

,
thuc khong (0,1)
1. Chng minh dy ( )
n
x hi t;
2. Hy tm gii hn .
Bi tp 1.24. (VMO 2007) Cho s thc 2 a > v
10 10
( ) ... 1
n n
n
f x a x x x
+
= + + + +

=========================================================== 113
HI CC TRNG THTP CHUYN KHU VC DUYN HI V NG BNG BC B

Hi tho khoa hc mn Ton hc ln th III - 2010
1. Chng minh rng vi mi s nguyn dng n , phng trnh ( )
n
f x a = lun c ng
mt nghim dng duy nht.
2. Gi nghim l
n
x , chng minh rng dy ( )
n
x c gii hn hu hn khi n dn n
v cng.

1.4. Kho st s hi t ca dy s dng
1
( )
n n
x f x
+
=
kho st s hi t ca dy s c dng
1
( )
n n
x f x
+
= , ta thng xt hm s
( ) y f x = v s dng mt s kt qu sau
nh l 1.10. Cho dy s ( )
n
x xc nh nh sau:
1 1
, ( ), 1, 2,...
n n
x a x f x n
+
= = = . Khi

1. Nu ( ) f x l hm s ng bin th dy s ( )
n
x n iu.
2. Nu ( ) f x l hm s nghch bin th dy s ( )
n
x c cha hai dy con
2 2 1
( ), ( )
k k
x x
+

n iu ngc chiu.
3. Khi ( ) f x l hm s nghch bin v dy ( )
n
x b chn th
2 2 1
lim ,lim
k k
k k
x a x b
+

= =
v do dy cho hi t khi v ch khi a b = .

V d 1.25. (VMO 1998A). Cho s thc 1 a . Xt dy s ( ), 1, 2,...
n
x n = c xc
nh bi
2
1 1
, 1 ln
1 ln
n
n
n
x
x a x
x
+
| |
= = +
|
+
\
vi 1, 2, 3,... n =
Chng minh rng dy s trn c gii hn hu hn v tm gii hn .
Li gii
Xt dy s ( )
n
x vi
1
( 1) x a a = v
2
1
1 ln , 1, 2,...
1 ln
n
n
n
x
x n
x
+
| |
= + =
|
+
\

(i) Nu 1 a = th 1( )
n
x n = suy ra lim 1
n
n
x
+
= .
(ii) Nu 1 a > . Ta chng minh bng quy np 1
n
x > vi mi
*
n . Gi s vi n sao
cho 1
n
x > . Ta nhn thy
2
1
1 1 ln 0
n n n
x x x
+
> > . D thy hm s
2
( ) 1 ln f x x x =
ng bin trn [1; ) + . Mt khc 1
n
x > suy ra
1
1
n
x
+
> . Vy 1 1
n
x n > .
Tip theo ta chng minh vi 1 1
n
x n > th
1
1
n n
x x n
+
> . Xt hm s
2
( ) 1 ln
1 ln
x
g x x
x
| |
=
|
+
\
trn [1; ) + . Bng cch kho st hm s ny ta ch ra c ( ) g x
ng bin trn [1; ) + m (1) 0 g = , suy ra ( ) 0 1 g x x > > v ( ) 0 1 g x x = = . Do nu
1 1
n
x n > th
1
1
n n
x x n
+
> . Do vy dy ( )
n
x l dy s gim v b chn di bi 1, nn
tn ti lim
n
b
+
= . D thy 1 b v t h thc truy hi chuyn qua gii hn ta c
2 2
1 ln 1 ln 0
1 ln 1 ln
b b
b b
b b
| | | |
= + =
| |
+ +
\ \
.
Theo kt qu kho st ca hm ( ) g x trn th ( ) 0 1 g b b = = . Vy lim 1
n
n
x
+
= .
V d 1.26. Cho dy s ( )
n
x tha mn iu kin
1 1
2
2, 9; 3 , 1, 2, 3,
1
n
n
n
x
x x n
x
+
= = + =



=========================================================== 114
HI CC TRNG THTP CHUYN KHU VC DUYN HI V NG BNG BC B

Hi tho khoa hc mn Ton hc ln th III - 2010
Chng minh rng dy s trn c gii hn v tm gii hn .
Li gii
Xt hm s
2
( ) 3
1
x
f x
x
= +

vi (1, ) x + . D thy ( ) f x l hm s nghch


bin trn (1, ) + .
(i) Ta chng minh dy ( )
n
x b chn. Ta s chng minh bng quy np
*
3
3 3
2
n
u n < < + (1). Tht vy
Vi 1 n = th bt ng thc trn lun ng. Gi s bt ng thc trn ng n n , tc l
3
3 3
2
n
u < < + . Ta c
1
( )
n n
u f u
+
= v f l nghch bin trn (1, ) + nn
1
3
( 3) 3
2
n
u f
+
< = + . Mt khc do 3
n
u < nn t h thc
1
( )
n n
u f u
+
= ta c
1
3
n
u
+
< .
Vy (1) c chng minh.
(ii) T suy ra
2 2 1
lim , lim
n n
n n
a x b x
+
+ +
= = , trong , a b l nghim ca h phng
trnh
( )
( )
a f b
b f a
=

.
(iii) Xt hm s ( ) ( ( )) g x f f x x = , vi
3
3 3
2
x < < + , c
( ) ( ). ( ( )) 1 g x f x f f x = . Do
3
3 ( ) 3
2
f x < < + v ( ) 0 f x < vi mi
3
3 3
2
x < < + nn ( ) 0 g x < vi mi
3
3 3
2
x < < + , cng vi
3
( 3). ( ) 0
2
g g <
suy ra phng trnh ( ) 0 g x = c nghim duy nht. Do dy ( )
n
x hi t.
V d 1.27. (VMO 2008) Cho dy s ( )
n
x xc nh nh sau
1 2
2
0, 2
1
2 , 1, 2,...
2
n
x
n
x x
x n

+
= =

= + =


Chng minh rng dy ( )
n
x hi t v tm lim
n
n
x
+
.
Li gii 1. Bng quy np, d dng chng minh c
1 3
2
2 2
n
x n < < > . Xt hm s
1 1 3
( ) 2 , ;
2 2 2
x
f x x

| |
= +
|
\
. Ta c
1 1 3
( ) 2 ln 0 ;
2 2 2
x
f x x

| |
= <
|
\
v vi mi
1 3
;
2 2
x
| |

|
\
th
3
1 1
2 ; (0;1)
4 2
x
| |

|
\
. Do
ln 2
| ( ) | 1
2
f x u < = < .

Mt khc, theo nh l Lagrange th vi mi
1 3
2 2
x y < < u tn ti ( ; ) t x y sao cho
2 2 ( )( )
x y
f t x y

= . Vy


=========================================================== 115
HI CC TRNG THTP CHUYN KHU VC DUYN HI V NG BNG BC B

Hi tho khoa hc mn Ton hc ln th III - 2010
2 3
4 5
1 2 3
2
4 5
| | 2 2 | | |
| 2 2 |
|
| |
n n
n n
x x
n n n n
x x
n n
x x u x x
u u x x






= <
= < LL

T
2 2 1 2 1
| | | | 0 ( )
n
n n
x x u x x n

< + .
T , theo nh l Cauchy, dy $(x_n)$ hi t v l nghim ca phng trnh
1
2
2


= +

Gii phng trnh ny, thu c 1 = . Vy, lim 1
n
n
x
+
= .

Li gii 2. Bng quy np, d dng chng minh c
1 3
2
2 2
n
x n < < > .
Xt hm s
1 1 3
( ) 2 , ;
2 2 2
x
f x x

| |
= +
|
\
. Ta c
1 1 3
( ) 2 ln 0 ;
2 2 2
x
f x x

| |
= <
|
\
. Do
hm
1 3
( ), ;
2 2
y f x x
| |
=
|
\
l hm gim. Vy, mi dy ( ) ( )
2 2 1
,
k k
x x
+
cha hai dy con
n iu ngc chiu. T , do
1 3
2
2 2
n
x n < < > suy ra bn dy con
4 4 1 4 2 4 3
( ), ( ), ( ), ( )
k k k k
x x x x
+ + +
hi t theo th t v , , , .

Xt h phng trnh
( )
( )
( )
( )
f
f
f
f




=


Gii h thu c 1 = = = = . Vy lim 1
n
n
x
+
= .

Li gii 3. Bng quy np, d dng chng minh c
1 3
2
2 2
n
x n < < > .
Xt hm s
1 1 3
( ) 2 , ;
2 2 2
x
f x x

| |
= +
|
\
. Ta c
1 1 3
( ) 2 ln 0 ;
2 2 2
x
f x x

| |
= <
|
\
v vi
mi
1 3
;
2 2
x
| |

|
\
th
3
1 1
2 ; (0;1)
4 2
x
| |

|
\
. Do
ln 2
| ( ) | 1
2
f x u < = < .

Mt khc, theo nh l Lagrange th vi mi
1 3
2 2
x y < < u tn ti ( ; ) t x y sao cho
2 2 ( )( )
x y
f t x y

= . Vy, vi mi
1 3
, ;
2 2
x y
| |

|
\
tn ti
ln 2
(0;1)
2
u = sao cho
| ( ) ( ) | . | | f x f y u x y = . Suy ra hm f l hm co. Bi vy, hai dy con
2 2 1
( ), ( )
k k
x x
+
(u
cho bi h thc truy hi
2
( )
n n
x f x
+
= hi t. Bng vic gii phng trnh gii hn, thu c
lim 1
n
n
x
+
= .

=========================================================== 116
HI CC TRNG THTP CHUYN KHU VC DUYN HI V NG BNG BC B

Hi tho khoa hc mn Ton hc ln th III - 2010

Bi tp tng t
Bi tp 1.28. Cho dy s ( )
n
x xc nh nh sau
0 1 2
1, , 0
1
n
n
n
x
x x n
x
+
= =
+
. Tm lim
n
n
x

.
Bi tp 1.29. Cho trc 0 a > . Xt dy s ( )
n
x xc nh nh sau:
0
2
1
0
1
= , 0,1, 2,...
2
n n
n
x
a
x x n
x
+
>

| |

+ =
|

\

Kho st s hi t ca dy.
Bi tp 1.30. Kho st s hi t ca dy
0 1
1
( ) : 1, , 0
2
n n
n
x x x n
x
+
= =
+
.
Bi tp 1.31. Kho st s hi t ca dy
0 1 2
2
( ) : 0, , 0
1
n n
n
x x x n
x
+
=
+
.
Bi tp 1.32. Kho st s hi t ca dy
0 1 2
6
( ) : 0, , 0
2
n n
n
x x x n
x
+
=
+
.
Bi tp 1.33. Kho st s hi t ca dy
0 1
2
( ) : 1, 1 , 0
n n
n
x x x n
x
+
= = .
Bi tp 1.34. Kho st s hi t ca dy
2
0 1
3
( ) : 0, , 0
2( 1)
n
n n
n
x
x x x n
x
+
+
> =
+
.
Bi tp 1.35. Kho st s hi t ca dy
3
0 1
( ) : , 7 6, 0
n n n
x x x x n
+
= .
Bi tp 1.36. Kho st s hi t ca dy
0 1
1
( ) : 0, 1, 0
n n n
n
x x x x n
x
+
> = + .
Bi tp 1.37. Kho st s hi t ca dy
2
0 1
( ) : , 2 , 0
n n n n
x x x x x n
+
= + .
Bi tp 1.38. Kho st s hi t ca dy
0 1
( ) : ( 1; 0), 1 ( 1) 1 , 0
n
n n n
x x x x n
+
= + + .
Bi tp 1.39. Kho st s hi t ca dy
0 1 1 0
( ) : 0, , 0
n n n n
x x x x x x n
+
> = + + L .
Bi tp 1.40. Kho st s hi t ca dy
1 1
( ) : 2, 2 1
n n n
x x x x n
+
= = .
Bi tp 1.41. Kho st s hi t ca dy
2
3
0 1 2 1
( ) : 1, , , 0
n n n n
x x x a x x x n
+ +
= = = .
Bi tp 1.42. Cho dy s ( )
n
x xc nh nh sau
( )
1
2 2
1
1 2 1.
n n n
x
x x a x a n
+

= + +


Tm tt c cc gi tr thc ca tham s a sao cho dy cho hi t. Khi , tm lim
n
n
x

.
Bi tp 1.43. (VMO 2005B). Cho dy s thc ( ), 1, 2, 3...
n
x n = c xc nh bi
1
x a = v
3 2
1
3 7 5
n n n n
x x x x
+
= + vi mi 1, 2, 3,... n = , trong a l mt s thc thuc
on
4
0,
3
(
(

.
Chng minh rng dy s ( )
n
x c gii hn hu hn v tm gii hn .
Bi tp 1.44. (VMO 2005A). Cho dy s thc ( ), 1, 2, 3...
n
x n = c xc nh bi
1
x a = v
3 2
1
3 7 5
n n n n
x x x x
+
= + vi mi 1, 2, 3,... n = , trong a l mt s thc.

=========================================================== 117
HI CC TRNG THTP CHUYN KHU VC DUYN HI V NG BNG BC B

Hi tho khoa hc mn Ton hc ln th III - 2010
Hy tm tt c cc gi tr ca a dy s ( )
n
x c gii hn hu hn. Hy tm gii hn
trong cc trng hp .\hbt
Bi tp 1.45. (VMO 2001A). Vi mi cp s thc ( , ) a b , xt dy s ( ),
n
x n , c
xc nh bi
0
x a = v
1
.sin
n n n
x x b x
+
= + vi mi n .
(1) Cho 1 b = . Chng minh rng vi mi s thc a , dy ( )
n
x c gii hn hu hn khi
n +. Hy tnh gii hn theo a .
(2) Chng minh rng vi mi s thc 2 b > cho trc, tn ti s thc a sao cho dy
( )
n
x tng ng khng c gii hn hu hn khi n +.
Bi tp 1.46. (VMO 2000A). Cho c l s thc dng. Dy s ( ), 0,1, 2,...
n
x n = c
xy dng theo cch sau:
1
, 0,1, 2,...
n n
x c c x n
+
= + = nu cc biu thc trong cn l khng m.
Tm tt c cc gi tr ca c vi mi gi tr ban u ( )
0
0; x c dy ( )
n
x c xc
nh vi mi gi tr n v tn ti gii hn hu hn lim
n
x khi n +.
Bi tp 1.47. (VMO 1998B). Cho s thc a . Xt dy s ( ), 1, 2, 3,...
n
x n = c xc
nh bi
( )
2
1 1 2
3
,
3 1
n n
n
n
x x
x a x
x
+
+
= =
+
vi n=1, 2, 3, ...
Chng minh rng dy s trn c gii hn hu hn v tm gii hn .
Bi tp 1.48. (VMO 1994B). Cho s thc a . Xt dy s ( ), 0,1, 2,...
n
x n = c xc
nh bi
3
0 1 1
, 6 6sin
n n n
x a x x x

= = vi mi n=1, 2, 3,...
Chng minh rng dy s trn c gii hn hu hn khi n dn ti dng v cc v tm
gii hn .
Bi tp 1.49. (VMO 1994A). Cho s thc a . Xt dy s ( ), 0,1, 2,...
n
x n = c xc
nh bi
0 1 1 2
4
, arccos .arcsin
2
n n n
x a x x x


| |
= = +
|
\
vi mi n=1, 2, 3,...
Chng minh rng dy s trn c gii hn hu hn khi n dn ti dng v cc v tm
gii hn .

1.5. nh l trung bnh Cesaro v dy s dng
1
a
n n n
x x x
+
=
y l trng hp c bit ca dy s dng
1
( )
n n
x f x
+
= . Tuy nhin, chng ta khng
t vn kho st s hi t ca nhng dy dng ny, bi v gii hn ca chng hoc l 0
hoc l ; m y chng ta quan tm ti tt c cc s sao cho dy
n
x
n

| |
|
\
hi t. Vi
nhng dy s dng ny, nh l trung bnh Cesaro t ra rt hu hiu.

nh l 1.11. Nu dy s ( )
n
x c gii hn hu hn l a th dy s cc trung bnh
1 2
...
n
x x x
n
+ + + | |
|
\
cng c gii hn l a .
Chng minh.

=========================================================== 118
HI CC TRNG THTP CHUYN KHU VC DUYN HI V NG BNG BC B

Hi tho khoa hc mn Ton hc ln th III - 2010
Khng mt tnh tng qut ta c th gi s 0 a = . Vi mi 0 > tn ti
*
N sao
cho vi mi n N th
2
n
u

< v
1 2
...
2
N
u u u
n
+ + +
< . T ta c
( )
1 2 1 2 1
... ... ...

2 2
n N N n
n N
u u u u u u u u
n N
n n n n

+

+ + + + + + + +
+ < + < .

V d 1.50. Nu ( )
1
lim
n n
n
x x a
+

= th lim .
n
n
x
a
n

=
Li gii. t
1 n n n
u x x

= . Khi d thy dy ( )
n
u tha mn iu kin ca nh l
Cesaro nn ta c
1
...
lim
n
n
u u
a
n

+ +
= hay lim
n
n
x
a
n

= .

V d 1.51. Chng minh rng nu dy s dng ( )
n
a hi t v a dng th
1 2
lim ...
n
n
n
a a a a

= .

Li gii. Ta c limln ln
n
n
a a

= . p dng nh l Cesaro, ta c:
1
ln ... ln
lim ln
n
n
a a
a
n

+ +
= hay
1 2
lim ...
n
n
n
a a a a

= .

V d 1.52. Cho dy s dng ( )
n
a . Chng minh rng
1
lim 0
n
n
n
a
a
a
+

= > th lim
n
n
n
a a

= .
Li gii. t
1
, 2
n
n
n
a
b n
a
+
= . D thy dy ( )
n
b tha mn v d 1.50 nn
1 2
lim ...
n
n
n
b b b a

= hay lim
n
n
n
a a

= .

Bi tp tng t

Bi tp 1.53. Cho dy ( )
n
x xc nh bi
2
1 1
1/ 2,
n n n
x x x x
+
= = . Chng minh rng
lim 1.
n
n
nx

=
Bi tp 1.54. Cho dy ( )
n
x xc nh bi
1 1
1, sin
n n
x x x
+
= = . Chng minh rng
lim 1.
n
n
nx

=
Bi tp 1.55. (TST VN 1993). Dy s { }
n
x xc nh bi
1 1
1
1,
n n
n
x x x
x
+
= = + . Hy
tm tt c cc s dy s
( )
n
a
n

c gii hn hu hn khc 0.
Bi tp 1.56. Cho dy s xc nh bi ( )
1 1
0, 1 sin 1 , 1
n n
a a a n
+
= = . Tnh
1
1
lim
n
k
n
k
a
n

.

=========================================================== 119
HI CC TRNG THTP CHUYN KHU VC DUYN HI V NG BNG BC B

Hi tho khoa hc mn Ton hc ln th III - 2010
Bi tp 1.57. Xt dy s ( )
n
x xc nh bi
1 1
3
1
1, 1
n n
n
x x x n
x
+
= = + . Chng minh
rng tn ti , a b sao cho lim 1
n
b
n
x
an

= .


2 Bi ton dy s qua cc k thi IMO

2.1 IMO 2009

Bi 2.1.1 (IMO 2009) . Gi s
1 2 3
, , ,... s s s l mt dy tng ngt cc s nguyn dng
sao cho cc dy con
1 2 3
, , ,...
s s s
s s s v
1 2 3
1 1 1
, , ,...
s s s
s s s
+ + +
u l cp s cng. Chng minh rng
1 2 3
, , ,... s s s cng l mt cp s cng.

Bi 2.1.2 (M rng IMO 2009). Cho k l mt s nguyn dng cho trc. Gi s
1 2 3
, , ,... s s s l mt dy tng nght cc s nguyn dng sao cho cc dy con
1 2 3
, , ,...
s s s
s s s v
1 2 3
, , ,...
s k s k s k
s s s
+ + +
u l cp s cng. Chng minh rng
1 2 3
, , ,... s s s cng l mt cp s cng.

Chng minh.
Gi D v E ln lt l cng sai ca cc cp s cng
1 2 3
, , ,...
s s s
s s s
v
1 2 3
, , ,...
s k s k s k
s s s
+ + +
. t
1
s
A s D = v
1
s k
B s E
+
= . Theo cng thc tnh s hng tng
qut ca mt cp s cng v vi s nguyn dng n ta c
1 1
( 1) , ( 1) .
n n
s s s k s k
s s n D A nD s s n E B nE
+ +
= + = + = + = +
T dy
1 2 3
, , ,... s s s l mt dy tng ngt, nn vi mi s nguyn dng n v vi ch
n n k
s k s
+
+ ta c
1 ,
n n n k
s s k s
s k s s
+
+
+ <
t ta thu c
1 ( 1) , A nD k B nE A n D + + < + + +
iu ny tng ng vi
0 1 ( ) , k B A n E D kD < + +
nu D khc E th cho n ta thy mu thun vi bt ng thc trn nn D E = v
do
0 1 . k B A kD + (1)
t { }
1
min : 1, 2,...
n n
m s s n
+
= = . Khi
1 1 1 1
( ) ( )
s k s s k s
B A s E s D s s km
+ +
= = (2)
v
1 1
1 1
( ) ( ) ( ).
s k s
s s B D A D
kD A s k D A s D s s s s m B A
+
+ +
= + + + = = (3)
Ta xt hai trng hp
(a) B A kD = .
Khi , vi mi s nguyn dng , ( )
n n k
s k s
n s B nD A n k D s
+
+
= + = + + = , t y kt hp
vi dy
1 2 3
, , ,... s s s l mt dy tng ngt ta c
n k n
s s k
+
= + . Mt khc do
1
...
n n n k n
s s s s k
+ +
< < < = + nn
1
1
n n
s s
+
= + v do
1 2 3
, , ,... s s s l mt cp s cng vi cng
sai bng 1.

=========================================================== 120
HI CC TRNG THTP CHUYN KHU VC DUYN HI V NG BNG BC B

Hi tho khoa hc mn Ton hc ln th III - 2010
(b) B A kD < .
Chn s nguyn dng N sao cho
1 N N
s s m
+
= . Khi
( ) (( ( 1) ) ( )) m A B D k m A N D B ND k + = + + + +
1
( 1)
s s k
N N
A N D B ND k s s k
s s s s
+
+
+ + + + +
=

1 1
( ) ( ( ) ) ( )
N N N N
A s D B s k D s s D A B kD
+ +
= + + + = +
, mD A B kD = +
do vy
( ) ( ( )) 0. B A km kD m B A + (4)
T cc bt ng thc (2), (3) v (4) ta thu c cc ng thc sau:
B A km = v ( ) kD m B A = .
Gi s tn ti s nguyn dng n sao cho
1 n n
s s m
+
> + . Khi
1
1
( 1) ( )
n n
n n s s
m m m s s s s
+
+
+
2
( )
( ( 1) ) ( )
m B A
A n D A nD D m
k

= + + + = = = ,
v l.
V vy iu gi s l sai nn
1 n n
s s m
+
= + vi mi n hay dy
1 2
, ,... s s l mt cp s
cng c cng sai bng m.

Nhn xt

By gi ta thay cp s cng bi cp s nhn khi bi ton trn cn ng khng?

Bi 2.1.3. Gi s
1 2 3
, , ,... s s s l mt dy tng nght cc s nguyn dng sao cho cc
dy con
1 2 3
, , ,...
s s s
s s s v
1 2 3
1 1 1
, , ,...
s s s
s s s
+ + +
u l cp s nhn. Chng minh rng
1 2 3
, , ,... s s s
cng l mt cp s nhn.

Bi 2.1.4. (M rng ca bi ton 2.1.3) Cho k l mt s nguyn dng. Gi s
1 2 3
, , ,... s s s l mt dy tng nght cc s nguyn dng sao cho cc dy con
1 2 3
, , ,...
s s s
s s s v
1 2 3
, , ,...
s k s k s k
s s s
+ + +
u l cp s nhn. Chng minh rng
1 2 3
, , ,... s s s cng l mt cp s nhn.

IMO 2010

Bi 2.2.1
Cho
1 2 3
, , ,... a a a l mt dy s thc dng. Gi s vi s nguyn dng s cho trc,
ta c
{ } max :1 1
n k n k
a a a k n

= + ,
vi mi n s > . Chng minh rng tn ti cc s nguyn dng l v N , vi l s v tha
mn
n l n l
a a a

= + vi mi n N .
Chng minh.

T iu kin bi ton v vi mi
n
a ( n s > ) ta c ng thc sau
1 2
n j j
a a a = + vi
1 2 1 2
, , j j n j j n < + = nu
1
j s > th ta c th vit c
1
j
a ging nh
n
a . Cui cng, ta c
th vit c ng thc di y
1 2
... ,
k
n i i i
a a a a = + + + (1)

=========================================================== 121
HI CC TRNG THTP CHUYN KHU VC DUYN HI V NG BNG BC B

Hi tho khoa hc mn Ton hc ln th III - 2010
1 2
1 , ... , 1, 2,...,
j k
i s i i i n j k + + + = = . (2)
C nh ch s 1 l s sao cho
1
min .
l i
i s
a a
m
l i

= =
Ta xc nh dy { }
n
b vi
n n
b a mn = , 0
l
b = .
Ta s chng minh vi mi n th 0
n
b , v dy { }
n
b tha mn cc tnh cht ging nh
dy { }
n
b .
Tht vy nu n s th ta c ngay 0
n
b theo cch xc nh ca m. By gi ta xt nu
n s > v s dng phng php quy np cng vi nh gi sau
1 1 1 1
max ( ) max ( )
n k n k k n k
k n k n
b a a nm b b nm nm


= + = + +
1 1
max ( ),
k n k
k n
b b


= +
ta thu c 0
n
b .
Nu 0
k
b = vi mi 1 k s , khi 0
n
b = vi mi n , v vy
n
a mn = , v trng hp
ny l tm thng.
Nu tn ti 1 1 k n sao cho
n
b khc 0, ta xc nh
{ }
1
max , min :1 , 0 .
i i i
i s
M b b i s b

= = >
Khi vi n s > ta t c
1 1
max ( ) ,
n k n k l n l n l
k n
b b b b b b


= + + =
v vy
2
... 0.
n n n l n l
M a b b b


Ta c dy ( )
n
b cng c tnh cht (1), (2) ging nh dy ( )
n
a , ta c vi mi
n
b cha
trong tp
{ } [ ]
1 2
1
... :1 ,..., 0,
k
i i i k
T b b b i i s M = + + + .
Ta chng minh tp ny ch c hu hn phn t. Tht vy, vi mi x T , biu din c
1 2
1
... (1 ,..., )
k
i i i k
x b b b i i s = + + + . Khi ch c ti a
M

s
j
i
b khc 0 (v nu ngc li
th .
M
x M

> = iu ny v l). V vy x ch c th biu thnh tng ca k s


j
i
b vi
M
k

, v do tp ny ch c hu hn.
T ta c ngay dy
n
b l mt dy tun hon vi chu k l t mt ch s N tr i, c
ngha l
n n l n l l
b b b b

= = + vi n N l > + ,
v do
( ( ) ) ( )
n n n l l n l l
a b nm b n l m b lm a a

= + = + + + = + vi mi n N l > + .
T bi ton ny ta c th xy dng c mt s dng bi tp sau v iu kin dy s
dng l khng cn thit.

Bi 2.2.2
Cho
1 2 3
, , ,... a a a l mt dy s thc. Gi s vi s nguyn dng s cho trc, ta c
{ } min :1 1
n k n k
a a a k n

= +

=========================================================== 122
HI CC TRNG THTP CHUYN KHU VC DUYN HI V NG BNG BC B

Hi tho khoa hc mn Ton hc ln th III - 2010
vi mi n s > . Chng minh rng tn ti cc s nguyn dng l v N , vi l s v tha
mn
n l n l
a a a

= + vi mi n N .

Bi 2.2.3
Cho
1 2 3
, , ,... a a a l mt dy s thc dng. Gi s vi mi s nguyn dng s , ta c
{ } max . :1 1
n k n k
a a a k n

=
vi mi n s > . Chng minh rng tn ti cc s nguyn dng l v N , vi l s v tha
mn .
n l n l
a a a

= vi mi n N .

Chng minh.

t ln
n n
b a = th dy
1 2 3
, , ,... b b b l mt dy s thc v vi cch chng minh tng
t nh bi 2.2.1 ta s thu c kt qu bi ton trn.





































=========================================================== 123
HI CC TRNG THTP CHUYN KHU VC DUYN HI V NG BNG BC B

Hi tho khoa hc mn Ton hc ln th III - 2010
S DNG CNG C S PHC GII CC BI TON
HNH HC PHNG
Trng THPT chuyn H Long

Ta bit rng mi s phc c biu din bi mt im trong mt phng phc. Do
cng nh phng php to , khi ng nht mi im trong mt phng bi mt s phc th
bi ton trong hnh hc phng thnh bi ton vi s phc m ta bit rng cc cng thc v
khong cch v gc c th a v cc cng thc n gin i vi s phc. Do vy ta c th
s dng s phc gii cc bi ton hnh hc t n gin n phc tp. Trong bi ny, ta
quy c mi im A c biu din n trong mt phng phc, do ta c cc khi nim
tng ng l ng thng ab, tam gic abc s dng c cng c ny ta cn nm c
cc cng thc v nh l sau:
1. Cc cng thc v nh l:
nh l 1.1
ng thng ab//cd khi v ch khi
a b c d
a b c b

=

.
Cc im a, b, c thng hng khi v ch khi
a b a c
a b a c

=


ng thng ab vung gc vi cd khi v ch khi
a b c d
a b c b

=


Gi l gc acb theo chiu dng t a n b th
i
c b c a
e
c b c a


=

.
nh l 1.2 Trn ng trn n v, ta c cc tnh cht sau:
Hai im a, b thuc ng trn n v th
a b
ab
a b


im c nm trn dy cung ab th
a b c
c
ab
+
= .
Giao ca hai tip tuyn ti hai im a, b l im
2ab
a b +
.
Chn ng cao h t mt im c bt k xung dy ab ca ng trn l im
1
( )
2
a b c abc + + .
Giao im ca hai dy cung ab v cd l im
( ) ( ) ab c d cd a b
ab cd
+ +

.
nh l 1.3
4 im a, b, c, d cng thuc mt ng trn khi v ch khi :
a c a d
b c b d


.
nh l 1.4
Tam gic abc v tam gic pqr ng dng v cng hng khi v ch khi
a c p r
b c q r

=

.
nh l 1.5 Din tch c hng ca tam gic abc l
( )
4
i
S ab bc ca ab bc ca = + + .

=========================================================== 124
HI CC TRNG THTP CHUYN KHU VC DUYN HI V NG BNG BC B

Hi tho khoa hc mn Ton hc ln th III - 2010
nh l 1.6 im c chia on thng ab theo t s 1 khi v ch khi
1
a b
c

.
im G l trng tm tam gic abc khi v ch khi
3
a b c
g
+ +
= .
Vi H l trc tm v O l tm ngoi tip th h+2o=a+b+c.
nh l 1.7 Gi s ng trn n v ni tip tam gic abc v tip xc vi cc cnh bc,
ca, ab ca tam gic abc ti p, q, r th
a.
2 2 2
, ,
qr rp pq
a b c
q r r p p q
= = =
+ + +

b. Vi h l trc tm tam gic abc ta c
2 2 2 2 2 2
2( ( ))
( )( )( )
p q q r r p pqr p q r
h
p q q r r p
+ + + + +
=
+ + +

c. Vi tm ng trn bng tip o, tng t ta c:
2 ( )
( )( )( )
pqr p q r
o
p q q r r p
+ +
=
+ + +
.
nh l 1.8 Cho tam gic abc ni tip ng trn n v, khi tn ti cc s u, v, w
sao cho
2 2 2
, , w a u b v c = = = v uv,-vw,-wu l trung im ca cc cung ab, bc, ca khng
cha cc nh i din. Khi tm ng trn ni tip i c i=-(uv+vw+wu).
nh l 1.9 Nu tam gic c mt nh trng vi gc to v cc nh cn li l x, y th
Trc tm l im
( )( ) xy xy x y
h
xy xy
+
=


Tm ng trn ngoi tip l im
( ) xy x y
o
xy x y

.
Ta bt u vi mt s v d nh sau:

1. Cho tam gic ABC tm ng trn ngoi tip O, trc tm H. Q l im i xng vi
H qua O. K hiu A;B;C l trng tm cc tam gic BCQ, ACQ, ABQ. Chng minh rng:
4
' ' '
3
AA BB CC R = = = .
Gii:
Gi s bn knh ng trn ngoi tip bng 1 v tm ng trn ngoi tip trng vi
gc to . Gi s cc im A, B, C biu din bi cc s a, b, c (|a|=|b|=|c|=1) khi trc
tm h=a+b+c v do O l trung im ca HQ nn q=-a-b-c. Do A l trng tm tam gic BCQ
nn A=(b+c+q)/3=-a/3. Ta c
4 4
AA'
3 3 3
a
a R = + = = . Lm tng t ta suy ra pcm.
2. Cho t gic ABCD ni tip v A, B, C, D ln lt l trc tm cc tam gic BCD,
CDA, DAB, ABC. Chng minh rng hai t gic ABCD v ABCD ng dng.
Gii:
Xt 4 s phc a, b, c, d trn ng trn n v. Khi a=b+c+d; b=c+d+a.
Khi d thy a-b=a-b, b-c=b-c; c-a=c-a nn suy ra tam gic ABC ng dng vi
tam gic ABC. Lm tng t vi cc tam gic cn li v d suy ra t gic ABCD v
ABCD ng dng.
3. Cho tam gic ABC, v pha ngoi tam gic dng cc hnh vung BCDE, CAFG,
ABHI. Dng cc hnh bnh hnh DCGQ, EBHP. Chng minh rng tam gic APQ vung
cn.

=========================================================== 125
HI CC TRNG THTP CHUYN KHU VC DUYN HI V NG BNG BC B

Hi tho khoa hc mn Ton hc ln th III - 2010
Ta s dng mt kt qu d chng minh nh sau: Nu b l nh ca a qua php quay
tm c gc quay th ( )
i
b c e a c

= .
Gii (ph thuc vo gc quay ca hnh v)
V H l nh ca A qua php quay tm B gc quay
2

nn
2
( ) ( )
i
h b e a b i a b

= = do (1 ) h i b ia = + . Tng t (1 ) e i b ic = + . Do
EBHP l hnh bnh hnh nn b+p=h+e nn tnh c p=b+ia-ic.
Tng t tnh c q=-ia+ib+c
Khi p-a=b+(i-1)a-ic v q-a=(-i-1)a+ib+c
D thy p-a=-i(q-a) nn p l nh ca q qua php quay tm A gc quay
2

.
Bi tp:
4. Cho t gic ABCD. V pha ngoi t gic dng cc tam u BCM, CDN, DAP. I, E,
F l trung im ca AB, MN, NP. Chng minh rng tam gic IEF cn.
5. Cho t gic li ABCD vi AC=BD. Dng pha ngoi t gic cc tam gic u cnh
AB, BC, CD, DA v gi G
1
;G
2
;G
3
;G
4
l trng tm cc tam gic . Chng minh rng G
1
G
3

vung gc vi G
2
G
4.


Vi a gic u n cnh
0 1 1
...
n
A A A

ta c th gi s chng ni tip trong ng trn


n v. Khi ta cn c th chn c cc nh ca n l cc cn bc n ca n v tc l
2 k
i
n
i
a e

= vi 0 1 i n . Nh vy cc nh ca n c th vit di dng
i
i
a = vi
0 1 i n v
1
a = .
6. Cho a gic u 7 cnh
0 1 6
... A A A . Chng minh rng
0 1 0 2 0 3
1 1 1
A A A A A A
= +
A0
A1
A2
A3
A4
A5
A6
A1'
A2'

Gii: Gi s cc nh ca a gic li trn l
i
i
a = vi 0 6 i v
2
7
i
e

= .

=========================================================== 126
HI CC TRNG THTP CHUYN KHU VC DUYN HI V NG BNG BC B

Hi tho khoa hc mn Ton hc ln th III - 2010
Khi d chng minh c

1 0 3
2
7
A A A

= v do nh ca
1
A qua php quay tm
0
A
gc quay
2
7

l im
1
' A thuc on
0 3
A A v
2
' ' 2 2 2
7
1 1 1
1 ( 1) ( 1) 1 ( 1) 1
i
a e a a w a w w

= = + = + trong
7
w
i
e

= .
Tng t

2 0 3
7
A A A

= v nh ca ca
2
A qua php quay tm
0
A gc quay
7

l im
2
' A thuc on
0 3
A A v
' ' 4
7
2 2 2
1 ( 1) ( 1) 1
i
a e a a w w

= = + .
Do cc im A
0
;A
1
,A
2
,A
3
thng hng nn ta ch cn chng minh h thc:
( )
2 2 4 6
1 1 1
( 1) 1 1 w w w w w
= +

vi
7
w
i
e

= . H thc ny xin dnh cho bn c.


7. Cho a gic u 15 cnh
0 1 14
... A A A . Chng minh h thc

0 1 0 2 0 4 0 7
1 1 1 1
A A A A A A A A
= + + .
8. Cho a gic u n cnh ni tip ng trn bn knh r. Chng minh rng vi mi
im P nm trn ng trn v vi mi s t nhin m<n th
1
2 2
2
0
n
m m m
k m
k
PA C nr

=
=

.
9. Cho tam gic ABC v hai im phn bit M, N sao cho AM:BM:CM=AN:BN:CN.
Chng minh rng MN i qua tm ng trn ngoi tip tam gic ABC.
10. Cho P l mt im ty trn cung

0 1 n
A A

ca ng trn ngoi tip a gic u


0 1 1
...
n
A A A

. Gi h
i
l khong cch t P n cc ng thng A
i-1
A
i
vi i=1,2,..,n. Chng
minh rng
1 1
1 1 1
...
n n
h h h

+ + = .
Vi tam gic u, ta c th a ra mt iu kin cn v mt tam gic l tam gic
u nh sau:
A2
A0
A1

Xt tam gic u
0 1 2
A A A vi
i
i
a = ,i=0,1,2 v
2 2
cos sin
3 3
i

= + v tam gic
u ABC bt k vi n: Theo nh l 1.4 ta c h thc:

=========================================================== 127
HI CC TRNG THTP CHUYN KHU VC DUYN HI V NG BNG BC B

Hi tho khoa hc mn Ton hc ln th III - 2010
1 0
2
2 0
1 1
1 1
a c a a
b c a a



= = =
+
(1) nu hai tam gic trn cng hng
1 0
2
2 0
1 1
1 1
a b a a
c b a a



= = =
+
(2) nu hai tam gic trn ngc hng.
Hai h thc trn c th vit li l
2 2
(1) 0 0 a b c a b c + + = + + =
2
2 2
(2) 0 0 0 a b c a b c a b c + + = + + = + + =
Do vy iu kin cn v tam gic ABC u l
2
0 a wb w c + + = trong w l
mt cn bc 3 khc 1 ca n v. Ch rng nu
2
3
w
i
e

= th tam gic trn c hng


dng cn nu
2
( )
3
w
i
e

= th tam gic trn c hng m.


p dng iu kin trn ta c th gii cc bi ton sau:
11. Cho tam gic ABC, v pha ngoi tam gic dng cc tam gic u MBC, NCA,
PAB. Gi G, H, I l trng tm ca cc tam gic trn. Chng minh rng tam gic GHI u.
Xt tam gic ABC c hng dng th cc tam gic MCB, NAC, PBA trn cng c
hng dng. Theo cc h thc trn ta c
2
2
2
0
0
0
m c b
n a c
p b a



+ + =
+ + =
+ + =

chng minh tam gic GHI u ta chng minh
2
0 g h i + + = trong
3
m b c
g
+ +
= . Vic chng minh xin dnh cho bn c.

12. Trong mt phng cho tam gic A
1
A
2
A
3
v im P
0
. K hiu cc im A
a
=A
s-3
vi
mi s t nhin s>3. Xt dy cc im P
0
;P
1
; cho bi P
k+1
l nh ca P
k
qua php quay
tm A
k+1
gc quay 120
0
. Chng minh rng nu P
1986
=P
0
th tam gic A
1
A
2
A
3
l tam gic u.

Trong cc bi ton v a gic ni tip ta c th gi s chng ni tip trong ng trn
n v. Sau y l mt s v d
13. Cho H l trc tm tam gic ABC v P l mt im tu trn ng trn ngoi tip.
E l chn ng cao k t B v dng cc hnh bnh hnh PAQB v PARC. X l giao im
ca AQ v HR. Chng minh rng EX//AP.

=========================================================== 128
HI CC TRNG THTP CHUYN KHU VC DUYN HI V NG BNG BC B

Hi tho khoa hc mn Ton hc ln th III - 2010
A
B
C
H
E
O
P
Q
R
X

Gii: Xt tam gic ABC ni tip trong ng trn n v, khi h=a+b+c v
1
( )
2
ac
e a b c
b
= + + . Do APBQ l hnh bnh hnh nn q=a+b-p, tng t r=a+c-p
Do x, a, q thng hng nn
x a a q p b
pb
x a a q p b

= = =

(p, b thuc ng trn n
v). Do
2
ax x a pb a
x a
pb abp
+
= + =

(1). Tng t cc im h,r,x thng hng nn ta


tnh c
x h
pb
x h

nn
bp bp
x a b c p
a c
x
bp
+ + +
= (2).

T (1) v (2) ta tnh c
1
(2 )
2
bp
x a b c p
c
= + + .
chng minh XE//AP ta chng minh
e x a p
ap
e x a b

= =

.
Ta c
2 2
1 ( )( )
( )
2 2 2
bp ac bcp b p abc ac b c bp ac
e x p a
c b bc bc
+ +
= + = =
V
1 1 1 1
( )( )
1
.
1 1
2
2 .
bp ac b c bp ac
e x
ap bc
b c
+

= = nn ta c iu phi chng minh.


14. Cho t gic ABCD ni tip, P v Q l cc im i xng vi C qua AB v AD.
Chng minh rng PQ i qua trc tm tam gic ABD.

=========================================================== 129
HI CC TRNG THTP CHUYN KHU VC DUYN HI V NG BNG BC B

Hi tho khoa hc mn Ton hc ln th III - 2010
A
D
C
B
Q
P

Gi s t gic ABCD ni tip trong ng trn n v. Khi
ab
p a b
c
= +
,
ad
q a d
c
= + + v h=a+b+d.
1 1 1 1 1
ab
a b a b d
p h abd
c
c
c p h
a b ab a b d
+

= =

+
. Tng t
q h abd
c q h

. Do P, Q, H
thng hng.
15. Tam gic ABC trc tm H ni tip ng trn (O) bn knh R. D l im i xng
vi A qua BC, E l im i xng vi B qua CA, F i xng vi C qua AB. Chng minh
rng cc im D, E, F thng hng khi v ch khi OH=2R.
16. Cho lc gic ABCDEF ni tip. Chng minh rng cc giao im ca AB v DE, BC
v EF, CD v FA thng hng.
17. Cho t gic ABCD ni tip, AB ct CD ti E, AD ct BC ti F, AC ct BD ti G.
Chng minh rng O l trc tm tam gic EFG.
18. Cho t gic ABCD ni tip v K, L, M, L l trung im ca AB, BC, CD, DA.
Chng minh rng cc trc tm tam gic AKN, BKL, CLM, DMN to thnh cc nh ca
mt hnh bnh hnh.
S dng nh l 1.7 ta c th gii c mt s bi ton lin quan n ng trn ni
tip a gic
19. ng trn (I) ni tip tam gic ABC v tip xc vi cc cnh ca tam gic ti P, Q,
R. Gi H l giao im ca PR v AC. Chng minh rng IH vung gc vi BQ.
20. Cho ng trn (O) ni tip t gic ABCD v tip xc vi cc cnh AB, BC, CD,
DA ti K, L, M, N. KL ct MN ti S. Chng minh rng OS vung gc vi BD.


Trn y l mt s ng dng n gin ca s phc i vi nhng bi ton hnh hc
phng. Hy vng sau bi vit ny, cng vi phng php to trong mt phng chng ta c
thm mt cch nhn na v cch gii cho cc bi ton hnh hc thng thng.

Ti liu tham kho
- Complex number in Geometry Marko Radovanovic
-Tp ch Mathematical Excalibur Vol. 1,No.3,May-Jun,95
- Mt s ti liu trn mng.


=========================================================== 130
HI CC TRNG THTP CHUYN KHU VC DUYN HI V NG BNG BC B

Hi tho khoa hc mn Ton hc ln th III - 2010
BT BIN TRONG CC BI TON L THUYT TR CHI
Phm Minh Phng
Gio vin trng THPT chuyn i hc S phm H Ni

Bt bin l khi nim quan trng ca ton hc. Ni mt cch n gin th bt bin l i
lng hay tnh cht khng thay i trong khi cc trng thi bin i. Ngi ta s dng bt
bin phn loi cc vt trong mt phm tr no . Hai vt thuc cng mt loi nu n c
cng tnh cht H v nu vt A c tnh cht H, vt B khng c tnh cht H th B khng cng
loi vi A.
Trong chuyn ny chng ti xin gii thiu v ng dng ca bt bin trong cc bi
ton v thut ton ca l thuyt tr chi. y l dng ton thng gp trong cc k thi
Olympic.
1. Mt s khi nim ca l thuyt tr chi
1. Thut ton
Cho tp A v ta gi l khng gian cc trng thi, mi phn t ca A l mt
trng thi. Khi , mi nh x: : T A A gi l mt thut ton (tmat).
2. Cc bi ton v thut ton
Bi ton 1 (Bi ton tm kim thut ton). Cho trng thi ban u
0
v trng
thi kt thc
n
. Hi c hay khng thut ton T trn A sao cho khi thc hin T hu
hn ln ta thu c
n
?
0 1 2
...
T T T T
n

Bi ton 2. Cho thut ton T trn A v trng thi ban u .
a) Xt trng thi . A Hi c th nhn c t sau hu hn ln thc hin
thut ton T hay khng?
b) Tm tp hp gm tt c cc trng thi c th nhn c t sau hu hn
bc thc hin thut ton T:
( ) { }
:
n
A T = =
3. Hm bt bin
Cho thut ton T trn A v I l mt tp hp khc rng m ta gi l khng gian
cc mu bt bin.
Khi , nh x : H A I gi l hm bt bin trn A nu
, : ( ) ( ). a b A b a H b H a =

2. Mt s bi ton minh ho
Bi ton 1. Hai ngi chi c. Sau mi vn ngi thng c 2 im, ngi thua c
0 im, nu ho th mi ngi c 1 im. Hi sau mt s vn liu c th xy ra trng
hp mt ngi c 7 im v ngi kia c 10 im c khng?
Li gii. Gi ( ) S n l tng s im ca c hai ngi sau vn th n. Ta c ( ) S n bt bin
theo modun 2. Do
( ) ( ) ( ) 0 0 mod 2 , 0. S n S n
Vy khng th xy ra trng hp mt ngi c 7 im v ngi kia c 10 im.

=========================================================== 131
HI CC TRNG THTP CHUYN KHU VC DUYN HI V NG BNG BC B

Hi tho khoa hc mn Ton hc ln th III - 2010
Bi ton 2. Thc hin tr chi sau: Ln u vit ln bng cp s
( )
2; 2 . T ln th
hai, nu trn bng c cp s ( ) ; B a b = th c php vit thm cp s
( ) ; .
2 2
a b a b
T B
+ | |
=
|
\

Hi c th vit c cp s
( )
1;1 2 + hay khng?
Li gii. Gi s bc th n ta vit cp s ( ) ; .
n n
a b Khi tng ( )
2 2
n n
S n a b = + l i
lng bt bin. Do
( )
( )
2
2 2 2
0 0
6 1 1 2 , 0. S n a b n = + = + +
Vy khng th vit c cp s
( )
1;1 2 + .
Bi ton 3. Trn bng c hai s 1 v 2. Thc hin tr chi sau: Nu trn bng c hai s
a v b th c php vit thm s . c a b ab = + + Hi bng cch c th vit c cc s
2001 v 11111 hay khng?
Li gii. Dy cc s vit thm l: 5; 11; 17; ...
D dng chng minh c dy cc s c vit thm u chia cho 3 d 2. Bt bin trn
cho php ta loi tr s 2001 trong dy cc s c vit thm. Tuy nhin, bt bin khng
cho php ta loi tr s 11111. Ta i tm mt bt bin khc. Quan st cc s vit c v quy
tc vit thm s, ta c
( )( ) 1 1 1 c a b ab c a b = + + + = + +
v nu cng thm 1 vo cc s thuc dy trn ta c dy mi: 6; 12; 18; ...
Nh vy, nu cng thm 1 vo cc s vit thm th cc s ny u c dng: 2 .3 .
m n
Do s
11111 1 11112 3.8.463 + = = nn 11111 khng thuc dy cc s c vit thm.
Bi ton 4 (VMO 2006). Xt bng vung ( ) , 3 . m n m n Thc hin tr chi sau:
mi ln t 4 vin bi vo 4 ca bng, mi mt vin bi, sao cho 4 to thnh mt
trong cc hnh di y:



Hi sau mt s ln ta c th nhn c bng m s bi trong cc bng nhau c khng
nu:
a) 2004, 2006? m n = =
b) 2005, 2006? m n = =
Li gii
a) Bng cho c th chia thnh cc hnh ch nht 4 2 nn c th nhn c trng
thi m s bi trong cc bng nhau.
b) T mu cc nh hnh v





=========================================================== 132
HI CC TRNG THTP CHUYN KHU VC DUYN HI V NG BNG BC B

Hi tho khoa hc mn Ton hc ln th III - 2010
D thy, mi ln t bi c 2 vin c t vo cc mu en v 2 vin c t vo
mu trng. Do , nu gi ( ) S n l s bi trong cc mu en v ( ) T n l s bi trong
cc mu trng sau ln t bi th n th ( ) ( ) S n T n l i lng bt bin. Ta c
( ) ( ) ( ) ( ) 0 0 0, 0. S n T n S T n = =
V 2005 m = l s l nn nu nhn c trng thi m s bi trong cc bng nhau
th
( ) ( ) 2005, S n T n m = =
v l.

Bi ton 5 (IMO 2004). Ta nh ngha vin gch hnh mc cu l hnh gm 6 vung
n v nh hnh v di y, hoc hnh nhn c do lt hnh (sang tri, sang phi, ln
trn, xung di) hoc hnh nhn c do xoay hnh i mt gc:


Hy xc nh tt c cc hnh ch nht m n , trong m, n l cc s nguyn dng sao
cho c th lt hnh ch nht bng cc vin gch hnh mc cu?
Li gii. D thy { } , 1; 2;5 . m n Chi hnh ch nht cho thnh cc m n vung v
nh s cc hng, cc ct t di ln trn, t tri sang phi. Ta gi ( ) ; p q l nm giao
ca hng th p v ct th q. Hai vin gch hnh mc cu ch c th ghp li c mt
trong hai hnh di y:


Do , lt c hnh ch nht m n th . mn phi chia ht cho 12. Nu t nht mt
trong hai s m, n chia ht cho 4 th c th lt c. Tht vy, gi s c m chia ht cho 4.
Ta c th vit n di dng: 3 4 n a b = + , do c th lt c.
Xt trng hp m, n u khng chia ht cho 4. Ta chng minh trng hp ny khng
th lt c. Gi s ngc li, khi m, n u chia ht cho 2 nhng khng chia ht cho 4.
Ta to bt bin nh sau: Xt ( ) ; p q . Nu ch mt trong hai to p, q chia ht cho 4 th
in s 1 vo . Nu c hai to p, q chia ht cho 4 th in s 2. Cc cn li in s
0. Vi cch in s nh vy ta thu c bt bin l tng cc s trong hnh (H1) v tng cc
s trong hnh (H2) u l s l. Do m, n chn nn tng cc s trong ton b hnh ch nht
m n l s chn. lt c th tng s hnh (H1) v (H2) c s dng phi l s chn.
Khi , . mn chia ht cho 24, v l.

3. Bi tp
Bi tp 1. Mt con robot nhy trong mt phng to theo quy tc sau: Xut pht t
im ( ) ; x y , con robot nhy n im ( ) '; ' x y xc nh nh sau:
2
' , ' .
2
x y xy
x y
x y
+
= =
+

Chng minh rng, nu ban u con robot ng im ( ) 2009; 2010 th khng bao gi
con robot nhy vo c trong ng trn (C) c tm l gc to O v bn knh
2840. R =


(H1)

(H2)

=========================================================== 133
HI CC TRNG THTP CHUYN KHU VC DUYN HI V NG BNG BC B

Hi tho khoa hc mn Ton hc ln th III - 2010
Bi tp 2. 6 nh ca mt lc gic li c ghi 6 s chn lin tip theo chiu kim ng
h. Thc hin thut ton sau: mi ln chn mt cnh v cng thm mi s trn cnh vi
cng mt s nguyn no . Hi c nhn c hay khng trng thi m 6 s 6 nh bng
nhau?
Bi tp 3. Mt dy c 19 phng. Ban u mi phng c mt ngi. Sau , c mi
ngy c hai ngi no chuyn sang hai phng bn cnh nhng theo hai chiu ngc
nhau. Hi sau mt s ngy c hay khng trng hp m:
a) Khng c ai phng c th t chn?
b) C 10 ngi phng cui dy?
Bi tp 4 ( thi chn i tuyn Bc Ninh nm 2007)
Trn bn c 2007 vin bi bm 667 bi xanh, 669 bi , 671 bi vng. Thc hin thut ton
sau: Mi ln ly i hai vin bi khc mu v t thm hai vin bi c mu cn li. Hi c th
nhn c trng thi m trn bn ch cn li cc vin bi cng mu c khng?
Bi tp 5 (VMO 1991). Cho bng 1991 1992. K hiu ( ) ; m n l nm giao ca
hng th m v ct th n. T mu cc ca bng theo quy tc sau:
Ln th nht: T ba : ( ) ( ) ( ) ; , 1; 1 , 2; 2 . r s r s r s + + + +
T ln th hai: mi ln t ng ba cha c mu nm cnh nhau trn cng mt hng
hoc trn cng mt ct.
Hi c th t ht tt c cc ca bng c khng?
Bi tp 6 (VMO 1992). Ti mi nh ca a gic li
1 2 1993
... A A A ta ghi mt du cng
(+) hoc mt du tr (-) sao cho trong 1993 du c c du (+) v du (-). Thc hin vic
thay du nh sau: mi ln thay du ng thi ti tt c cc nh ca a gic cho theo quy
tc:
- Nu du ti
i
A v
1 i
A
+
l nh nhau th du ti
i
A c thay bng du (+).
- Nu du ti
i
A v
1 i
A
+
khc nhau th du ti
i
A c thay bng du (-).
(Quy c:
1994
A l
1
. A )
Chng minh rng, tn ti s 2 k sao cho sau khi thc hin lin tip k ln thay du ta
c a gic
1 2 1993
... A A A m du ti mi nh trng vi du ti chnh nh ngay sau ln
thay du th nht.
Bi tp 7 (Shortlist). Cho k, n l cc s nguyn dng. Xt mt bng vung v hn,
t 3k n qun c trong hnh ch nht 3k n . Thc hin tr chi sau: mi qun c s nhy
ngang hoc dc qua mt k vi n v c cha qun c, n trng k vi n va
nhy qua. Sau khi lm nh trn ta loi b qun c b nhy qua ra khi bn c. Chng
minh rng, vi cch chi trn bng vung s khng bao gi cn li ng mt qun c.
Bi tp 8 (Belarus 1999). Cho bng 7 7 v cc qun c c mt trong ba loi sau:
3 1 , 1 1 v hnh ch L gm 3 . Ngi th nht c v hn qun 3 1 v mt qun hnh
ch L, trong khi ngi th hai ch c duy nht mt qun 1 1 . Chng minh rng
a) Nu cho ngi th hai i trc, anh ta c th t qun c ca mnh vo mt no
sao cho ngi th nht khng th ph kn phn cn li ca bng.
b) Nu cho ngi th nht thm mt qun hnh ch L th bt k ngi th hai t qun
c ca mnh u th ngi th hai cng ph knh c phn cn li ca bn c.

=========================================================== 134
HI CC TRNG THTP CHUYN KHU VC DUYN HI V NG BNG BC B

Hi tho khoa hc mn Ton hc ln th III - 2010
Bi tp 9. Xt bng 9 9 . ( ) ; p q ta vit s: ( ) 9 1 . p q + Thc hin thut ton sau:
mi ln ly ra mt hnh vung 4 4 v tng ng thi cc s trong cc ca hnh vung ny
ln mt n v. Chng minh rng ti mi thi im, c s chung ln nht ca tt c cc s
trong bng lun bng 1.
Bi tp 10. Chia gc vung Oxy thnh li vung n v. Cc hng v cc ct c
nh th t t di ln, t tri sang phi. Ban u, t vo ( ) 1;1 mt vin bi. Thc hin
thut ton sau: mi ln ly ra khi gc vin bi nm ( ) ; p q no m ti cc ( ) 1; p q +
v ( ) ; 1 p q + khng c bi, ng thi thm vo hai ni trn mi mt vin bi. Hi c nhn
c hay khng trng thi m
a) Cc ( ) ( ) ( ) ( ) 1;1 , 1; 2 , 2;1 , 2; 2 u khng c bi?
b) Cc ( ) ( ) ( ) ( ) ( ) ( ) 1;1 , 1; 2 , 2;1 , 2; 2 , 1;3 , 3; 2 u khng c bi?

You might also like